[obm-l] Re: [obm-l] Pesagens ( Balança Eletrônica)

2023-11-19 Por tôpico Ralph Costa Teixeira
Existem 2022*2021/2 possibilidades para as 2 falsas. Qualquer estratégia
que seja criada com k pesagens que dão apenas 2 respostas cada distingue no
máximo dentre 2^k possibilidades. Então devemos ter 2^k >= 2022*2021/2...
hmm, isso daria k como pelo menos 21? Errei algo?

On Sun, Nov 19, 2023, 12:16 Jeferson Almir  wrote:

> Amigos, encontrei como K mínimo o valor 11 mas desconfio que seja menos.
> Se alguém souber uma ideia que acabe o problema serei grato.
>
>
> Em Villa Par todas as moedas autênticas pesam uma quantidade par de gramas
> e todas as moedas falsas pesam uma quantidade impar de gramas.
>
> Se você tiver 2022 moedas entre as quais sabe que exatamente 2 são falsas.
>
> Se tiver uma balança eletrônica que informe apenas se o peso total dos
> objetos colocados nela é par ou impar.
>
> Determine o valor mínimo de k para qualquer estratégia que permita
> identificar as moedas falsas usando a balança  no máximo k vezes.
>
> --
> Esta mensagem foi verificada pelo sistema de antivírus e
> acredita-se estar livre de perigo.

-- 
Esta mensagem foi verificada pelo sistema de antiv�rus e
 acredita-se estar livre de perigo.



Re: [obm-l] x^x^x^x....=2 e x^x^x...=4

2023-11-01 Por tôpico Ralph Costa Teixeira
Oi, Alexandre. Quando a gente escreve uma "pilha" de potências sem
parênteses, a convenção é que ela deve ser calculada "de cima para baixo."
Por exemplo:

2^3^4 = 2^(3^4)=2^81 (convenção usual)
ao invés de
(2^3)^4=2^12 (essa precisa de parênteses ali no 2^3).

No caso, acho que o pessoal falava de
x^x^x^x = x^(x^(x^x)) = coisa complicada que depende do x e que eu não sei
simplificar mais que isso ;D
ao invés de
((x^x)^x)^x = x^(x^3)

Ralph.

On Wed, Nov 1, 2023 at 7:45 PM Alexandre Antunes <
prof.alexandreantu...@gmail.com> wrote:

> Boa noite,
>
> Tem uma coisa que não estou entendendo  ... Enxergo ,  a expressão
> infinita de x elevada a x elevada a x (aplicando a propriedade de potência
> de potência) ... Como segue
>
> x^(x^(n-1)) = 2
> E
> x^(x^(n-1)) = 4
> Com n tendendo a infinito.
>
> log x . log x = log (log 2))/(n-1)
> E
> log x . log x = log (log 4))/(n-1)
>
> Para n tendendo a infinito
>
> log x . log x =0
>
> log^2 x = 0
>
> Tem sentido?!!? Ou viajei?
>
>
> Outra coisa, essas equações soltas, sem algum tipo de restrição do valor
> de x fica um pouco sem rumo!
>
>
>
> Em qua, 1 de nov de 2023 18:37, Pacini Bores 
> escreveu:
>
>> Oi Claudio, mas sabe,  o que mais me incomoda é o fato de que em  lnx =
>> lnL/L, se tomarmos a função g(L) = lnL/L , teremos  0< g(L) <= 1/e. Para
>> um único valor de "x" temos dois valores para L e, daí reforçando ( não sei
>> se estou bobeando em algo) a ideia  de que na hipótese de existir lim
>> a(n+1) = lim a(n) = L ,e se tomarmos  L=15 por exemplo , teremos um único
>> "x" no intervalo em que colocastes anteriormente. No Wolfram ou geogebra
>> fui fazendo f(x)= x^x^x... com o aumento na quantidade  de"x" , o gráfico
>> me pareceu crescente a partir de um certo momento e tendo sempre uma reta
>> paralela ao eixo horizontal intersectando sempre o gráfico de "f(x)" . Ou
>> seja, aquele fato de que x^x^x...=4 e dizer que é impossível me causou
>> estranheza. Desculpem se estou cometendo erros conceituais, mas de qualquer
>> forma agradeço a  atenção de todos.
>>
>> Pacini
>>
>> Em qua., 1 de nov. de 2023 às 16:17, Claudio Buffara <
>> claudio.buff...@gmail.com> escreveu:
>>
>>> Dando um Google em x^x^x, eu achei sites que NADA tinham a ver com este
>>> problema...
>>> Mas procurando um pouco mais, achei a afirmação (sem demonstração) de
>>> que a sequência converge para e^(-e) <= x <= e^(1/e).
>>> Explorando numericamente, me convenci de que isso está (provavelmente)
>>> correto.
>>> Ou seja, dado x naquele intervalo, existe L tal que x^L = L
>>> Em particular, L = 1/e ==> (e^(-e))^(1/e) = 1/e,  e  L = e ==>
>>> (e^(1/e))^e = e.
>>> Ou seja, minha conjectura é: a função f é crescente, tem domínio
>>> [e^(-e),e^(1/e)] e imagem [1/e,e].
>>>
>>> []s,
>>> Claudio.
>>>
>>> On Wed, Nov 1, 2023 at 1:21 PM Claudio Buffara <
>>> claudio.buff...@gmail.com> wrote:
>>>
 A ideia me parece ser definir a sequência (a(n)) por:
 a(0) = x   e   a(n+1) = x^a(n)
 e daí ver para que valores de x ela converge e, se convergir, para qual
 limite.

 Se a(n) convergir para L, então  x^L = L.

 Com L = 2 e L = 4, x^L = L implica que x = raiz(2).

 Explorando numericamente com uma planilha, eu noto que para x =
 raiz(2), a sequência parece convergir para 2.

 O problema pode ser reformulado como sendo o de obter o maior intervalo
 I de R para o qual é possível definir uma função f:I -> R tal que f(x) =
 limite da sequência (a(n)) acima com valor inicial a(0) = x.
 Daí, a análise informal acima sugere que raiz(2) pertence a I,
 f(raiz(2)) = 2, e 4 não pertence a f(I).

 O que você está dizendo é que e^(1/e) = sup(I).  Vamos ver...

 Se f(x) = L, então x^L = L ==> x = L^(1/L).
 Agora, a função g(L) = L^(1/L) atinge seu valor máximo, igual a
 e^(1/e), para L = e.
 ( g(L) = e^log(L^(1/L)) = e^(log(L)/L) ==> g'(L) = g(L)*(1 -
 log(L))/L^2 = 0 para L = e )
 Assim, se f(x) está definida, deve ser x <= e^(1/e).
 Além disso, numericamente parece plausível que f(e^(1/e)) = e.
 Se este for o caso, então, dado que e < 4, realmente 4 não pertence à
 imagem de f.

 []s,
 Claudio.



 On Wed, Nov 1, 2023 at 8:47 AM Pacini Bores 
 wrote:

> Olá pessoal, gostaria da opinão de vocês com relação a essas duas
> equações, em que ambas , é claro garantindo a convergência, temos a mesma
> resposta para "x". O que muitos falam que a segunda igualdade não é
> possível. O que me intriga é que é possível mostrar( se não estiver
> errado), é que o "x"  é que varia entre "0" e  " e^(1/e)" para que a
> igualdade x^x^x..=k(k>0) e não o "k". Ou seja, há dois valores possíveis
> para "k", enquanto há apenas um valor para "x".
>
> A minha pergunta : Estou errando em algo ?
>
> Pacini
>
> --
> Esta mensagem foi verificada pelo sistema de antivírus e
> acredita-se estar livre de perigo.


>>> --
>>> Esta 

Re: [obm-l] f(f(x))=e^(-x)

2023-09-23 Por tôpico Ralph Costa Teixeira
P.S.: Existe um argumento simples para mostrar que NÃO existe *f:R->R*
*contínua* com f(f(x))=g(x) que serve para qualquer g estritamente
decrescente (como esta g(x)=e^(-x)). Funciona assim:

i) f teria que ser bijetiva. Afinal, f(a)=f(b) implica f(f(a))=f(f(b)) e,
daqui (g bijetiva) vem a=b.
ii) Mas f bijetiva continua em R implica f (estritamente) monótona!
iiia) se f (estritamente) crescente, absurdo, pois f(f(x))=g(x) seria
crescente;
iiib) se f (estritamente) decrescente, absurdo, pois f(f(x))=g(x) seria
crescente de novo!

Ralph.

On Sat, Sep 23, 2023 at 9:03 PM Ralph Costa Teixeira 
wrote:

> Tecnicamente esta f existe: você pode tomar f:{a}->{a} dada por f(a)=a
> onde a=LambertW(1)~0,56714...  (a raiz de e^(-x)=x).  ;D ;D ;D
>
> Ou melhor dizendo: o problema fala algo sobre o domínio dessa f? Ou dela
> ser contínua, pelo menos?
>
>
> On Sat, Sep 23, 2023 at 8:25 PM Luís Lopes  wrote:
>
>> Saudações,
>>
>> Existe tal f? Se sim, qual seria?
>>
>> Recebi um e-mail com esta pergunta, sem maiores detalhes. Pelo e-mail,
>> tal f não existe. Problema encontrado pelo remetente no YouTube.
>>
>> Luís
>>
>>
>> --
>> Esta mensagem foi verificada pelo sistema de antivírus e
>>  acredita-se estar livre de perigo.
>>
>>
>> =
>> Instruções para entrar na lista, sair da lista e usar a lista em
>> http://www.mat.puc-rio.br/~obmlistas/obm-l.html
>> =
>>
>

-- 
Esta mensagem foi verificada pelo sistema de antiv�rus e
 acredita-se estar livre de perigo.



Re: [obm-l] f(f(x))=e^(-x)

2023-09-23 Por tôpico Ralph Costa Teixeira
Tecnicamente esta f existe: você pode tomar f:{a}->{a} dada por f(a)=a onde
a=LambertW(1)~0,56714...  (a raiz de e^(-x)=x).  ;D ;D ;D

Ou melhor dizendo: o problema fala algo sobre o domínio dessa f? Ou dela
ser contínua, pelo menos?


On Sat, Sep 23, 2023 at 8:25 PM Luís Lopes  wrote:

> Saudações,
>
> Existe tal f? Se sim, qual seria?
>
> Recebi um e-mail com esta pergunta, sem maiores detalhes. Pelo e-mail, tal
> f não existe. Problema encontrado pelo remetente no YouTube.
>
> Luís
>
>
> --
> Esta mensagem foi verificada pelo sistema de antivírus e
>  acredita-se estar livre de perigo.
>
>
> =
> Instruções para entrar na lista, sair da lista e usar a lista em
> http://www.mat.puc-rio.br/~obmlistas/obm-l.html
> =
>

-- 
Esta mensagem foi verificada pelo sistema de antiv�rus e
 acredita-se estar livre de perigo.



Re: [obm-l] problema de probabilidade

2022-11-08 Por tôpico Ralph Costa Teixeira
Mis ou menos... O que faltou foi a hipótese exata da distribuição de
probabilidade dos aniversários.

Se a gente supõe que cada mês tem os mesmos 1/12 de chance para cada aluno,
e que os meses são independentes entre si, sim, p=12/12^2=1/12~8.3%.

Agora, talvez um modelo um pouco mais preciso seria supor que cada DIA do
ano tem a mesma probabilidade (e que são independentes entre si). Isto
afeta um tiquinho a resposta, porque cada mes têm um número ligeiramente
diferente de dias! Ignorando anos bissextos (huh!?!), temos:
-- 7 meses com 31 dias;
-- 4 meses com 30 dias;
-- 1 mes com 28 dias;
Portanto, seria um pouco mais "realista" usar:
p=(7*31^2+4*30^2+28^2)/(365^2) ~ 8.34003%

Eu ponho esse "realista" bem entre aspas; primeiro, porque eu ignorei
anos bissextos (fique à vontade para inclui-los e refazer a conta :D :D
:D); mas a hipótese de que todos os dias do ano tem a mesma probabilidade
não é tão realista quanto parece! Existe uma certa "concentração" de
aniversários em determinadas épocas do ano... mas, sem dados exatos sobre
como seja a tal concentração, o melhor que podemos fazer seria uma das
estimativas acima.

Ainda tem um segundo problema sutil: *mesmo que todos os dias tivessem a
mesma probabilidade, talvez n*ã*o seja 100% correto supor que os
aniversários dos alunos da mesma turma do CMBel sejam independentes*! Por
exemplo, existe uma probabilidade maior que zero de ter gêmeos numa mesma
turma (comum uma família com gêmeos colocá-los na mesma escola), o que
afeta a independência dos dados, e muda um pouquinho aqueles 8.3% (para
cima)... sem uma estimativa desta probabilidade de ter gêmeos na mesma
turma, não conseguimos calcular a resposta "exata".

Isto tudo dito... em quase qualquer problema de probabilidade a gente vai
ter que fazer ALGUMA hipótese simplificadora para poder sair do lugar.
Assim, eu diria que o problema não está 100% bem posto, mas não acho
ridículo fazer uma das hipóteses simplificadoras acima que levam a 8.3%
ou 8.34003% (e a diferença me parece tão pequena que eu aceitaria ambas as
respostas como corretas, desde que as hipóteses utilizadas em cada caso
fossem citadas).

Abraço, Ralph.

On Tue, Nov 8, 2022 at 3:07 PM Luis Paulo  wrote:

> Prezados, o problema abaixo está bem posto?
>
> Uma turma do CMBel tem 25 alunos. Escolhendo-se aleatoriamente dois
> estudantes dessa turma, qual a probabilidade de eles façam aniversário no
> mesmo mês?
>
> A resposta da banca: 1/12.
>
>
>
> --
> Esta mensagem foi verificada pelo sistema de antivírus e
> acredita-se estar livre de perigo.

-- 
Esta mensagem foi verificada pelo sistema de antiv�rus e
 acredita-se estar livre de perigo.



Re: [obm-l] Ajuda em Repunits

2022-07-09 Por tôpico Ralph Costa Teixeira
Argh, corrigindo um detalhe ali perto do fim:
-- Sabemos que 10^q*B-B=r/10^w, portanto 9*(111...)**x**10^w = r*n.
Novamente, como n é primo com 2, 3 e 5 *e x*, conclui-se que n divide
111 (com q 1's), e portanto q>=p=k.

On Sun, Jul 10, 2022 at 1:24 AM Ralph Costa Teixeira 
wrote:

> A chave: *os "restos parciais" que aparecem são exatamente os restos que
> x, 10x, 100x,  deixam na divisão por n.*
> ---///---
>
> MAIS SPOILERS ABAIXO
>
>
> ...
>
>
> 
>
>
> ...
>
>
> 
>
> Acho que facilita bastante pensar no "período" de 1/n de outro jeito:
> ---///---
> LEMA:
> (i) Dado n não divisível por 2 ou 5, existe algum número da forma
> 111...111 que é múltiplo de n.
> (ii) Se n não for divisível por 2, 3 ou 5, o *menor* número do tipo
> 111...111 que é múltiplo de n tem k dígitos, onde k é exatamente o tamanho
> do período (fundamental) da dízima em 1/n.
> PROVA:
>
> (i) Olhe os restos de 1, 11, 111, , ... na divisão por n. São n
> possibilidades, de 0 a n-1, então alguma hora algum resto tem que repetir.
> Isto significa que .. (com A dígitos) e 11...111 (com B dígitos,
> B ...1110 (A 1's e B 0's) = 111 * (10^B) é múltiplo de n. Mas
> n não tem fator comum com aquele 10^B (pois não é divisível por 2 nem por
> 5), portanto ...111 (com k=A-B dígitos) é divisível por n.
>
> (ii) Denote por P=111111 (com p dígitos) o menor daqueles caras com
> apenas "1s" que é múltiplo de n, e denote por k o "período fundamental" na
> dízima de 1/n.
> Por um lado, como 9P=999=10^p-1 é múltiplo de n, temos 10^p *
> (1/n) - 1/n inteiro. Mas isso significa que a parte decimal de 1/n "se
> repete" de p em p dígitos, ou seja, que a dízima de 1/n tem período p. Em
> particular, p>=k.
> Por outro lado, sendo k o período fundamental, temos 10^k * (1/n) - (1/n)
> com número finito de casas decimais, ou seja, (10^k-1)/n = m/10^z com m
> inteiro, e z=número de casas decimais que "sobraram". Mas daqui vem
> 9*(111...111)*10^z = m*n (com k dígitos 1s). Como n é primo com 2, 3 e 5,
> conclui-se que 111... (k 1's) tem que ser múltiplo de n, e portanto
> k>=p.
>
> Note um efeito colateral disso tudo: provamos que 10^k*(1/n)- 1/n =
> 10^p*(1/n)-1/n = inteiro. Assim aquele z vale 0, ou seja, não tem "casas
> decimais que sobram" -- a dízima periódica do 1/n se inicia logo no
> primeiro dígito!
>
> ---///---
> Agora fica tudo bem simples:
> a) Na notação acima, provamos que k=p, e n divide 111 com p
> dígitos.
> b) Seja q o período (fundamental) da dízima de B=x/n irredutível.
>
> Em primeiro lugar, provemos que q=k. Basicamente repetimos o que fizemos
> no lema:
> -- Sabemos que 10^q*B-B=r/10^w, portanto 9*(111...)*10^w = r*n.
> Novamente, como n é primo com 2, 3 e 5, conclui-se que n divide 111
> (com q 1's), e portanto q>=p=k.
> -- Por outro lado, como (10^k-1)/n é inteiro, (10^k-1)*x/n=10^k*B-B também
> é inteiro, ou seja, a dízima de B tem período k (e se inicia no primeiro
> dígito!). Portanto k>=q.
>
> *Enfim, note que os tais "restos parciais" que aparecem são exatamente os
> restos que x, 10x, 100x, , 10^q.x deixam na divisão por n. *A soma
> desses caras vale (...)*x, que é divisível por n pois temos ali
> q=k=p dígitos 1. Por isso, ao dividir esses restos parciais por n, a soma
> dos novos restos tem que ser múltiplo de n tambem.
>
> Foi?
>
>
> On Sat, Jul 9, 2022 at 7:16 PM Rubens Vilhena Fonseca <
> rubens.vilhen...@gmail.com> wrote:
>
>> Gostaria de uma demonstração para o seguinte teorema.
>> *Teorema*. Seja n um inteiro positivo não divisível por 2, 3 ou 5, e
>> suponha que a expansão decimal de l/n tenha período k. Então n é um fator
>> do inteiro 111 ... 11 (k 1 's). Além disso, a soma dos restos parciais na
>> divisão obtida de cada fração irredutível x/n é um múltiplo de n.
>> Comentário:
>> Pelo que entendi, se 1/13  tem período k =6. Então 13  divide 11 (
>> k=6 1's).
>> Essa parte consegui provar.
>> Quanto à segunda parte  para 1/13 os resto da divisão sem repetição são
>> {10, 9, 12, 3, 4, 1}. Então 10+9+12+3+4+1= 13q . (Não soube provar)
>> Não consigo organizar uma sequência  de passos para a demonstração
>> dos dois fatos.
>> Agradeço qualquer ajuda.
>> [[ ]]'s
>>
>> --
>> Esta mensagem foi verificada pelo sistema de antivírus e
>> acredita-se estar livre de perigo.
>
>

-- 
Esta mensagem foi verificada pelo sistema de antiv�rus e
 acredita-se estar livre de perigo.



Re: [obm-l] Ajuda em Repunits

2022-07-09 Por tôpico Ralph Costa Teixeira
A chave: *os "restos parciais" que aparecem são exatamente os restos que x,
10x, 100x,  deixam na divisão por n.*
---///---

MAIS SPOILERS ABAIXO


...





...




Acho que facilita bastante pensar no "período" de 1/n de outro jeito:
---///---
LEMA:
(i) Dado n não divisível por 2 ou 5, existe algum número da forma 111...111
que é múltiplo de n.
(ii) Se n não for divisível por 2, 3 ou 5, o *menor* número do tipo
111...111 que é múltiplo de n tem k dígitos, onde k é exatamente o tamanho
do período (fundamental) da dízima em 1/n.
PROVA:

(i) Olhe os restos de 1, 11, 111, , ... na divisão por n. São n
possibilidades, de 0 a n-1, então alguma hora algum resto tem que repetir.
Isto significa que .. (com A dígitos) e 11...111 (com B dígitos,
B=k.
Por outro lado, sendo k o período fundamental, temos 10^k * (1/n) - (1/n)
com número finito de casas decimais, ou seja, (10^k-1)/n = m/10^z com m
inteiro, e z=número de casas decimais que "sobraram". Mas daqui vem
9*(111...111)*10^z = m*n (com k dígitos 1s). Como n é primo com 2, 3 e 5,
conclui-se que 111... (k 1's) tem que ser múltiplo de n, e portanto
k>=p.

Note um efeito colateral disso tudo: provamos que 10^k*(1/n)- 1/n =
10^p*(1/n)-1/n = inteiro. Assim aquele z vale 0, ou seja, não tem "casas
decimais que sobram" -- a dízima periódica do 1/n se inicia logo no
primeiro dígito!

---///---
Agora fica tudo bem simples:
a) Na notação acima, provamos que k=p, e n divide 111 com p dígitos.
b) Seja q o período (fundamental) da dízima de B=x/n irredutível.

Em primeiro lugar, provemos que q=k. Basicamente repetimos o que fizemos no
lema:
-- Sabemos que 10^q*B-B=r/10^w, portanto 9*(111...)*10^w = r*n.
Novamente, como n é primo com 2, 3 e 5, conclui-se que n divide 111
(com q 1's), e portanto q>=p=k.
-- Por outro lado, como (10^k-1)/n é inteiro, (10^k-1)*x/n=10^k*B-B também
é inteiro, ou seja, a dízima de B tem período k (e se inicia no primeiro
dígito!). Portanto k>=q.

*Enfim, note que os tais "restos parciais" que aparecem são exatamente os
restos que x, 10x, 100x, , 10^q.x deixam na divisão por n. *A soma
desses caras vale (...)*x, que é divisível por n pois temos ali
q=k=p dígitos 1. Por isso, ao dividir esses restos parciais por n, a soma
dos novos restos tem que ser múltiplo de n tambem.

Foi?


On Sat, Jul 9, 2022 at 7:16 PM Rubens Vilhena Fonseca <
rubens.vilhen...@gmail.com> wrote:

> Gostaria de uma demonstração para o seguinte teorema.
> *Teorema*. Seja n um inteiro positivo não divisível por 2, 3 ou 5, e
> suponha que a expansão decimal de l/n tenha período k. Então n é um fator
> do inteiro 111 ... 11 (k 1 's). Além disso, a soma dos restos parciais na
> divisão obtida de cada fração irredutível x/n é um múltiplo de n.
> Comentário:
> Pelo que entendi, se 1/13  tem período k =6. Então 13  divide 11 ( k=6
> 1's).
> Essa parte consegui provar.
> Quanto à segunda parte  para 1/13 os resto da divisão sem repetição são
> {10, 9, 12, 3, 4, 1}. Então 10+9+12+3+4+1= 13q . (Não soube provar)
> Não consigo organizar uma sequência  de passos para a demonstração
> dos dois fatos.
> Agradeço qualquer ajuda.
> [[ ]]'s
>
> --
> Esta mensagem foi verificada pelo sistema de antivírus e
> acredita-se estar livre de perigo.

-- 
Esta mensagem foi verificada pelo sistema de antiv�rus e
 acredita-se estar livre de perigo.



[obm-l] Re: [obm-l] Re: [obm-l] Re: [obm-l] Re: [obm-l] Questão de probabilidade

2022-06-22 Por tôpico Ralph Costa Teixeira
Ponce está provocando a gente... senti que esta flecha tinha um bocado a
minha direção...  :D :D :D

Olha, tem duas "visões" sobre o que "probabilidade" significa.

A primeira vai na linha de que só podemos falar de probabilidade sobre
coisas que ainda não aconteceram. Vai nessa linha: se os eventos estão no
passado, então já aconteceram, já estão definidos, e não faz sentido dizer
que tinha x de chance de ser assim ou y de ser assado. Se você já jogou a
moeda justa, não é mais 50/50 --  é 100% de ser cara, ou 100% de ser coroa,
dependendo do que ocorreu. Quem pensa assim vai dizer que dado um certo
evento (sempre no futuro), ele tem uma probabilidade dada; se duas pessoas
diferentes derem duas probabilidades diferentes para o mesmo evento, uma
delas errou.

Outra linha diz que podemos falar de probabilidade sempre que houver
incerteza; não interessa o que aconteceu ou o que vai acontecer, o que
interessa é o que você SABE sobre o acontecimento. Se você jogou a moeda
justa mas eu não sei nada mais sobre o lançamento, continua sendo 50/50
**PARA MIM**. Probabilidade passa a ser um conceito sobre INFORMAÇÃO, não
sobre os fatos em si (a probabilidade não está na moeda, está no que você
sabe sobre a moeda). Quem pensa assim vai dizer que a probabilidade do
evento depende não apenas do evento em si, mas da informação que se tem em
mãos. Quem pensa assim admite que duas pessoas diferentes podem dar
probabilidades diferentes ao mesmo evento SE SOUBEREM FATOS DIFERENTES a
respeito do evento, ou seja, probabilidade passa a ser bastante "subjetivo".

Eu talvez tenha descrito mal a primeira interpretação, pois sou ferrenho
defensor da segunda. Ela simplesmente engloba a primeira, porque você pode
ter informação parcial sobre fatos que ocorrem no futuro. E falar de
probabilidade para descrever incerteza presente ou passada é MUITO útil! Eu
quero poder expressar incerteza sobre fatos passados com frases do tipo
"fulano tem x% de probabilidade de ter cometido tal crime", ou "tem y% de
probabilidade de ter petróleo nesse poço", ou "tem z% de chance de eu ter
COVID"... Se você tem uma reação negativa a essas frases, lembre o que elas
realmente significam (na segunda interpretação): claro que ou o cara
cometeu o crime ou não, não faz sentido dizer que ele cometeu o crime x%
das vezes em que fizermos um experimento de ele cometer o crime... mas o
que aquilo significa é "com a informação que eu tenho, numa escala de 0 a
1, eu tenho x/100 de certeza que fulano cometeu o crime". E "certeza
baseada em informações" é sim quantificável -- e satisfaz exatamente as
leis das probabilidades com as quais concordamos. "Subjetivo" não significa
"posso falar qualquer coisa", significa apenas que a conta pode variar de
pessoa para pessoa... mas, de novo, SE ESSAS PESSOAS TIVEREM INFORMAÇÕES
DIFERENTES sobre o evento.

Abraço, Ralph.

On Wed, Jun 22, 2022 at 12:09 PM Rogerio Ponce  wrote:

> Olá Pedro e pessoal da lista!
>
> Segundo a opinião do Pedro, nao faz sentido perguntar qual a probabilidade
> de Jose ter conseguido um 6 ao jogar o dado ontem, pois isso ja' aconteceu,
> e, portanto, ja' esta' definido.
>
> Sera' que e' isso mesmo?
>
> []'s
> Rogerio Ponce
>
>
> On Mon, Jun 20, 2022 at 9:45 PM Pedro José  wrote:
>
>> Eu na minha humilde opinião creio que a probabilidade exista quando pode
>> ser uma coisa ou outra. No caso já é definido o que os animais são. Então
>> já está tudo errado. A questão seria viável se dessem esses limitantes para
>> uma criança que pintaria os desenhos dos animais. Aí sim há probabilidade.
>>
>> Em sáb., 18 de jun. de 2022 03:33, Rogerio Ponce da Silva <
>> abrlw...@gmail.com> escreveu:
>>
>>> Ola' Vanderlei e pessoal da lista!
>>>
>>> Sem perda de generalidade, podemos imaginar que vamos fazer o seguinte:
>>>
>>> - uma pintura preta em um dos caes, escolhido aleatoriamente
>>>
>>> - uma pintura "malhada" em um dos animais, escolhido aleatoriamente
>>> entre os 7 animais nao pintados
>>>
>>> - duas pintura pretas, em dois animais, escolhidos aleatoriamente entre
>>> os 6 animais restantes,
>>>
>>> - quatro pinturas brancas nos 4 animais restantes
>>>
>>>
>>> Analisando a afirmacao 04, por exemplo, verificamos que, no segundo
>>> passo (pintura malhada) existem 4 opcoes de cachorro e 3 opcoes de gato.
>>>
>>> Assim, a probabilidade de haver um cachorro malhado (4/7) e' maior que a
>>> probabilidade de haver um gato malhado (3/7).
>>> Portanto, a afirmacao 04 esta' correta.
>>> (e o gabarito esta' errado).
>>>
>>> []'s
>>> Rogerio Ponce
>>>
>>>
>>>
>>> On Wed, Mar 16, 2022 at 8:08 AM Professor Vanderlei Nemitz <
>>> vanderma...@gmail.com> wrote:
>>>
 Bom dia!
 Na questão a seguir, do vestibular da UEM, penso que o espaço amostral
 tem 105 elementos, pois um cachorro é preto (desconsideramos esse). Porém,
 com esse pensamento, não consigo obter o gabarito, que diz que 02 e 16 são
 corretas.
 Alguém poderia ajudar?
 Muito obrigado!

 *Em um pet shop há 3 gatos e 5 cães. 

Re: [obm-l] Retirar cadastro e recebimento de e-mails

2022-06-14 Por tôpico Ralph Costa Teixeira
Oi, pessoal.

Nao sei se isso ajuda, mas deixa eu tentar: na mensagem que recebi quando
me inscrevi, tem o seguinte:

"Como sair da lista: basta enviar novo e-mail para majord...@mat.puc-rio.br
com o texto:
unsubscribe obm-l
end"

Não sei se estas instruções ainda valem (faz muito tempo!), mas vale a pena
tentar, talvez.

-- Ralph

On Wed, Jun 15, 2022 at 12:33 AM Vitor Sousa 
wrote:

> Por favour, também retire meu email do cadastro de recebimento de emails
>
>
>
> Sent from Mail  for
> Windows
>
>
> --
> *From:* owner-ob...@mat.puc-rio.br  on behalf
> of luizbga18 
> *Sent:* Tuesday, March 10, 2020 11:05:07 PM
> *To:* obm-l@mat.puc-rio.br 
> *Subject:* Re: [obm-l] Retirar cadastro e recebimento de e-mails
>
> O meu também, por favor.
>
>
>
> Enviado do meu smartphone Samsung Galaxy.
>
>  Mensagem original 
> De: Francisco Nazário 
> Data: 10/03/20 19:18 (GMT-03:00)
> Para: obm-l@mat.puc-rio.br
> Assunto: Re: [obm-l] Retirar cadastro e recebimento de e-mails
>
> Eu também! Por favor!
>
> Em ter., 10 de mar. de 2020 às 01:21, Larissa Fernandes <
> larissafernande2010...@gmail.com> escreveu:
>
> Boa tarde,
> solicito que meu e-mail seja retirado do cadastro de recebimento de
> e-mails.
>
> --
> Esta mensagem foi verificada pelo sistema de antivírus e
> acredita-se estar livre de perigo.
>
>
> --
> Esta mensagem foi verificada pelo sistema de antiv�rus e
> acredita-se estar livre de perigo.
> --
> Esta mensagem foi verificada pelo sistema de antiv�rus e
> acredita-se estar livre de perigo.
>
> --
> Esta mensagem foi verificada pelo sistema de antivírus e
> acredita-se estar livre de perigo.
>

-- 
Esta mensagem foi verificada pelo sistema de antiv�rus e
 acredita-se estar livre de perigo.



Re: [obm-l] Irracionalidade

2021-12-27 Por tôpico Ralph Costa Teixeira
Um segmento de reta de comprimento x sempre pode ser preenchido com n
segmentos de reta iguais de comprimento x/n (sem superposição), mesmo que x
seja irracional.

Agora: se o segmento "maior" tiver comprimento x irracional e o segmento
"menor" tiver comprimento y RACIONAL, não podemos preencher o maior com n
cópias do menor, sem superposição.

Afinal, se pudéssemos, teríamos x=ny; mas como y=p/q com p e q inteiros,
viria que x=(np)/q, onde np e q são inteiros. Ou seja, x seria racional.

Era isso?

On Mon, Dec 27, 2021 at 2:01 PM Armando Staib 
wrote:

> Acredito que sim , porque se pudéssemos dividir por n seria um número
> racional.  Concorda?
> São segmentos incomensuráveis.
>
> Se eu estiver errado DESCULPE-ME
>
> Em dom., 26 de dez. de 2021 às 16:14, Israel Meireles Chrisostomo <
> israelmchrisost...@gmail.com> escreveu:
>
>> Uma dada reta tem comprimento irracional então é impossível preenche-la
>> com n segmentos de retas iguais?
>>
>> --
>> Esta mensagem foi verificada pelo sistema de antivírus e
>> acredita-se estar livre de perigo.
>
>
> --
> Esta mensagem foi verificada pelo sistema de antivírus e
> acredita-se estar livre de perigo.

-- 
Esta mensagem foi verificada pelo sistema de antiv�rus e
 acredita-se estar livre de perigo.



[obm-l] Re: [obm-l] Números de tentativas

2021-12-14 Por tôpico Ralph Costa Teixeira
Hm, primeiro precisamos deixar o enunciado mais preciso:

i) Eu preciso apenas DESCOBRIR a senha, ou preciso INSERI-LA no dispositivo?
ii) O dispositivo avisa quando a gente acerta a senha totalmente (acho que
o usual seria "sim")? Ou apenas diz "não"/"quase"?
iii) "Coincidente" significa digito correto na posição correta, ou apenas
"aparece em algum lugar da senha"?
iv) A priori, a senha pode ter dígitos repetidos (acho que o usual seria
"sim")?
v) A senha seria um CONJUNTO de 3 dígitos, ou a ordem importa (acho que o
usual seria "ordem importa")?

Para uma cota inferior (usualmente bem ruinzinha), tem uma ideia que
funciona em vários problemas deste tipo: qualquer algoritmo vai pegar uma
sequência de respostas do dispositivo (digamos, Q="quase", N="nao" e
A="acertou!") e traduzir isso numa possivel senha. Em outras palavras, por
mais complexo que seja o algoritmo, no final das contas ele "gera" uma
grande tabela, algo assim:

Se as respostas forem QQNQNQA, a senha vai ser 127;
Se as respostas forem NNQQNA, a senha vai ser 889;
...
e assim por diante. Por isso, se o número de sequências de letras for MENOR
que o número possivel de senhas, não tem como o algoritmo funcionar
GARANTIDAMENTE -- haverá senhas fora da tabela (ou sequências que levam a
mais de uma senha, evidenciando a falha do algoritmo nesses casos)!

Para ser um pouco mais concreto, vou supor 10^3 possíveis senhas (dígitos
ordenados, com repetição). Vou provar que, neste caso, um algoritmo com 9
tentativas NUNCA descobre a senha -- tem que ser pelo menos 10.

Duas outras observações interessantes:
a) Obviamente, se em algum momento seu algoritmo chega em A, PARE, você
achou a senha correta e **nenhuma das tentativas seguintes te
providencia nenhuma informação adicional**. Se você inventar um algoritmo
doido que continua tentando coisas depois do A, eu posso fazê-lo ficar MAIS
EFICIENTE retirando os passos adicionais; ou seja, fazendo todas as
sequências com terminarem nesse A;
b) Por outro lado, vou supor que você TEM QUE INSERIR a senha correta; ou
seja todas as sequências da sua "tabela" terminam em "A".

Assim, o número MÁXIMO de sequências de letras na sua tabela seria:
Comprimento 1: 1 sequência (a saber, "A")
Comprimento 2: 2 sequências (NA e QA)
Comprimento 3: 4 sequências (NNA, NQA, QNA, QQA)
...
Comprimento 9: 2^8=256 sequências
Total: 511 sequências ("máximo" pois, dependendo do algoritmo, talvez
algumas nunca ocorram). Como são 1000 possíveis senhas, é impossível seu
algoritmo distingui-las todas!



On Mon, Dec 13, 2021 at 10:00 AM Jeferson Almir 
wrote:

> Amigos peço ajuda nessa questão.
>
> Tem uma senha de 3 digitos
> (Qualquer digito  de 0 a 9)
> E nos temos um dispositivo
> Que compara a senha
> Com um número que escolhemos
> E retorna não se tem todos os digitos diferentes da senha
> E retorna quase se tem pelo menos 1 digito coincidente com a senha
> Qual é o menor numero de tentativas que precisamos usar esse dispositivo
> tal que podemos descobrir a senha com certeza, independente de qual ela
> seja?
>
> --
> Esta mensagem foi verificada pelo sistema de antivírus e
> acredita-se estar livre de perigo.

-- 
Esta mensagem foi verificada pelo sistema de antiv�rus e
 acredita-se estar livre de perigo.



Re: [obm-l] OBMEP 2021 - Fase 2 - N3

2021-11-09 Por tôpico Ralph Costa Teixeira
Oi, Matheus.

Concordo, olhando apenas as moedas sob o ponteiro, não dá para reverter mas
olhando as vizinhas, ou seja olhando TODO o sistema, TODAS AS MOEDAS a todo
o tempo, dá sim!

Mais exatamente, posso denotar o estado do sistema assim:

ABC(D*)EFGHIJ

onde cada A, B, C, ... assumem o valor "Cara=0" ou "Coroa=1", e o * marca
onde o ponteiro aponta nesse momento. Ou seja, nesta notação começaria com:

Tempo 0: (1*)1
Tempo 1: 1(1*)0111
Tempo 2: 11(0*)111
Tempo 3: 110(1*)01
Tempo 4: 1101(0*)1
Tempo 5: 11010(1*)0111
...

Pois bem, se no tempo (n+1) for, digamos
ABC(D*)EFGHIJ
entao no tempo n tinha que ser...
AB(C*)DXFGHIJ
onde a unica moeda que eu tenho que descobrir eh X (as outras não mudam de
n para n+1). Mas eu descubro X olhando para **D e E juntas** (nao apenas
uma delas)!

Abraço, Ralph.



On Tue, Nov 9, 2021 at 3:24 PM Matheus Bezerra Luna <
matheusbezerr...@gmail.com> wrote:

> Não é completamente reversível não, vai ter que usar o item C para
> concluir o D. Se num tempo T o ponteiro está em uma cara, no tempo T-1 ele
> poderia estar tanto numa cara (pois então nesse tempo não aconteceu nada e
> a moeda seguinte permanceu cara) ou então coroa (o ponteiro em uma coroa
> sendo a moeda seguinte também coroa)
>
> On Tue, Nov 9, 2021, 13:47 Pedro Júnior 
> wrote:
>
>> Obrigado, Ralph!
>>
>> Em ter., 9 de nov. de 2021 às 13:21, Ralph Costa Teixeira <
>> ralp...@gmail.com> escreveu:
>>
>>> Suponho que (A) e (B) sejam fáceis -- basta seguir o algoritmo na mão e
>>> ver o que acontece.
>>>
>>> Para facilitar a conversa, vou pensar em "tempo" como o número de
>>> movimentos feitos... Ou seja, o tempo 0 corresponde à posição inicial; o
>>> tempo 1 seria logo após o primeiro movimento; etc.
>>>
>>> Para (C), pense assim: se o sistema tem alguma coroa no tempo (n), eu
>>> afirmo que vai ter alguma coroa no tempo (n+1). De fato:
>>> -- Se o ponteiro aponta para uma cara no tempo (n), o sistema não muda,
>>> e a tal coroa continua ali;
>>> -- Se o ponteiro aponta para uma coroa no tempo (n), ESTA coroa vai
>>> ficar presente no tempo (n+1).
>>> Portanto, sempre teremos coroas.
>>> (Talvez seja mais natural pensar assim: como que o sistema passaria de
>>> "ter coroas" para "não ter coroas"? Bom, para ele mudar o ponteiro tem que
>>> apontar para alguma coroa, e esta coroa NÃO MUDA. Ou seja,
>>> impossível passar de "ter coroas" para "não ter coroas".)
>>>
>>> Para (D), note que o sistema tem apenas (2^10) * 10 configurações
>>> possíveis (o número não interessa tanto, o que importa é que é FINITO; note
>>> que incluo ali as posições das moedas E a do ponteiro), enquanto o tempo
>>> avança sempre, então em algum momento alguma configuração vai ter que
>>> repetir.
>>> Mas pense como "desfazer" o último movimento realizado e você vai
>>> perceber que existe apenas um jeito de "voltar no tempo" (deixo para você
>>> descrever exatamente isso)! Ou seja, o sistema é reversível (olhando como
>>> ficou o sistema no tempo (n+1), você consegue deduzir como ele estava no
>>> tempo (n), revertendo o último movimento, de maneira única). Portanto, se o
>>> sistema tinha a mesma configuração nos tempos A e A+T, revertendo os
>>> movimentos, concluímos que vai ter a mesma configuração nos tempos 0 e T;
>>> ou seja, no tempo T tínhamos todas coroas como no tempo 0 (e o ponteiro
>>> apontando para A! Bônus!)
>>>
>>> Abraço, Ralph.
>>>
>>> On Tue, Nov 9, 2021 at 12:22 PM Pedro Júnior <
>>> pedromatematic...@gmail.com> wrote:
>>>
>>>> Olá pessoal, alguém aí conseguiu fazer essa questão da prova da OBMEP
>>>> 2021 N3, fase 2? Se puder, ajuda aí... Valeu!
>>>>
>>>> 6) há 10 moedas em um círculo nomeadas de A a J, inicialmente todas com
>>>> a face coroa virada para cima. No centro desse círculo, há um ponteiro que
>>>> inicialmente aponta para a moeda A. Esse ponteiro se movimenta, girando no
>>>> sentido anti-horário (A->B->C->...->J->A->...). Ao movimentar-se, há duas
>>>> opções:
>>>> •Quando o ponteiro termina o movimento apontando para uma moeda com a
>>>> face coroa virada para cima, a moeda seguinte é virada.
>>>> •Quando o ponteiro termina o movimento apontando para uma moeda com a
>>>> face cara virada para cima, nada acontece.
>>>>
>>>> Há exemplo, no primeiro movimento (de A para B), o ponteiro termina em

Re: [obm-l] OBMEP 2021 - Fase 2 - N3

2021-11-09 Por tôpico Ralph Costa Teixeira
Suponho que (A) e (B) sejam fáceis -- basta seguir o algoritmo na mão e ver
o que acontece.

Para facilitar a conversa, vou pensar em "tempo" como o número de
movimentos feitos... Ou seja, o tempo 0 corresponde à posição inicial; o
tempo 1 seria logo após o primeiro movimento; etc.

Para (C), pense assim: se o sistema tem alguma coroa no tempo (n), eu
afirmo que vai ter alguma coroa no tempo (n+1). De fato:
-- Se o ponteiro aponta para uma cara no tempo (n), o sistema não muda, e a
tal coroa continua ali;
-- Se o ponteiro aponta para uma coroa no tempo (n), ESTA coroa vai ficar
presente no tempo (n+1).
Portanto, sempre teremos coroas.
(Talvez seja mais natural pensar assim: como que o sistema passaria de "ter
coroas" para "não ter coroas"? Bom, para ele mudar o ponteiro tem que
apontar para alguma coroa, e esta coroa NÃO MUDA. Ou seja,
impossível passar de "ter coroas" para "não ter coroas".)

Para (D), note que o sistema tem apenas (2^10) * 10 configurações possíveis
(o número não interessa tanto, o que importa é que é FINITO; note que
incluo ali as posições das moedas E a do ponteiro), enquanto o tempo avança
sempre, então em algum momento alguma configuração vai ter que repetir.
Mas pense como "desfazer" o último movimento realizado e você vai perceber
que existe apenas um jeito de "voltar no tempo" (deixo para você descrever
exatamente isso)! Ou seja, o sistema é reversível (olhando como ficou o
sistema no tempo (n+1), você consegue deduzir como ele estava no tempo (n),
revertendo o último movimento, de maneira única). Portanto, se o sistema
tinha a mesma configuração nos tempos A e A+T, revertendo os movimentos,
concluímos que vai ter a mesma configuração nos tempos 0 e T; ou seja, no
tempo T tínhamos todas coroas como no tempo 0 (e o ponteiro apontando para
A! Bônus!)

Abraço, Ralph.

On Tue, Nov 9, 2021 at 12:22 PM Pedro Júnior 
wrote:

> Olá pessoal, alguém aí conseguiu fazer essa questão da prova da OBMEP 2021
> N3, fase 2? Se puder, ajuda aí... Valeu!
>
> 6) há 10 moedas em um círculo nomeadas de A a J, inicialmente todas com a
> face coroa virada para cima. No centro desse círculo, há um ponteiro que
> inicialmente aponta para a moeda A. Esse ponteiro se movimenta, girando no
> sentido anti-horário (A->B->C->...->J->A->...). Ao movimentar-se, há duas
> opções:
> •Quando o ponteiro termina o movimento apontando para uma moeda com a face
> coroa virada para cima, a moeda seguinte é virada.
> •Quando o ponteiro termina o movimento apontando para uma moeda com a face
> cara virada para cima, nada acontece.
>
> Há exemplo, no primeiro movimento (de A para B), o ponteiro termina em B,
> e assim, vira-se a moeda C, que fica com a face cara para cima.
>
> Letra A) o que acontece com as moedas C e D após o segundo movimento?
>
> Letra B) Depois do 12º movimento, quais moedas estão com a face coroa
> virada para cima?
>
> Letra C) mostre que é impossível que, após certo número de movimentos,
> todas as moedas fiquem com a face cara para cima.
>
> Letra D) Mostre que, após um certo número de movimentos, todas as moedas
> voltarão a ficar com a face coroa para cima.
>
>
>
> --
> Esta mensagem foi verificada pelo sistema de antivírus e
> acredita-se estar livre de perigo.

-- 
Esta mensagem foi verificada pelo sistema de antiv�rus e
 acredita-se estar livre de perigo.



[obm-l] Re: [obm-l] Álgebra

2021-07-25 Por tôpico Ralph Costa Teixeira
Sem ser muito formal: (a,b) e (c,d) sao dois vetores do plano, unitários e
ortogonais. Ou seja, um deles eh igual ao outro girado de 90 graus. Assim
(c,d)=(-b,a) ou (c,d)=(b,-a). De um jeito ou de outro, cd=-ab, ou seja,
resposta 0.

On Sun, Jul 25, 2021 at 10:03 AM marcone augusto araújo borges <
marconeborge...@hotmail.com> wrote:

> a, b, c, d são números reais tais que a^2+b^2 = c^2 + d^2 = 1, ac + bd =
> 0. Calcule ab + cd
> Desde já agradeço
>
> --
> Esta mensagem foi verificada pelo sistema de antivírus e
> acredita-se estar livre de perigo.
>

-- 
Esta mensagem foi verificada pelo sistema de antiv�rus e
 acredita-se estar livre de perigo.



Re: [obm-l] probabilidade condicional

2021-06-24 Por tôpico Ralph Costa Teixeira
Sim, são falsas!

Seu exemplo mata o problema! Seus eventos A e B são independentes, mas:

Em (1), P (A | B e C) = 0, enquanto P(A | C) = 1/2.
Em (2), P(A e B | C) = 0, enquanto P(A | C) = P (B | C) = 1/2.

Em suma, quando uma nova informação (C) chega, eventos (A) e (B) que eram
independentes podem deixar de sê-lo!

Abraco, Ralph.

On Thu, Jun 24, 2021 at 9:57 PM Daniel Jelin  wrote:

> Caros, duas dúvidas elementares sobre probabilidade condicional, quem sabe
> possam me ajudar. Leio, em mais de um lugar, que:
>
> 1) Se A e B são independentes, então P(A | B e C) = P (A | C)
>
> A explicação parece fazer sentido: se A não depende de B, tanto que faz
> que B seja dado ou não.
>
> Em conexão com esse problema, leio também que:
>
> 2) Se A e B são independentes, então P(A e B | C)=P(A | C)*P(B | C).
>
> A explicação, que tb parece boa, é que se P(A e B)=P(A)*P(B), então
> podemos "condicionar" toda a igualdade a C, e ela ainda será verdadeira.
>
> Tenho tentado demonstrar essas afirmações, usando Bayes, mas não chego a
> lugar nenhum... Além disso, penso que haja contra-exemplos simples pra
> essas duas afirmações. Por exemplo: lanço dois dados e faço A={o primeiro
> dado é par}, B={o segundo dado é par}, C={a soma dos dois dados é ímpar}. O
> que acontece aqui? Essas afirmações fazem mesmo sentido?
>
> abs
>
> --
> Esta mensagem foi verificada pelo sistema de antivírus e
> acredita-se estar livre de perigo.

-- 
Esta mensagem foi verificada pelo sistema de antiv�rus e
 acredita-se estar livre de perigo.



Re: [obm-l] Probabilidade

2021-04-23 Por tôpico Ralph Costa Teixeira
Ah, Pacini, você levanta um ponto interessante...

Primeiro, deixa eu esclarecer: eu usei p(n) = Pr (A vai vencer o jogo | A
tem n pontos a mais do que B agora); ou seja, não seria exatamente o que
você interpretou ali.

Daqui meu argumento de simetria: a partir do momento em que A tem 0 pontos
a mais do que B, ou seja, eles estão empatados, o jogo é completamente
simétrico, ou seja, eu posso permutar A e B sem alterar nenhuma
probabilidade. Por isso eu digo que:

p(0) = Pr (A vencer | empatados agora) = Pr (B vencer | empatados agora)

Aqui entra o seu ponto interessante: É POSSÍVEL QUE ESTE JOGO CONTINUE PARA
SEMPRE, SEM QUE HAJA VENCEDOR. De fato, se os lançamentos a partir de agora
forem CKCKCKCK..., o jogo nunca termina.

Entao eu deveria escrever Pr (A vencer | empatados agora) + Pr (B vencer |
empatados agora) + Pr (jogo nunca terminar | empatados agora) = 1. Para eu
poder afirmar que os dois primeiros termos valem 1/2, **eu tenho que te
convencer primeiro que o terceiro termo vale 0**.

Bom, vale 0 sim, mas eu usei isso baseado em experiência prévia com este
tipo de experimento; por exemplo, sei que:

---///---
LEMA: Lance uma moeda infinitas vezes, onde cada lançamento é independente
dos outros e tem probabilidade p de dar "Cara" e 1-p de dar "Koroa", com
0Inf, isso vai para 0, portanto a probabilidade de fracasso nos
"infinitos" lançamentos vale 0.
---///---

O que isso tem a ver com nosso problema? No nosso problema, note que se
tivermos 7 lances consecutivos onde A marca ponto mas B não (deixa eu
chamar isso de "cara"), certamente A vai vencer em algum momento desta
sequência.

Assim, "jogo nunca terminar" IMPLICA "nunca existe uma sequência de 7
caras". Portanto:
Pr (jogo não terminar) <= Pr(nunca ter sequência com 7 "caras") = 0
e assim eu posso completar o argumento que eu usei, afirmando que p(0)=1/2.
Ufa!

(Note que este argumento vale mesmo no caso em que cada "lance" tem 4
opções (1,0); (0,1); (0,0); (1,1) para o número de pontos que A e B ganham;
aqui teríamos p("cara")=1/4, continua valendo!)

---///---

Enfim, antes que alguém estranhe isso, deixa eu explicitar algo que pode
parecer estranho:
-- SIM, é possível que o jogo nunca termine...
-- ...e a probabilidade disso acontecer vale 0.
Os axiomas da probabilidade dizem que Pr(vazio)=0; SE um evento é
impossível ENTÃO ele tem probabilidade 0. Mas nunca dizem a volta disso!
Podemos ter Pr(A)=0 sem ter A=vazio nem impossível! Eventos POSSÍVEIS podem
ter probabilidade 0 sim senhor.
Exemplo simples: jogando uma moeda justa infinitas vezes, qual a
probabilidade de todas as vezes darem cara? Reposta: ZERO. PODE
acontecer... mas, huh, eu não apostaria nisso. :D
Pior: eventos de probabilidade 0 ACONTECEM. Exemplo: jogue a moeda
infinitas vezes, anote a sequência exata que saiu, na ordem. A
probabilidade de sair exatamente esta sequência era ZERO antes de você
fazer o experimento... mas aconteceu. :P





On Fri, Apr 23, 2021 at 9:48 AM Pacini Bores  wrote:

> Desculpe  Ralph,
>
> O que não ficou claro pra mim  foi o fato de que p(0) =1/2 , já que p(0)
> traduz a probabilidade de de ficar com diferença de  zero ponto  agora ou
> depois, ou seja, partindo de zero ponto de diferença entre os dois
> jogadores, poderia ficar assim a vida toda, não ? Em que estou pensando
> errado.
>
> Agradeço desde já ( acho que tenho que estudar mais)
>
> Pacini
>
> Em 03/04/2021 18:08, Ralph Costa Teixeira escreveu:
>
> Vou dizer que "o jogo está na posicao n" quando A tem n pontos de
> vantagem; e vou chamar de p(n) a probabilidade de A vencer o jogo (agora ou
> depois) sabendo que (agora) A tem n pontos mais do que B.
>
> Por exemplo, p(3)=1, p(-3)=0 e p(0)=1/2 (por simetria).
>
> Aliás, por simetria, vemos que p(1)=1-p(-1) e p(2)=1-p(-2). Vou chamar
> a=p(1) e b=p(2) para facilitar a escrita (o "p(n)" seria util para jogos
> maiores, quando a gente escreveria tudo em forma matricial -- mas aqui nem
> vou precisar).
>
> A partir da posicao 1, no próximo "lance", temos 50% de chance de ir para
> 2 (e dali chance b de A ganhar) e 50% de chance de ir para 0 (e dali chance
> 50% de A ganhar). Portanto:
>
> a= 1/2 . b + 1/2. 1/2
>
> Analogamente, a partir de 2, temos 50% de chance de ir para 1 e 50% de
> chance de termos vitória de A, portanto:
>
> b=1/2 + 1/2.a
>
> Resolvendo o sistema, vem a=2/3  e b = 5/6. Resposta (B)?
>
> Abraco, Ralph.
>
> P.S.: Em geral seria : p(n)=p_A . p(n+1) + (1-p_A) . p(n-1), e as regras
> sobre a vitória determinam "condições de contorno". Ou seja, considerando o
> vetor v = (p(-m), p(-m+1), ... p(0), ... p(m)), temos v=Mv onde M é uma
> matriz tridiagonal (de fato, com 0s na diagonal). Ou seja, no fundo no
> fundo estamos falando de um problema de achar o autovetor

Re: [obm-l] Probabilidade

2021-04-09 Por tôpico Ralph Costa Teixeira
Primeiro: sim, Albert tem razão, eu assumi que em cada rodada apenas um
entre A e B marcariam pontos, portanto ignorei os casos (A=B), e nada dizia
isso claramente no enunciado.

Mas a conta do Daniel revela que não importa, o que é bem interessante
E, agora, depois de ver a conta, digo: era de se esperar! Afinal, no jogo
"tipo Bouskela", as rodadas onde A e B marcam pontos juntos podem ser
jogadas fora, pois o fim do jogo é determinado por quantos pontos um
jogador tem A MAIS do que o outro, e tais rodadas não tem efeito nenhum
nisso. Como estas rodadas do tipo A=B podem ser jogadas fora, o jogo "tipo
Bouskela" é de fato equivalente ao jogo "tipo Ralph" que eu analisei (bom,
pelo menos com relação a determinar QUEM ganha; se a gente perguntasse algo
do tipo "QUANDO" ganha, os jogos seriam bem distintos).

(Outra coisa: eu tinha dito que achar essas probabilidades era equivalente
a achar os autovetores de uma certa matriz M; note que a minha matriz M tem
probabilidades de transição entre estados. Se a gente incluir as
"transições tipo Bouskela" no jogo, a gente de fato estah colocando algumas
probabilidades p na diagonal principal, e re-escalando correspondentemente
as probabilidades do jogo do "tipo Ralph". Ou seja, estamos trocando M por
X=(1-p).M+p.I. Mas M e X=(1-p).M+pI tem os mesmos autovetores, que eh a
maneira "Algebra Linear" de explicar porque a resposta não muda! :D :D )



On Sat, Apr 10, 2021 at 1:19 AM Daniel Jelin  wrote:

> Me parece que a interpretação dada não muda a resposta, se entendi
> direito. Teríamos: 50% de chance de continuar na mesma posição (ponto pros
> dois ou ponto pra ninguém), 25% de avançar (ponto pra um), 25% de recuar
> (ponto pro adversário). Assim, acho que dá para usar o esquema do Ralph:
> a=(1/4)*b+(1/4)*(1/2)+(1/2)*a
> b=(1/4)+(1/4)*a+(1/2)*b
> E resolvendo, temos os mesmos a=2/3 e b=5/6.
>
> Ainda que as probabilidades de fazer e de não fazer o ponto fossem
> diferentes, creio que dá na mesma. Seja x a probabilidade de A fazer 1
> ponto, então, pelo enunciado, x também é a probabilidade de B fazer 1
> ponto. Aí a probabilidade de A não fazer ponto é 1-x, e a de B não fazer
> ponto são os mesmos 1-x. Então:
>
> a=(x)*(1-x)*b + (1-x)*(x)*1/2 + (x)(x)*a+(1-x)*(1-x)*a
> b=(x)*(1-x) + (1-x)*(x)*a + (x)(x)*b+(1-x)*(1-x)*b
> E resolvendo, eliminamos x e voltamos a a=2/3 e b=5/6.
>
> On Thu, Apr 8, 2021 at 8:27 PM  wrote:
>
>> Este é um problema bastante interessante, contudo o seu enunciado, tal
>> como está, apresenta uma falha: - É necessário fixar quais são os
>> resultados possíveis numa determinada rodada do jogo! Dito assim, o
>> enunciado admite, para cada rodada 4 possibilidades: (A=1, B=1); (A=1,
>> B=0); (A=0, B=1); (A=0, B=0).
>>
>>
>>
>> *Albert Bouskelá*
>>
>> bousk...@gmail.com
>>
>>
>>
>> *De:* owner-ob...@mat.puc-rio.br  *Em nome
>> de *Professor Vanderlei Nemitz
>> *Enviada em:* quinta-feira, 8 de abril de 2021 14:34
>> *Para:* OBM 
>> *Assunto:* Re: [obm-l] Probabilidade
>>
>>
>>
>> Muito legal esse tipo de problema.
>>
>> Em que ano caiu, você sabe, Pacini?
>>
>>
>>
>> Em sáb., 3 de abr. de 2021 às 15:22, Pacini Bores 
>> escreveu:
>>
>> Olá pessoal, Encontrei uma resposta que não está entre as opções desta
>> questão do Canguru.
>>
>> " um certo jogo tem um vencedor quando este atinge 3 pontos a frente do
>> oponente. Dois jogadores A e B estão jogando e, num determinado momento, A
>> está 1 ponto a frente de B. Os jogadores  têm probabilidades iguais de
>> obter 1 ponto. Qual a probabilidade de A vencer o jogo ?
>>
>> (A) 1/2   (B) 2/3  (C) 3/4   (D) 4/5  (E) 5/6
>>
>>
>>
>> O que vocês acham ?
>>
>>  Pacini
>>
>>
>>
>>
>> --
>> Esta mensagem foi verificada pelo sistema de antivírus e
>> acredita-se estar livre de perigo.
>>
>>
>> --
>> Esta mensagem foi verificada pelo sistema de antiv?s e
>> acredita-se estar livre de perigo.
>>
>> --
>> Esta mensagem foi verificada pelo sistema de antivírus e
>> acredita-se estar livre de perigo.
>>
>
>
> 
>  Livre
> de vírus. www.avast.com
> .
> <#m_-7953325398812603413_DAB4FAD8-2DD7-40BB-A1B8-4E2AA1F9FDF2>
>
> --
> Esta mensagem foi verificada pelo sistema de antivírus e
> acredita-se estar livre de perigo.

-- 
Esta mensagem foi verificada pelo sistema de antiv�rus e
 acredita-se estar livre de perigo.



Re: [obm-l] Probabilidade

2021-04-03 Por tôpico Ralph Costa Teixeira
Vou dizer que "o jogo está na posicao n" quando A tem n pontos de vantagem;
e vou chamar de p(n) a probabilidade de A vencer o jogo (agora ou depois)
sabendo que (agora) A tem n pontos mais do que B.

Por exemplo, p(3)=1, p(-3)=0 e p(0)=1/2 (por simetria).

Aliás, por simetria, vemos que p(1)=1-p(-1) e p(2)=1-p(-2). Vou chamar
a=p(1) e b=p(2) para facilitar a escrita (o "p(n)" seria util para jogos
maiores, quando a gente escreveria tudo em forma matricial -- mas aqui nem
vou precisar).

A partir da posicao 1, no próximo "lance", temos 50% de chance de ir para 2
(e dali chance b de A ganhar) e 50% de chance de ir para 0 (e dali chance
50% de A ganhar). Portanto:

a= 1/2 . b + 1/2. 1/2

Analogamente, a partir de 2, temos 50% de chance de ir para 1 e 50% de
chance de termos vitória de A, portanto:

b=1/2 + 1/2.a

Resolvendo o sistema, vem a=2/3  e b = 5/6. Resposta (B)?

Abraco, Ralph.

P.S.: Em geral seria : p(n)=p_A . p(n+1) + (1-p_A) . p(n-1), e as regras
sobre a vitória determinam "condições de contorno". Ou seja, considerando o
vetor v = (p(-m), p(-m+1), ... p(0), ... p(m)), temos v=Mv onde M é uma
matriz tridiagonal (de fato, com 0s na diagonal). Ou seja, no fundo no
fundo estamos falando de um problema de achar o autovetor associado ao
autovalor 1 da matriz M, e as condicoes de contorno apenas normalizam v.




On Sat, Apr 3, 2021 at 3:22 PM Pacini Bores  wrote:

> Olá pessoal, Encontrei uma resposta que não está entre as opções desta
> questão do Canguru.
>
> " um certo jogo tem um vencedor quando este atinge 3 pontos a frente do
> oponente. Dois jogadores A e B estão jogando e, num determinado momento, A
> está 1 ponto a frente de B. Os jogadores  têm probabilidades iguais de
> obter 1 ponto. Qual a probabilidade de A vencer o jogo ?
>
> (A) 1/2   (B) 2/3  (C) 3/4   (D) 4/5  (E) 5/6
>
>
>
> O que vocês acham ?
>
>  Pacini
>
>
> --
> Esta mensagem foi verificada pelo sistema de antivírus e
> acredita-se estar livre de perigo.
>

-- 
Esta mensagem foi verificada pelo sistema de antiv�rus e
 acredita-se estar livre de perigo.



[obm-l] Re: [obm-l] Re: [obm-l] Re: [obm-l] Sequência Injetiva

2021-02-13 Por tôpico Ralph Costa Teixeira
Sim, voce tem razao -- eu achei que era a_2n = a_{2n-1} +1. Que pena, era
uma boa questao com Fibonacci. :)

On Sun, Feb 14, 2021 at 12:35 AM Claudio Buffara 
wrote:

> Oi, Ralph:
>
> Eu posso ter entendido errado a definição da sequência, mas achei termos
> diferentes dos seus:
> 1:  1
> 2:  2
> 3:  1/2
> 4:  3
> 5:  1/3
> 6:  3/2
> 7:  2/3
> 8:  4
> 9:  1/4
> 10:  4/3
> 11:  3/4
> 12:  5/2
> 13:  2/5
> 14:  5/3
> 15:  3/5
> 16:  5
> ...
>
> []s,
> Claudio.
>
>
> On Sat, Feb 13, 2021 at 7:59 PM Ralph Costa Teixeira 
> wrote:
>
>> Meio enrolado, vou escrever meio vagamente.
>>
>> Eu sugiro olhar primeiro para os caras com indice impar. Sao eles:
>> a1=1/1
>> a3=1/2
>> a5=2/3
>> a7=3/5
>> a8=5/8
>> ...
>> Ou seja, mostre que eles sao quocientes de numeros de Fibonacci
>> consecutivos (os caras de indice par sao os inversos desses). Agora tem
>> varias maneiras de continuar:
>>
>> -- Voce pode mostrar que os numeros de Fibonacci consecutivos sao primos
>> entre si; portanto cada fracao dessas fica unicamente determinada por
>> numerador e denominador, e (como os numeros de Fibonacci formam uma
>> sequencia crescente) vao ser distintos entre si;
>> -- Se voce nao quiser entrar no merito do Fibonacci, tente mostrar (pode
>> ser por inducao) que a3 < a7 < a11 <...> < ... < a13 < a9 < a5 < 1  (phi ali seria (raiz(5)-1) / 2, acho).
>>
>> De qualquer forma, como a_(2n+1)<1, a1=1 e os "a_2n" sao os inversos dos
>> "a_2n+1, vao ser todos diferentes.
>>
>> Abraco, Ralph.
>>
>>
>> On Sat, Feb 13, 2021 at 5:56 PM Jeferson Almir 
>> wrote:
>>
>>> Amigos, peço ajuda em provar a injetividade dessa sequência que seria
>>> uma saída para provar a unica ocorrência do racional que aparece nela.
>>> Estou andando em círculos tentando montar uma possível indução.
>>>
>>>
>>> Dado a sequência a_1 = 1 e a_2n = a_n  + 1 e a_2n+1 = 1/a_2n.
>>>
>>> Prove que para todo racional positivo que ocorre na sequência, ocorre
>>> uma única vez.
>>>
>>> --
>>> Esta mensagem foi verificada pelo sistema de antivírus e
>>> acredita-se estar livre de perigo.
>>
>>
>> --
>> Esta mensagem foi verificada pelo sistema de antivírus e
>> acredita-se estar livre de perigo.
>
>
> --
> Esta mensagem foi verificada pelo sistema de antivírus e
> acredita-se estar livre de perigo.

-- 
Esta mensagem foi verificada pelo sistema de antiv�rus e
 acredita-se estar livre de perigo.



[obm-l] Re: [obm-l] Sequência Injetiva

2021-02-13 Por tôpico Ralph Costa Teixeira
Meio enrolado, vou escrever meio vagamente.

Eu sugiro olhar primeiro para os caras com indice impar. Sao eles:
a1=1/1
a3=1/2
a5=2/3
a7=3/5
a8=5/8
...
Ou seja, mostre que eles sao quocientes de numeros de Fibonacci
consecutivos (os caras de indice par sao os inversos desses). Agora tem
varias maneiras de continuar:

-- Voce pode mostrar que os numeros de Fibonacci consecutivos sao primos
entre si; portanto cada fracao dessas fica unicamente determinada por
numerador e denominador, e (como os numeros de Fibonacci formam uma
sequencia crescente) vao ser distintos entre si;
-- Se voce nao quiser entrar no merito do Fibonacci, tente mostrar (pode
ser por inducao) que a3 < a7 < a11 <...
wrote:

> Amigos, peço ajuda em provar a injetividade dessa sequência que seria uma
> saída para provar a unica ocorrência do racional que aparece nela. Estou
> andando em círculos tentando montar uma possível indução.
>
>
> Dado a sequência a_1 = 1 e a_2n = a_n  + 1 e a_2n+1 = 1/a_2n.
>
> Prove que para todo racional positivo que ocorre na sequência, ocorre uma
> única vez.
>
> --
> Esta mensagem foi verificada pelo sistema de antivírus e
> acredita-se estar livre de perigo.

-- 
Esta mensagem foi verificada pelo sistema de antiv�rus e
 acredita-se estar livre de perigo.



[obm-l] Re: [obm-l] Re: [obm-l] Re: [obm-l] Função parte inteira

2021-02-03 Por tôpico Ralph Costa Teixeira
Hm, confere o enunciado - era parte inteira, ou inteiro mais proximo?

On Wed, Feb 3, 2021, 18:39 joao pedro b menezes 
wrote:

> Obrigado pela dica! Honestamente creio que existe um erro nesse problema.
> Fazendo alguns casos na mão é possivel perceber que isso sempre resulta em
> 8n + 7. Essa é a prova:
> "Provar que ( n^(1/3) + ( n + 2)^(1/3) )³  < 8n + 8. Abrindo a potência,
> temos:
> 2n + 2 + 3 * ( (n² ( n + 2))^(1/3) + (n(n + 2)²)^(1/3)) < 8n + 8
>   (n² ( n + 2))^(1/3) + (n(n + 2)²)^(1/3)   < 2n + 2
> Porém temos que  (n² ( n + 2))^(1/3) < n + 2/3  , e  (n(n + 2)²)^(1/3) <
> n + 4/3 ( eu testei elevando ambos os lados ao cubo deu certo) . Isso
> confirma a inequação inicial.
> Agora se 8n + 7 <  ( n^(1/3) + ( n + 2)^(1/3) )³  o exercício acaba. De
> fato, trabalhando a expressão:
>(n² ( n + 2))^(1/3) + (n(n + 2)²)^(1/3)   > 2n + 5/3
> Mas novamente, tem se que  (n² ( n + 2))^(1/3) > n + 1/2 e  (n(n +
> 2)²)^(1/3) > n + 7/6 para qualquer n > 1 ( no caso n =1 basta testar na
> mão). E como 1/2 + 7/6 = 5/3 ,  tem se que ela é verdade, logo:
> 8n + 7 <  ( n^(1/3) + ( n + 2)^(1/3) )³  < 8n + 8 ==> [ ( n^(1/3) + ( n +
> 2)^(1/3) )³ ] = 8n + 7"
> Eu estranhei bastante porque nunca tinha acontecido de um exercicio do
> POTI estar errado.
> obs: Se a minha solução estiver errada de alguma forma, adoraria saber!
>
> On Wed, Feb 3, 2021 at 12:42 PM Ralph Costa Teixeira 
> wrote:
>
>> Sem tempo agora, mas olhando por alto eu aproximaria o que estah dentro
>> do () por 2(n+1)^(1/3), o que levaria imediatamente a 8(n+1). Serah que a
>> parte inteira daquela coisa eh 8(n+1)?
>>
>> Entao eu tentaria abrir os cubos, subtrair 8(n+1), e mostrar que o que
>> sobra eh menor que 1.
>>
>> Serah que funciona?
>>
>> On Wed, Feb 3, 2021 at 10:03 AM joao pedro b menezes <
>> joaopedrobmene...@gmail.com> wrote:
>>
>>> Olá, estava tentando fazer esta questão:
>>>   Prove que [ ( n^(1/3) + (n + 2)^(1/3)  )³] é divisível por 8.
>>> obs: não tinha a tecla de função parte inteira, por isso escolhi [ ]
>>> Se alguém tiver alguma dica ou souber como resolver, ajudaria bastante.
>>>
>>


[obm-l] Re: [obm-l] Função parte inteira

2021-02-03 Por tôpico Ralph Costa Teixeira
Sem tempo agora, mas olhando por alto eu aproximaria o que estah dentro do
() por 2(n+1)^(1/3), o que levaria imediatamente a 8(n+1). Serah que a
parte inteira daquela coisa eh 8(n+1)?

Entao eu tentaria abrir os cubos, subtrair 8(n+1), e mostrar que o que
sobra eh menor que 1.

Serah que funciona?

On Wed, Feb 3, 2021 at 10:03 AM joao pedro b menezes <
joaopedrobmene...@gmail.com> wrote:

> Olá, estava tentando fazer esta questão:
>   Prove que [ ( n^(1/3) + (n + 2)^(1/3)  )³] é divisível por 8.
> obs: não tinha a tecla de função parte inteira, por isso escolhi [ ]
> Se alguém tiver alguma dica ou souber como resolver, ajudaria bastante.
>


Re: [obm-l] Limites

2021-01-29 Por tôpico Ralph Costa Teixeira
Oi, João Pedro. Voce sabe Calculo -- em particular, a Regra de L'Hopital?

Isso eh o que os livros chamam de "indeterminação do tipo Infinito^0". Ao
inves de achar o limite desta função, vamos passar o logaritmo primeiro,
lembrando que
ln (1+x)^(1/x) = 1/x * ln(1+x)
ou seja, ache primeiro este limite aqui:
lim x->Inf  (ln(1+x)) /x
Esse é do tipo "Inf/Inf" e sai por L^Hopital (vale 0); portanto o limite
que você pediu vale (para desfazer o logaritmo, que é uma função contínua)
e^0=1, como você suspeitava.

Abraço, Ralph.





On Fri, Jan 29, 2021 at 10:26 PM joao pedro b menezes <
joaopedrobmene...@gmail.com> wrote:

> Olá a todos, boa noite, estou com um pouco de dificuldade em encontrar uma
> prova para esse limite
> lim x-> infinito (1 + x)^(1/x)
> Creio que ele seja 1, mas não conheço nenhuma maneira de provar isso
> Se alguém tiver uma dica ou souber como provar, ajudaria bastante
> Já agradeço pela ajuda :)
>


Re: [obm-l] Amigo secreto ENEM

2021-01-28 Por tôpico Ralph Costa Teixeira
A wikipedia tem um comecinho:
https://pt.wikipedia.org/wiki/Desarranjo
https://en.wikipedia.org/wiki/Derangement
Serve?

On Thu, Jan 28, 2021 at 1:15 PM Luiz Antonio Rodrigues <
rodrigue...@gmail.com> wrote:

> Olá, pessoal!
> Boa tarde!
> Estou acompanhando com interesse a discussão, mas gostaria de pedir uma
> indicação de site ou outro material que trate de permutações caóticas.
> Muito obrigado!
> Abraços!
> Luiz
>
> Em qui, 28 de jan de 2021 11:38 AM, Arthur Queiroz 
> escreveu:
>
>> Uma pergunta: você assume que o número de sorteios é !10. Mas e se, em
>> meio ao sorteio, nossa permutação caótica seja tal que seja formado um
>> ciclo indesejado? Digamos A->B->C->A. Como o sorteio continuará nesse caso?
>> Será escolhida aleatoriamente uma pessoa de fora do ciclo para continuar?
>> Isso não afetaria esse !10?
>>
>> Em ter, 26 de jan de 2021 17:26, Ralph Costa Teixeira 
>> escreveu:
>>
>>> Deixa eu copiar o que escrevi em outro lugar... :D :D
>>>
>>> Primeiro: não fica claro do enunciado se "auto-sorteios" (alguém sortear
>>> o próprio nome) são permitidos ou não, e isto ALTERA a resposta. :(
>>>
>>> Vejamos possíveis respostas corretas:
>>>
>>> ---///---
>>>
>>> SE AUTO-SORTEIOS FOREM PERMITIDOS:
>>> Em resumo, temos 1/10 de chance de A iniciar o sorteio, e 1/10 de chance
>>> de B terminar (1/10 sim, pois A *pode* terminar). Assim, a resposta seria
>>> 1/10*1/10*2=1/50.
>>>
>>> Com mais detalhes para justificar o segundo "1/10":
>>> -- Número de sorteios possíveis = 10!
>>> -- Número de sorteios que formam um único ciclo de tamanho 10 = 9!
>>> -- Note que ter um ciclo de tamanho 10 equivale a terminar com quem
>>> inicia; portanto, a chance de o amigo secreto terminar com o mesmo que
>>> iniciou seria 9!/10!=1/10 (*que é independente de quem começa*).
>>>
>>> Assim:
>>> -- Chance de A iniciar = 1/10;
>>> Agora, DADO QUE A INICIOU:
>>>  Chance de A terminar = 9!/10! = 1/10
>>>  Portanto, chance de não terminar com A: 9/10
>>>  Chance de B terminar (por simetria): (9/10) /9 = 1/10
>>>
>>> Isso nos dá 1/10*1/10 = 1/100 de chance do amigo secreto começar por A e
>>> terminar com B. Portanto a resposta seria o dobro, 1/50.
>>>
>>> ---///---
>>> SE AUTO-SORTEIOS SÃO PROIBIDOS:
>>> -- Número de sorteios (desarranjos) possíveis = !10 (vou escrever K=!10
>>> daqui por diante);
>>> -- Número de sorteios que formam um único ciclo de tamanho 10 = 9!
>>> -- Portanto, a chance de o amigo secreto terminar com o mesmo que
>>> iniciou seria 9!/K (que é independente de quem começa).
>>>
>>> Assim:
>>> -- Chance de A iniciar = 1/10;
>>> Agora, DADO QUE A INICIOU:
>>>  Chance de A terminar = 9!/K
>>>  Portanto, chance de não terminar com A: 1-9!/K
>>>  Chance de B terminar (por simetria): (1-9!/K) /9 = (K-9!)/(9K)
>>>
>>> Isso nos dá 1/10* (K-9!)/(9K) = (K-9!)/(90K) de chance do amigo secreto
>>> começar por A e terminar com B. Portanto a resposta seria o dobro,
>>> (K-9!)/(45K). Fazendo a conta com a ajuda do computador, achei 12001/741645.
>>>
>>> Abraço, Ralph.
>>>
>>>
>>>
>>> On Tue, Jan 26, 2021 at 1:45 PM Professor Vanderlei Nemitz <
>>> vanderma...@gmail.com> wrote:
>>>
>>>> Oi, pessoal!
>>>>
>>>> Com certeza vocês estão acompanhando desde domingo as resoluções da
>>>> questão do ENEM do amigo secreto.
>>>> Além da resposta proposta, *1/45*, que parece não estar correta, já vi
>>>> outras duas, *12001/741645* (ETAPA e ANGLO), que consideram também que
>>>> o sorteio anterior para definir "quem presenteia quem", e *7/360*, do
>>>> vídeo a seguir:
>>>>
>>>> https://www.youtube.com/watch?v=c-t_BAMASKE
>>>>
>>>> Gostaria da opinião (e se possível, uma resolução) dos especialistas da
>>>> lista (Ralph e cia :))
>>>>
>>>> Muito obrigado!
>>>>
>>>>
>>>>
>>>>
>>>>
>>>


[obm-l] Re: [obm-l] Álgebra

2021-01-28 Por tôpico Ralph Costa Teixeira
Ok, vamos escrever a primeira linha como:
a= tb
c=(-1-t)d

A segunda linha diz que t^2.b^2+(1+t)^2.d^2=1, ou seja,
t^2 + 2t.d^2 + d^2 = 1  (**)
(Estou tentando botar tudo em termos de t e d!)

Agora: b^3/a + d^3/c = b^2/t - d^2/(1+t) = (1-d^2)/t - d^2/(1+t) =
= (1-2t.d^2 +t -d^2) / (t^2+t)

Use (**) para trocar o 1-2t.d^2-d^2 do numerador por t^2 e foi! :D

Abraço, Ralph.

On Thu, Jan 28, 2021 at 12:04 PM Professor Vanderlei Nemitz <
vanderma...@gmail.com> wrote:

> Oi, pessoal, tudo bem?
> Tentei algumas coisas nesse problema, enxergar a, b, c, d como senos e
> cossenos ou utilizar números complexos, mas não obtive êxito.
> A resposta é 1.
> Para casos particulares é fácil chegar nesse valor.
>
> Se alguém resolver, agradeço muito!
>
> a/b + c/d = –1
> a^2 + c^2 = 1
> b^2 + d^2 = 1
> Calcule b^3/a + d^3/c.
>
> Imagino que a, b, c, d são reais, certo? Nada é dito...
>
>


Re: [obm-l] Amigo secreto ENEM

2021-01-26 Por tôpico Ralph Costa Teixeira
Fiz uma versão ligeiramente mais "limpa" do que escrevi antes, vejam se
vocês gostam mais:

1) COM AUTO-SORTEIOS:
p(Mesma Pessoa Inicia e Termina) = p(Apenas um Grande Ciclo de Tamanho N) =
(N-1)! / N!=1/N
Portanto, p(Pessoas diferentes Iniciam e Terminam) = 1-1/N
Por simetria esta segunda probabilidade fica distribuída igualmente entre
os C(N,2) pares de pessoas, ou seja o pedido vale
p = (N-1)/(N C(N,2)) = 2 / N^2
Tomando N=10, deu 2/100=1/50=2%.

2) SEM AUTO-SORTEIOS:
p(Mesma Pessoa Inicia e Termina)=p(Apenas um Grande Ciclo de Tamanho N) =
(N-1)! / K onde K=!N.
Portanto, p (Pessoas diferentes Iniciam e Terminam) = 1 - (N-1)! / K = (K-
(N-1)!)/K.
Por simetria esta segunda probabilidade fica distribuída igualmente entre
os C(N,2) pares de pessoas, ou seja o pedido vale
p = (K- (N-1)!)/(K C(N,2))  = 2 (K-(N-1)!) / (KN(N-1))
Tomando N=10, vem K=1334961, portanto deu p=12001/741645~=1.618%


> On Tue, 26 Jan 2021 at 13:45, Professor Vanderlei Nemitz <
> vanderma...@gmail.com> wrote:
>
>> Oi, pessoal!
>>
>> Com certeza vocês estão acompanhando desde domingo as resoluções da
>> questão do ENEM do amigo secreto.
>> Além da resposta proposta, *1/45*, que parece não estar correta, já vi
>> outras duas, *12001/741645* (ETAPA e ANGLO), que consideram também que o
>> sorteio anterior para definir "quem presenteia quem", e *7/360*, do
>> vídeo a seguir:
>>
>> https://www.youtube.com/watch?v=c-t_BAMASKE
>>
>> Gostaria da opinião (e se possível, uma resolução) dos especialistas da
>> lista (Ralph e cia :))
>>
>> Muito obrigado!
>>
>>
>>
>>
>>
>


Re: [obm-l] Amigo secreto ENEM

2021-01-26 Por tôpico Ralph Costa Teixeira
Oi, Claudio.

Primeiro, parece que o video supõe que NÃO podem haver "auto-sorteios"
(isto fica implícito quando ele diz que a primeira a entregar não pode ser
a primeira e receber nem a penúltima, evitando que o último se de um
presente). Vou supor isso daqui para a frente.

Mas o problema é que o video usa implicitamente que o amigo secreto todo
(SORTEIO + ENTREGA) com N pessoas fica determinado por:
(i) A primeira pessoa que entrega.
E
(ii) A sequência de N pessoas que recebem.

Bom, não funciona para N acima de, huh, 5 eu acho. Deixa eu dar um exemplo
com N=6 para facilitar. Se a gente tivesse:
A + BACDEF
O que significa isso? Vou usar ">" para indicar "deu presente para". Note
as possíveis interpretações disso:
A>B B>A; C>D D>E E>F F>C
ou
A>B B>A; C>D D>C; E>F F>E
Ou seja, esta sequência em particular representa DOIS possíveis sorteios.
Por outro lado:
A + BCDEFA
tem uma unica interpretação possível:
A>B B>C C>D D>E E>F F>A
Por causa disso, as "sequências" que ele criou não são equiprováveis, e
isso derruba o argumento.

(Vou escrever isso no canal dele)

Abraço, Ralph.

On Tue, Jan 26, 2021 at 6:37 PM Claudio Buffara 
wrote:

> Oi, Ralph:
>
> Onde está o erro da solução apresentada no vídeo abaixo?
> https://www.youtube.com/watch?v=c-t_BAMASKE=youtu.be
>
> Eu entendo que se um dado desarranjo tiver 2 ou mais ciclos, então quando
> cada ciclo até o penúltimo for "exaurido", uma nova pessoa deverá ser
> sorteada (dentre aquelas que ainda não deram nem receberam presentes) para
> continuar o jogo.
> Neste caso, um mesmo desarranjo pode dar origem a várias sequências
> distintas de presenteados.
> Por exemplo, com 4 pessoas (numeradas de 1 a 4), se o desarranjo for
> (12)(34) e a pessoa 1 for sorteada para começar, então:
> 1 presenteia 2 que presenteia 1.
> Daí, uma nova pessoa deverá ser sorteada (no caso, 3 ou 4) e a brincadeira
> poderá continuar de 2 maneiras diferentes:
> - 3 presenteia 4 que presenteia 3
> ou
> - 4 presenteia 3 que presenteia 4.
> Mas ambas correspondem ao mesmo desarranjo (12)(34).
>
> A necessidade destes sorteios intermediários para continuar o jogo parece
> complicar bastante a análise com base em desarranjos.
> Daí eu achei interessante o raciocínio apresentado no vídeo, que leva em
> conta apenas a pessoa A sorteada pra dar o primeiro presente e a sequências
> de presenteados, e toma o cuidado de excluir dos casos possíveis as
> sequências de presenteados que têm A na primeira posição (para evitar que A
> se auto-presenteie) e na penúltima posição (para evitar que o último
> presenteado se auto-presenteie).
> Não consegui ver onde está o erro.
>
> []s,
> Claudio.
>
> On Tue, Jan 26, 2021 at 5:26 PM Ralph Costa Teixeira 
> wrote:
>
>> Deixa eu copiar o que escrevi em outro lugar... :D :D
>>
>> Primeiro: não fica claro do enunciado se "auto-sorteios" (alguém sortear
>> o próprio nome) são permitidos ou não, e isto ALTERA a resposta. :(
>>
>> Vejamos possíveis respostas corretas:
>>
>> ---///---
>>
>> SE AUTO-SORTEIOS FOREM PERMITIDOS:
>> Em resumo, temos 1/10 de chance de A iniciar o sorteio, e 1/10 de chance
>> de B terminar (1/10 sim, pois A *pode* terminar). Assim, a resposta seria
>> 1/10*1/10*2=1/50.
>>
>> Com mais detalhes para justificar o segundo "1/10":
>> -- Número de sorteios possíveis = 10!
>> -- Número de sorteios que formam um único ciclo de tamanho 10 = 9!
>> -- Note que ter um ciclo de tamanho 10 equivale a terminar com quem
>> inicia; portanto, a chance de o amigo secreto terminar com o mesmo que
>> iniciou seria 9!/10!=1/10 (*que é independente de quem começa*).
>>
>> Assim:
>> -- Chance de A iniciar = 1/10;
>> Agora, DADO QUE A INICIOU:
>>  Chance de A terminar = 9!/10! = 1/10
>>  Portanto, chance de não terminar com A: 9/10
>>  Chance de B terminar (por simetria): (9/10) /9 = 1/10
>>
>> Isso nos dá 1/10*1/10 = 1/100 de chance do amigo secreto começar por A e
>> terminar com B. Portanto a resposta seria o dobro, 1/50.
>>
>> ---///---
>> SE AUTO-SORTEIOS SÃO PROIBIDOS:
>> -- Número de sorteios (desarranjos) possíveis = !10 (vou escrever K=!10
>> daqui por diante);
>> -- Número de sorteios que formam um único ciclo de tamanho 10 = 9!
>> -- Portanto, a chance de o amigo secreto terminar com o mesmo que iniciou
>> seria 9!/K (que é independente de quem começa).
>>
>> Assim:
>> -- Chance de A iniciar = 1/10;
>> Agora, DADO QUE A INICIOU:
>>  Chance de A terminar = 9!/K
>>  Portanto, chance de não terminar com A: 1-9!/K
>>  Chance de B termi

Re: [obm-l] Amigo secreto ENEM

2021-01-26 Por tôpico Ralph Costa Teixeira
Deixa eu copiar o que escrevi em outro lugar... :D :D

Primeiro: não fica claro do enunciado se "auto-sorteios" (alguém sortear o
próprio nome) são permitidos ou não, e isto ALTERA a resposta. :(

Vejamos possíveis respostas corretas:

---///---

SE AUTO-SORTEIOS FOREM PERMITIDOS:
Em resumo, temos 1/10 de chance de A iniciar o sorteio, e 1/10 de chance de
B terminar (1/10 sim, pois A *pode* terminar). Assim, a resposta seria
1/10*1/10*2=1/50.

Com mais detalhes para justificar o segundo "1/10":
-- Número de sorteios possíveis = 10!
-- Número de sorteios que formam um único ciclo de tamanho 10 = 9!
-- Note que ter um ciclo de tamanho 10 equivale a terminar com quem inicia;
portanto, a chance de o amigo secreto terminar com o mesmo que iniciou
seria 9!/10!=1/10 (*que é independente de quem começa*).

Assim:
-- Chance de A iniciar = 1/10;
Agora, DADO QUE A INICIOU:
 Chance de A terminar = 9!/10! = 1/10
 Portanto, chance de não terminar com A: 9/10
 Chance de B terminar (por simetria): (9/10) /9 = 1/10

Isso nos dá 1/10*1/10 = 1/100 de chance do amigo secreto começar por A e
terminar com B. Portanto a resposta seria o dobro, 1/50.

---///---
SE AUTO-SORTEIOS SÃO PROIBIDOS:
-- Número de sorteios (desarranjos) possíveis = !10 (vou escrever K=!10
daqui por diante);
-- Número de sorteios que formam um único ciclo de tamanho 10 = 9!
-- Portanto, a chance de o amigo secreto terminar com o mesmo que iniciou
seria 9!/K (que é independente de quem começa).

Assim:
-- Chance de A iniciar = 1/10;
Agora, DADO QUE A INICIOU:
 Chance de A terminar = 9!/K
 Portanto, chance de não terminar com A: 1-9!/K
 Chance de B terminar (por simetria): (1-9!/K) /9 = (K-9!)/(9K)

Isso nos dá 1/10* (K-9!)/(9K) = (K-9!)/(90K) de chance do amigo secreto
começar por A e terminar com B. Portanto a resposta seria o dobro,
(K-9!)/(45K). Fazendo a conta com a ajuda do computador, achei 12001/741645.

Abraço, Ralph.



On Tue, Jan 26, 2021 at 1:45 PM Professor Vanderlei Nemitz <
vanderma...@gmail.com> wrote:

> Oi, pessoal!
>
> Com certeza vocês estão acompanhando desde domingo as resoluções da
> questão do ENEM do amigo secreto.
> Além da resposta proposta, *1/45*, que parece não estar correta, já vi
> outras duas, *12001/741645* (ETAPA e ANGLO), que consideram também que o
> sorteio anterior para definir "quem presenteia quem", e *7/360*, do vídeo
> a seguir:
>
> https://www.youtube.com/watch?v=c-t_BAMASKE
>
> Gostaria da opinião (e se possível, uma resolução) dos especialistas da
> lista (Ralph e cia :))
>
> Muito obrigado!
>
>
>
>
>


Re: [obm-l] Problema da IMO

2020-09-11 Por tôpico Ralph Costa Teixeira
Essa eh da IMO 1992. Tem uma solucao aqui:
http://sms.math.nus.edu.sg/Simo/IMO_Problems/92.pdf

On Fri, Sep 11, 2020 at 10:06 PM Pedro José  wrote:

> Bom dia!
>
> Recebi de um filho de um amigo, um problema que já o fizera.
> (a-1)(b-1)(c-1) | abc-1;  1
> Confesso que desta feita gastei mais tempo que da  primeira vez. Curioso,
> da primeira ,eu pensei, dessa vez, eu tentei lembrar como eu resolvera, aí
> nem lembrava, nem pensava. Apelei para a internet, mas não encontrei nada.
> Mas no fim, recordei o que havia feito.
> (1+1/(a-1))(1+1/(b-1))(1+1/(c-1)) = k, onde k é inteiro.
> vê-se que k>1, e para um dado a k é máximo para b e c mínimos logo b=a+1 e
> c=a+2
> [a(a+1)(a+2)]/[(a-1)(a)(a+1)] > [a(a+1)(a+2)-1]/[(a-1)(a)(a+1)]>=2, então
> (a+2)/(a-1)>2 ==> a <4, a=2 ou a=3.
> O k é máximo para a=2, b=3 e c=4 ==> k <4, logo k=2 ou k=3.
> S.p.g, se a é ímpar (a-1)(b-1)(c-1) é par; então b,c ímpares e k é livre.
> S.p.g se a é par abc-1 é ímpar; então b,c são pares e k ímpar.
> a=2, temos  2b(b+1)/[(b-1)b] >3, não usei a restrição de paridade para c
> para facilitar a simplificação. b<5 Logo a=2 é ímpar k=3. Logo c= 8. (2,4,8) é uma solução.
>
> a=3 temos 3b(b+1)/[2(b-1)b] > 2; b<7 e 3 para a=3 e b=5. kmax <= (15*7-1)/(2*4*6) <=2;pois k é inteiro.
> 1 k=2 e c= 15. (3,5,15) é a outra solução.
>
> Só agora me apercebi de que c=ab nas duas soluções. Então tentei uma nova
> solução.
> (a-1)(b-1)(c-1) | abc-1 e (a-1)(b-1)(c-1) | abc + c(1 - (a+b)) -ab+ (a+b)
> - 1 logo divide a diferença:
> (a-1)(b-1)(c-1) | (a+b) (c-1) + ab -1 - (c-1) logo c-1 | ab-1, então ab-1=
> w(c-1), para algum w inteiro e ab=w(c-1) +1 (i)
> Como a=2 ou a=3
> Se a=2. e w>=2
> Temos por (i) 2b>= 2 (c-1) +1 c-1>=b, logo absurdo.
> Se a=3
> Temos por (i) 3b>= w(c-1)+1; w=3 ==>3b< 3 (c-1) +1 pois c>b
> w=2 ==> 3b =2(c-1) +1 ==> c=(3b+1)/2
> 2(b-1)(3b-1)/2 | 3b(3b+1)/2 -1 ==>  2(b-1)(3b-1)/ | 3b(3b+1) -2 ==>
> 6b^2-8b-2 | 9b^2+3b-1 ==>  6b^2-8b-2 | 3b^2 +11b+ 4
> ==> b <=5. Como b>a=3 ==> b=5 e c= 8, ferindo a paridade.
> Logo ab-1=c-1 ==> ab=c ==> (a-1)(b-1)(c-1) | c^2-1 ==>  (a-1)(b-1) |  c+1
> (a-1)(b-1) |ab+1==> (a-1)(b-1)!a+b
> a=2 ==> 2b-2= 2+b. b=4 e c=ab=8 (2,4,8)
> a=3 ==> 2(b-1) | 3+b ==> 2(b-1) = 3+. b=5 e c=ab=15. (3,5,15),
> Forcei um pouco a barra para mostrar que c=ab.
> Alguém teria uma outra solução, ou um endereço onde se tem as questões da
> IMO e suas resoluções?
>
> Grato!
> Saudações,
> PJMS
>
>
>
> --
> Esta mensagem foi verificada pelo sistema de antivírus e
> acredita-se estar livre de perigo.

-- 
Esta mensagem foi verificada pelo sistema de antiv�rus e
 acredita-se estar livre de perigo.



[obm-l] Re: [obm-l] Elipse e lugar geométrico

2020-08-19 Por tôpico Ralph Costa Teixeira
As coordenadas do incentro sao a media ponderada das coordenadas dos
vertices, usando os lados como pesos. Ou seja, se escrevo P=(5cost,4sint),
F1=(-3,0), F2=(3,0) e Incentro=(x,y):

x = (30cost + (-3)b + 3c) / 16
y = (24sint + 0 + 0) / 16

onde b=d(P,F2) e c=d(P,F1). Note que b+c=eixo maior = 10.

Mais especificamente:

b^2=d(P,F2)^2=(5cost-3)^2+(4sint)^2=9(cost)^2-30cost+25=(3cost-5)^2, ou
seja, b=5-3cost, portanto c=5+3cost.

Jogando na formula de x e y:

x= 3cost ; y=3sint/2. Outra elipse, a saber, (x^2)/9+(y^2)/(9/4)=1 (talvez
tirando os pontos onde tudo degenera, para ser chato).

Abraço, Ralph.



Hmm Assim:

On Wed, Aug 19, 2020 at 11:58 PM Professor Vanderlei Nemitz <
vanderma...@gmail.com> wrote:

> Oi!
> Venho com mais uma envolvendo incentro.
>
> *O ponto P pertence a uma elipse de focos F1 e F2 e de equação (x^2)/25 +
> (y^2)/16 = 1. Determine o lugar geométrico do incentro do triângulo PF1F2.*
>
> Muito obrigado!
>
>
> 
>  Livre
> de vírus. www.avast.com
> .
> <#m_2344932968934913062_DAB4FAD8-2DD7-40BB-A1B8-4E2AA1F9FDF2>
>
> --
> Esta mensagem foi verificada pelo sistema de antivírus e
> acredita-se estar livre de perigo.

-- 
Esta mensagem foi verificada pelo sistema de antiv�rus e
 acredita-se estar livre de perigo.



[obm-l] Re: [obm-l] Re: [obm-l] Mostrar que está função não existe

2020-08-10 Por tôpico Ralph Costa Teixeira
Acho que isso caiu numa IMO que eu fiz Ah, achei, 1987. Aqui tem uma
resposta bem legal:

https://math.stackexchange.com/questions/325504/imo-1987-function-such-that-ffn-n1987


On Tue, Aug 11, 2020 at 12:50 AM  wrote:

> É, fatou dizer que k é ímpar
>
> Artur
>
> Em 10 de ago de 2020 22:33, Ralph Costa Teixeira 
> escreveu:
>
> K inteiro... ímpar? Porque tomando f(n)=n+k/2...
>
> On Mon, Aug 10, 2020, 22:05 Artur Costa Steiner <
> artur.costa.stei...@gmail.com> wrote:
>
> Me pareceu que isto era simples, mas segui um caminho errado e ainda não
> cheguei lá.
>
> Mostre que não existe f:N --> N, N os naturais com o 0, tal que f(f(n)) =
> n + k, k > 0 inteiro.
>
> Obrigado
>
> Artur
>
> --
> Esta mensagem foi verificada pelo sistema de antivírus e
> acredita-se estar livre de perigo.
>
>
> --
> Esta mensagem foi verificada pelo sistema de antiv�rus e
> acredita-se estar livre de perigo.
>
>
>
> --
> Esta mensagem foi verificada pelo sistema de antivírus e
> acredita-se estar livre de perigo.

-- 
Esta mensagem foi verificada pelo sistema de antiv�rus e
 acredita-se estar livre de perigo.



[obm-l] Re: [obm-l] Mostrar que está função não existe

2020-08-10 Por tôpico Ralph Costa Teixeira
K inteiro... ímpar? Porque tomando f(n)=n+k/2...

On Mon, Aug 10, 2020, 22:05 Artur Costa Steiner <
artur.costa.stei...@gmail.com> wrote:

> Me pareceu que isto era simples, mas segui um caminho errado e ainda não
> cheguei lá.
>
> Mostre que não existe f:N --> N, N os naturais com o 0, tal que f(f(n)) =
> n + k, k > 0 inteiro.
>
> Obrigado
>
> Artur
>
> --
> Esta mensagem foi verificada pelo sistema de antivírus e
> acredita-se estar livre de perigo.

-- 
Esta mensagem foi verificada pelo sistema de antiv�rus e
 acredita-se estar livre de perigo.



Re: [obm-l] Desafio de probabilidade

2020-07-25 Por tôpico Ralph Costa Teixeira
Argh: tem um errinho de digitação... Era p=7b *MENOS* 3...

Mas o resto continua valendo, achei p=25/43 e b=22/43... que condiz com
minha intuição de que, partindo de um número par (que não se repetiu),
tenho uma pequena vantagem (b=22/43 é ligeiramente maior que 1/2).

On Sat, Jul 25, 2020 at 3:37 PM Ralph Costa Teixeira 
wrote:

> Oi, Vanderlei.
>
> Para facilitar a notação, eu serei Zé Roberto. :D
>
> Intuitivamente: como você desconfiou, p não pode ser isso tudo. Para eu
> ganhar, tenho que rolar um 6, **ou** rolar outra coisa e "praticamente"
> começar o jogo de novo. Isto daria a estimativa:
> p = 1/6 + 5/6 . 1/2 = 7/12
> Mas esta estimativa está errada pois, o jogo "recomeçaria" a partir de 1,
> 2, 3, 4 ou 5, dando uma pequena vantagem para Umberto! Ou seja, sem fazer
> muita conta afirmo que:
> p = 1/6 + 5/6 . Prob(Eu ganhar a partir de um 1,2,3,4,5 simples) < 1/6 +
> 5/12 = 7/12.
>
> ---///---
> Se entendi o que você pensou: para Umberto ganhar, temos que NÃO ROLAR 6
> agora (prob=5/6), **e** depois "começar o jogo do zero" (prob=3/6). Então
> Umberto ganharia com probabilidade q=5/6.3/6=5/12 (acho que você devia usar
> 5/6; e não entendi o p extra). Era isso? Mas mesmo assim não funciona, pelo
> mesmo motivo da minha estimativa do p estar furada: não rolando 6, o jogo
> não "começaria do zero"; Umberto teria uma pequena vantagem (pois rolamos
> 1, 3 ou 5 com mais chance do que 2 ou 4). Em suma, o que conseguimos
> concluir daqui eh que q > 5/12, ou seja p<7/12, que nem acima!
>
> ---///---
> Vamos calcular p, definido como"
> p = probabilidade de eu vencer sendo que os dois últimos números foram
> iguais e pares
> E vamos inventar:
> b = probabilidade de eu vencer sendo que o último número foi par, mas o
> penúltimo foi diferente do último.
> Agora, na situação do problema (terminando em x-6-6, com x<>6), a partir
> daqui temos duas possibilidades:
> -- Se o próximo número for 6 (prob = 1/6), ganhei.
> -- Se o próximo número for 2 ou 4 (prob = 2/6), passamos para uma nova
> situação onde eu tenho probabilidade b de vencer.
> -- Se o próximo número for 1, 3 ou 5 (prob=3/6), passamos para uma nova
> situação onde eu tenho probabilidade 1-b de vencer.
>
> Assim, p = 1/6 + (2/6)b + (3/6)(1-b) = 2/3 - b/6.
>
> Por outro lado, a partir de uma situação do tipo "b" -- (digamos, para
> fixar ideias, sequência terminando em -2-4), temos as seguintes
> possibilidades:
> -- Se o próximo número for 4 (prob = 1/6), passo a ter probabilidade p de
> ganhar;
> -- Se for 2 ou 6 (prob = 2/6), passo a ter probabilidade b de ganhar.
> -- Se for 1, 3 ou 5 (prob = 3/6), passo a ter probabilidade 1-b de ganhar.
>
> Assim, b = (1/6)p + 2/6b + 3/6(1-b), ou seja, p=7b+3.
>
> Juntando as duas coisas, achei p=25/43 Hein, sério??
>
> ---///---
>  Vamos resolver de outro jeito mais "adulto", para ver o PODER DAS
> MATRIZES... :D :D:
>
> Depois de alguns lançamentos, o jogo tem 6 estados possíveis dependendo
> apenas dos 3 últimos lançamentos:
>
> E1: ...y-y-y com y par (eu ganhei! pare o jogo!)
> E2: ...x-y-y com x<>y e y par (estou quase ganhando!)
> E3: ...x-y com x<>y e y par (tenho pequena vantagem)
> E4: ...x-y com x<>y e y ímpar (tenho pequena desvantagem)
> E5: ...x-y-y com x<>y e y í mpar (estou quase perdendo!)
> E6: ...y-y-y com y ímpar (perdi! vire a mesa!)
>
> A matriz M de transição entre esses estados:
>
> 1 1/6000 0
> 00 1/600 0
> 0 2/6 2/6 3/6 3/6 0
> 0 3/6 3/6 2/6 2/6 0
> 000 1/60 0
> 0000 1/6 1
>
> Pois bem, M^k.v (onde v=e_2=[0; 1; 0; 0; 0; 0]) seria a distribuição de
> probabilidade dos estados, começando de e_2 (situação do problema), daqui a
> k jogadas. Ou seja, a gente quer determinar p onde lim(k->Inf) M^k.v = [p;
> 0; 0; 0; 0; 1-p].
>
> Diagonalizei M usando o computador (que não liga para a elegância das
> simetrias :( ), deu M = PDP^(-1) onde
>
> P=[1 1 1 1 1 0
> 0 (1/(√5-3))(√5+3) (3/2)√5-(7/2) -(1/2)i√3-((13)/2) (1/2)i√3-((13)/2) 0
> 0 -2((√5)/(√5-3)) (5/2)-(3/2)√5 (7/2)i√3+(5/2) (5/2)-(7/2)i√3 0
> 0 -2((√5)/(√5-3)) (5/2)-(3/2)√5 -(7/2)i√3-(5/2) (7/2)i√3-(5/2) 0
> 0 (1/(√5-3))(√5+3) (3/2)√5-(7/2) (1/2)i√3+((13)/2) ((13)/2)-(1/2)i√3 0
> -1 1 1 -1 -1 1]
>
> D = diag(1; 5/12- √5 /4;  5/12 +√5 /4; (-i√3-1)/12; (i√3-1)/12; 1]
>
> Q=P^(-1)= [1 ((25)/(43)) ((22)/(43)) ((21)/(43)) ((18)/(43)) 0
> 0 (1/(12))√5-(1/4) (7/(60))√5-(1/4) (7/(60))√5-(1/4) (1/(12))√5-(1/4) 0
> 0 -(1/(12))√5-(1/4) -(7/(60))√5-(1/4) -(7/(60))√5-(1/4) -(1/(12))√5-(1/4) 0
> 0 -(5/(516))i√3-(7/(172)) -((13)/(516))i√3-(1/(172))
> ((13)/(516))i√3+(1/(172)) (5/(516))i√3+(7/(172)) 0
&g

Re: [obm-l] Desafio de probabilidade

2020-07-25 Por tôpico Ralph Costa Teixeira
Oi, Claudio

Eu também pensei em trocar o dado por uma moeda, mas se entendi bem o
enunciado, não podemos! O problema eh que, se o dado der 2,4,6,2,4,6,1,1,1,
quem ganha eh Umberto; trocando pela moeda, vemos par,par,par e vamos dar o
trofeu para o Ze Roberto... Muda o jogo!

On Sat, Jul 25, 2020 at 3:24 PM Claudio Buffara 
wrote:

> Pra facilitar, podemos substituir o dado por uma moeda, com cara = par = 0
> e coroa = ímpar = 1, já que o que importa é apenas a paridade do número na
> face superior do dado lançado e, neste caso, P(par) = P(ímpar) = 1/2.
>
> Como 3 caras seguidas ou 3 coroas seguidas encerra o jogo, basta
> considerar os dois últimos lançamentos.
>
> Suponha que dois lançamentos seguidos tenham sido 1 e 0 (cara e coroa).
> Após sair o 0, digamos que a probabilidade de ZR vencer seja p.
>
> Se o terceiro lançamento for 0, a probabilidade de ZR vencer aumentará
> para q  (p e q são incógnitas a serem determinadas), e q é justamente a
> probabilidade desejada, já que é a probabilidade de ZR vencer dado que os
> dois últimos lançamentos foram 0 e 0.
> Se o quarto lançamento for 0, ZR vence. Mas se for 1, sua probabilidade de
> vencer cai para 1-p pois, neste caso, por simetria, Umberto passa a ter
> probabilidade p de vencer, em virtude dos dois últimos lançamentos terem
> sido 0 e 1.
> Ou seja, q = (1/2)*(1 + (1-p))  <==>  p + 2q = 2.
>
> Se o terceiro lançamento for 1, a probabilidade de ZR vencer cai para 1-p.
> Neste caso, podemos escrever p = (1/2)*(q + (1-p))  <==>  3p - q = 1.
>
> Resolvendo este sistema, achamos p = 4/7 e q = 5/7.
>
> Na verdade, isso tudo fica mais fácil de ver se você fizer uma árvore.
>
> []s,
> Claudio.
>
> On Sat, Jul 25, 2020 at 2:03 PM Professor Vanderlei Nemitz <
> vanderma...@gmail.com> wrote:
>
>> Então meu raciocínio foi muito errado, pois pensei assim:
>> Seja p a probabilidade de Zé Roberto vender. Podemos considerar que o
>> jogo "começa" com Zé Roberto precisando obter um 6 para vencer.
>> Assim, a probabilidade de Humberto vencer é:
>> q = (3/6).(1/6).p, ou seja, p = 12q
>> Assim, p = 12/13 e q = 1/13
>>
>> Prezado Cláudio, você pode explicar sua resolução?
>>
>> Muito obrigado!
>>
>>
>>
>>
>>
>>
>> Em sáb., 25 de jul. de 2020 às 13:43, Claudio Buffara <
>> claudio.buff...@gmail.com> escreveu:
>>
>>> Eu achei 5/7.
>>>
>>> On Sat, Jul 25, 2020 at 7:28 AM Professor Vanderlei Nemitz <
>>> vanderma...@gmail.com> wrote:
>>>
 Bom dia!
 O problema a seguir encontra-se em uma prova de desafios da PUC-RJ,
 muito boas!!!
 Acho que são organizadas pelo professor Nicolau Saldanha.
 Encontrei uma resposta bem alta, mais de 90%. Será que está correto?
 Muito obrigado!

 Zé Roberto e Umberto gostam de jogar par ou ímpar; Zé Roberto sempre
 pede par e Umberto sempre pede íımpar. Eles gostam de inventar novas
 maneiras de jogar. A última maneira que eles inventaram usa um dado comum,
 com seis faces numeradas de 1 a 6. Eles jogam o dado várias vezes até que
 um número saia três vezes seguidas; Zé Roberto ganha se este número for
 par, Umberto ganha se for ímpar. Sábado de manhã o dado teve os resultados:
 5, 3, 4, 2, 6, 1, 1, 3, 1, 4, 2, 3, 5, 6, 3, 4, 5, 4, 4, 4 e neste ponto Zé
 Roberto se declarou vitorioso. Sábado de tarde o dado teve os resultados:
 6, 1, 4, 2, 3, 5, 6, 6; neste momento o jogo foi interrompido pela queda de
 um meteorito. Quando a situação se acalmou, eles concordaram em continuar
 do ponto em que estavam. Qual é a probabilidade de que Zé Roberto seja o
 vencedor?

 --
 Esta mensagem foi verificada pelo sistema de antivírus e
 acredita-se estar livre de perigo.
>>>
>>>
>>> --
>>> Esta mensagem foi verificada pelo sistema de antivírus e
>>> acredita-se estar livre de perigo.
>>
>>
>> --
>> Esta mensagem foi verificada pelo sistema de antivírus e
>> acredita-se estar livre de perigo.
>
>
> --
> Esta mensagem foi verificada pelo sistema de antivírus e
> acredita-se estar livre de perigo.

-- 
Esta mensagem foi verificada pelo sistema de antiv�rus e
 acredita-se estar livre de perigo.



Re: [obm-l] Desafio de probabilidade

2020-07-25 Por tôpico Ralph Costa Teixeira
Oi, Vanderlei.

Para facilitar a notação, eu serei Zé Roberto. :D

Intuitivamente: como você desconfiou, p não pode ser isso tudo. Para eu
ganhar, tenho que rolar um 6, **ou** rolar outra coisa e "praticamente"
começar o jogo de novo. Isto daria a estimativa:
p = 1/6 + 5/6 . 1/2 = 7/12
Mas esta estimativa está errada pois, o jogo "recomeçaria" a partir de 1,
2, 3, 4 ou 5, dando uma pequena vantagem para Umberto! Ou seja, sem fazer
muita conta afirmo que:
p = 1/6 + 5/6 . Prob(Eu ganhar a partir de um 1,2,3,4,5 simples) < 1/6 +
5/12 = 7/12.

---///---
Se entendi o que você pensou: para Umberto ganhar, temos que NÃO ROLAR 6
agora (prob=5/6), **e** depois "começar o jogo do zero" (prob=3/6). Então
Umberto ganharia com probabilidade q=5/6.3/6=5/12 (acho que você devia usar
5/6; e não entendi o p extra). Era isso? Mas mesmo assim não funciona, pelo
mesmo motivo da minha estimativa do p estar furada: não rolando 6, o jogo
não "começaria do zero"; Umberto teria uma pequena vantagem (pois rolamos
1, 3 ou 5 com mais chance do que 2 ou 4). Em suma, o que conseguimos
concluir daqui eh que q > 5/12, ou seja p<7/12, que nem acima!

---///---
Vamos calcular p, definido como"
p = probabilidade de eu vencer sendo que os dois últimos números foram
iguais e pares
E vamos inventar:
b = probabilidade de eu vencer sendo que o último número foi par, mas o
penúltimo foi diferente do último.
Agora, na situação do problema (terminando em x-6-6, com x<>6), a partir
daqui temos duas possibilidades:
-- Se o próximo número for 6 (prob = 1/6), ganhei.
-- Se o próximo número for 2 ou 4 (prob = 2/6), passamos para uma nova
situação onde eu tenho probabilidade b de vencer.
-- Se o próximo número for 1, 3 ou 5 (prob=3/6), passamos para uma nova
situação onde eu tenho probabilidade 1-b de vencer.

Assim, p = 1/6 + (2/6)b + (3/6)(1-b) = 2/3 - b/6.

Por outro lado, a partir de uma situação do tipo "b" -- (digamos, para
fixar ideias, sequência terminando em -2-4), temos as seguintes
possibilidades:
-- Se o próximo número for 4 (prob = 1/6), passo a ter probabilidade p de
ganhar;
-- Se for 2 ou 6 (prob = 2/6), passo a ter probabilidade b de ganhar.
-- Se for 1, 3 ou 5 (prob = 3/6), passo a ter probabilidade 1-b de ganhar.

Assim, b = (1/6)p + 2/6b + 3/6(1-b), ou seja, p=7b+3.

Juntando as duas coisas, achei p=25/43 Hein, sério??

---///---
 Vamos resolver de outro jeito mais "adulto", para ver o PODER DAS
MATRIZES... :D :D:

Depois de alguns lançamentos, o jogo tem 6 estados possíveis dependendo
apenas dos 3 últimos lançamentos:

E1: ...y-y-y com y par (eu ganhei! pare o jogo!)
E2: ...x-y-y com x<>y e y par (estou quase ganhando!)
E3: ...x-y com x<>y e y par (tenho pequena vantagem)
E4: ...x-y com x<>y e y ímpar (tenho pequena desvantagem)
E5: ...x-y-y com x<>y e y í mpar (estou quase perdendo!)
E6: ...y-y-y com y ímpar (perdi! vire a mesa!)

A matriz M de transição entre esses estados:

1 1/6000 0
00 1/600 0
0 2/6 2/6 3/6 3/6 0
0 3/6 3/6 2/6 2/6 0
000 1/60 0
0000 1/6 1

Pois bem, M^k.v (onde v=e_2=[0; 1; 0; 0; 0; 0]) seria a distribuição de
probabilidade dos estados, começando de e_2 (situação do problema), daqui a
k jogadas. Ou seja, a gente quer determinar p onde lim(k->Inf) M^k.v = [p;
0; 0; 0; 0; 1-p].

Diagonalizei M usando o computador (que não liga para a elegância das
simetrias :( ), deu M = PDP^(-1) onde

P=[1 1 1 1 1 0
0 (1/(√5-3))(√5+3) (3/2)√5-(7/2) -(1/2)i√3-((13)/2) (1/2)i√3-((13)/2) 0
0 -2((√5)/(√5-3)) (5/2)-(3/2)√5 (7/2)i√3+(5/2) (5/2)-(7/2)i√3 0
0 -2((√5)/(√5-3)) (5/2)-(3/2)√5 -(7/2)i√3-(5/2) (7/2)i√3-(5/2) 0
0 (1/(√5-3))(√5+3) (3/2)√5-(7/2) (1/2)i√3+((13)/2) ((13)/2)-(1/2)i√3 0
-1 1 1 -1 -1 1]

D = diag(1; 5/12- √5 /4;  5/12 +√5 /4; (-i√3-1)/12; (i√3-1)/12; 1]

Q=P^(-1)= [1 ((25)/(43)) ((22)/(43)) ((21)/(43)) ((18)/(43)) 0
0 (1/(12))√5-(1/4) (7/(60))√5-(1/4) (7/(60))√5-(1/4) (1/(12))√5-(1/4) 0
0 -(1/(12))√5-(1/4) -(7/(60))√5-(1/4) -(7/(60))√5-(1/4) -(1/(12))√5-(1/4) 0
0 -(5/(516))i√3-(7/(172)) -((13)/(516))i√3-(1/(172))
((13)/(516))i√3+(1/(172)) (5/(516))i√3+(7/(172)) 0
0 (5/(516))i√3-(7/(172)) ((13)/(516))i√3-(1/(172))
(1/(172))-((13)/(516))i√3 (7/(172))-(5/(516))i√3 0
1 1 1 1 1 1]

Mas não olhe para tudo isso! Veja bem, M^k.v = P D^k P^(-1) e_2 = P D^k
Q_2, onde Q_2 eh a segunda coluna de Q. Mas D^k vai para diag(1, 0, 0, 0,
0, 1) quando k->Inf, portanto ligamos apenas para Q(1,2)=25/43 e Q(1,6)=1.
Queremos apenas a primeira coordenada de P . [25/43; 0; 0; 0; 0; 1], ou
seja, p = 25/43 P(1,1) + 1 P(6,1) = 25/43.

Ou seja: sério!! :D

Abraco, Ralph.


On Sat, Jul 25, 2020 at 2:03 PM Professor Vanderlei Nemitz <
vanderma...@gmail.com> wrote:

> Então meu raciocínio foi muito errado, pois pensei assim:
> Seja p a probabilidade de Zé Roberto vender. Podemos considerar que o jogo
> "começa" com Zé Roberto precisando obter um 6 para vencer.
> Assim, a probabilidade de Humberto vencer é:
> q = (3/6).(1/6).p, ou seja, p = 12q
> Assim, p = 12/13 e q = 1/13
>
> Prezado 

Re: [obm-l] Probabilidade

2020-07-21 Por tôpico Ralph Costa Teixeira
Vou chamar coroa de C e cara de K. Vamos criar duas funcoes:

f(n)=numero de sequências de n lançamentos sem CC, terminando com K.
g(n)=numero de sequências de n lançamentos sem CC, terminando com C.

Por exemplo:
f(1)=1 (K); g(1)=1 (C); f(2)=2 (CK, KK); g(2)=1 (KC)...

Pois bem, note que f(n+1)=f(n)+g(n) -- para a sequência de n+1 elementos
terminar com K, basta que não haja CC nos n primeiros;
Por outro lado, g(n+1)=f(n) -- para a sequência de n+1 elementos terminar
com C, a sequência dos n primeiros (nao pode ter CC e tem que terminar com
K).

Juntando as coisas, temos f(n+1)=f(n)+f(n-1) -- Fibonacci! O que queremos
deve ser (f(10)+g(10)) / 2^10. Mas g(10)=f(9), então queremos f(11)/1024.

Bom, melhor fazer logo no braco:
{f(n)} = 1,2,3,5,8,13,21,34,55,89,144...
(Fibonacci, com um ligeiro "shift" pois nao começa com 1,1,...)

Portanto, acho que a resposta deve ser 144/1024. Acertei?

Abraço, Ralph.



On Tue, Jul 21, 2020 at 10:33 PM marcone augusto araújo borges <
marconeborge...@hotmail.com> wrote:

> Uma moeda honesta é lançada 10 vezes. Qual a probabilidade de não sair
> duas caras consecutivas?
> Eu achei que fosse (3/4)^9, mas fui informado que a resposta não é essa.
>
> --
> Esta mensagem foi verificada pelo sistema de antivírus e
> acredita-se estar livre de perigo.
>

-- 
Esta mensagem foi verificada pelo sistema de antiv�rus e
 acredita-se estar livre de perigo.



[obm-l] Re: [obm-l] Diferencial de uma função de uma variável

2020-06-21 Por tôpico Ralph Costa Teixeira
Voce diz, aquele "dy" sozinho?

Eu gosto de pensar assim: considere uma função f(x) diferenciável num ponto
a. A *linearizacão* de f(x) em x=a é dada por:
L(x) = f(a) + f'(a) (x-a)
e a ideia é que L(x) aproxima "bastante bem" f(x) ali perto de x=a (o
gráfico de L(x) é a reta tangente).

Para dar contexto, escreva y=f(x) e, a partir de "a", vamos aplicar uma
variação Delta_x (um número real, possivelmente grande), indo para
x=a+Delta_x. Esta variação no domínio provoca variação na imagem de f, a
saber:
Delta_y = Delta_f = f(a+Delta_x)-f(a)
Analogamente, olhe para L(x) e, a partir de "a", aplique uma variação de dx
(um número real, possivelmente grande), indo para x=a+dx. A *diferencial de
f no ponto a (associada a dx) *é
dy = Delta_L=L(a+dx)-L(a)
ou seja, dy é simplesmente *a variação em y MEDIDA PELA LINEARIZAÇÃO*, ou
seja, *USANDO A RETA TANGENTE* (ao invés de usar a f(x) original).

Note que podemos escrever dy explicitamente em termos de f, pois temos
aquela fórmula ali em cima para L:
dy =  L(a+dx)-L(a) = (f(a)+f'(a).dx)-(f(a)+f'(a).0) = f'(a).dx
Em suma:
dy = f'(a).dx
Esta última expressão é exatamente a equação da reta tangente, escrita dum
jeito mais curto (pois fizemos L(x)-L(a)=dy e x-a=dx)!

Comparando:
-- Não há diferença prática entre "dx" e "Delta_x"; apenas por convenção,
quando eu estiver trabalhando com a linearização, vou escrever dx ao invés
de Delta_x. Voce não perde praticamente nada se pensar que dx=Delta_x.
-- Por outro lado, "dy" e "Delta_y" podem ser bem diferentes (em nenhum
momento eu disse que dx ou dy são pequenos!). Isto dito, o grande barato da
derivada é que, voce pode usar a aproximação Delta_y ~= dy para Delta_x =
dx suficientemente pequeno! Por isso que muita gente acaba pensando em dy
como um "Delta_y infinitesimal" (uma intuição útil, mas apenas intuição --
repito que dy tem o direito de ser imenso e muito diferente de Delta_y).

Abraço, Ralph.


On Sun, Jun 21, 2020 at 11:22 AM Pacini Bores 
wrote:

> Olá Pessoal,
>
>
>
> Qual é a melhor forma de se definir a diferencial de uma função de uma
> única variável ?
>
> Abraços
>
> Pacini
>
>
> --
> Esta mensagem foi verificada pelo sistema de antivírus e
> acredita-se estar livre de perigo.
>

-- 
Esta mensagem foi verificada pelo sistema de antiv�rus e
 acredita-se estar livre de perigo.



[obm-l] Re: [obm-l] polígono regular - 13 lados

2020-06-18 Por tôpico Ralph Costa Teixeira
Hm... Fiz um raciocínio aqui, confiram se errei algo. Vou chamar os
vértices de P1, P2, ..., P13.

Primeiro: o enunciado tinha que deixar mais claro como contar triângulos...
Por exemplo, triângulos congruentes em si contam apenas uma vez? P1P2P6
conta igual a P2P3P7? Normalmente, eu diria que eles são **distintos**, mas
neste caso a resposta seria muito mais que 36 (e seria um múltiplo de 13,
pois, para cada triângulo válido, teríamos suas 13 rotações também, que
seriam distintas).

---///---

Mas vamos supor que o enunciado considera triângulos congruentes como um
único triângulo. Neste caso, o triângulo ABC (suponha A-B-C no sentido
anti-horário) fica completamente determinado pelos comprimentos dos 3 arcos
AB, AC e BC no círculo circunscrito (e vice-versa: dados os 3 arcos, em
qualquer ordem, eles determinam os comprimentos dos lados, e portanto
determinariam um único triângulo). Escrevendo os arcos como AB=x.2pi/13,
BC=y.2pi/13 e CA=z.2pi/13, um triângulo ABC corresponde exatamente a uma
tripla ***desordenada*** de inteiros positivos {x,y,z} satisfazendo
x+y+z=13.

Para que o circuncentro esteja no interior do triângulo, basta que ele seja
acutângulo, ou seja, x,y,z<=6. Então agora temos um problema combinatório:

"Determinar o número de soluções inteiras de x+y+z=13 satisfazendo
1<=x,y,z<=6, onde a ordem das variáveis não importa."
Fazendo a=6-x, b=6-y e c=6-z, o problema vira
"Determinar o número de soluções inteiras distintas de a+b+c=5 satisfazendo
0<=a,b,c<=5 (sem ordem)"
Opa, assim o "<=5" fica desnecessário, pois a+b+c=5 e a,b,c>=0 implicam
a,b,c<=5! Então agora é (quase) um problema clássico daqueles com bolinhas
e barrinhas para separar bolinhas... "Quase" porque dissemos que a ordem
não importa! Como os números sao pequenos, melhor fazer logo no braço...
Suponha s.p.d.g que a>=b>=c, e teste a=5, depois a=4... e liste os casos:
{a,b,c}={{5,0,0},{4,1,0},{3,2,0},{3,1,1},{2,2,1}}
Ou seja, sao apenas 5 triangulos:
{x,y,z} = {1,6,6},{2,5,6},{3,4,6},{3,5,5},{4,4,5}

---///---

Agora, se você quiser a minha interpretação original onde cada posição de
cada vértice importa... Bom, basta notar que:
a) Cada uma das triplas (1,6,6), (3,5,5) e (4,4,5) gera 13 triângulos (tome
um triângulo desse tipo e rode sucessivamente de ângulo 2pi/13)
b) Cada uma das triplas (2,5,6) e (3,4,6) gera 2x13=26 triângulos... Isto
ocorre aqui pois (2,5,6) gera um triângulo "distinto" de (2,6,5) (e um não
pode ser obtido do outro por rotações, pois estas mantêm a ordem circular
dos números).
(Note que esta "duplicação" não ocorria em (a) por conta dos números
repetidos! Por exemplo, se você tentasse montar um triângulo (6,6,1),
digamos, P1-P7-P13, ele seria uma das rotações do (1,6,6) que eu jah tinha
contado (a saber: P13-P1-P7). Ou seja, temos que contar todas as
permutações de cada tripla, dividindo por 3 por conta das
3 permutações circulares que não geram nada de novo.)
Então com a minha interpretação original a resposta seria (3x1+2x2) x 13 =
91? Errei algo?

Abraço, Ralph.




On Thu, Jun 18, 2020 at 9:31 PM Vitório Batista Lima da Silva <
vitorio.si...@trf1.jus.br> wrote:

> 3 vértices distintos de um polígono regular de 13 lados formam um
> triângulo. Quantos desses triângulos contém o centro do círculo
> circunscrito ao polígono?
>
> A resposta é 36???
>
> At.te,
>
> Vitório
>
> --
> Esta mensagem foi verificada pelo sistema de antivírus e
> acredita-se estar livre de perigo.
>

-- 
Esta mensagem foi verificada pelo sistema de antiv�rus e
 acredita-se estar livre de perigo.



[obm-l] Re: [obm-l] Encontrar K mínimo

2020-05-23 Por tôpico Ralph Costa Teixeira
Hm: "qualquer uma das 2021 possui voo direto", assim mesmo? Tenho ideias,
mas tem que completar.

Vou dizer que uma marcação "cuida" de uma cidade X quando existe alguma
cidade marcada com voo direto para X. Observe que, interpretando ao pé da
letra o enunciado, X não necessariamente cuida de X!

LEMA 1: Para um grafo do tipo "filete", Y1-Y2-...-YN conectadas em linha,
N>=3, o número mínimo f(N) de cidades marcadas satisfaz f(N)<=2N/3. Mais:
se N não é divisível por 3, podemos "economizar mais uma marcação" para
obter f(N)<2N/3.

Prova:
-- Para N=3, marque Y1Y2, portanto f(3)<=2.
-- Para N=4, marque Y2Y3, portanto f(4)<=2<8/3.
-- Para N=5, marque Y2Y3Y4, portanto f(5)<=3<10/3.
-- Para N=6, marque Y1Y2Y4Y5, portanto f(6)<=4.
-- Para N=7, marque Y2Y3Y5Y6 portanto f(7)<=4<14/3.
-- Para N=8, marque Y2Y3Y6Y7, portanto f(8)<=4<16/3.
...
Em geral, para N>=6, a marcação depende do resto de N na divisão por 3:
i) Se N=3k, divida as cidades em grupos de 3 e marque as 2 primeiras de
cada grupo: note que cada 2 cidades "cuidam" do seu grupo de 3, portanto
f(N)<=2N/3.
ii) Se N=3k+1, separe Y1, e divida **as outras** em grupos de 3; novamente,
marque as 2 primeiras de cada grupo (note que Y2 cuida de Y1!):
f(N)<=2(N-1)/3<2N/3.
iii) Se N=3k+2, separe as 5 primeiras e divida o resto em grupos de 3;
marque Y2Y3 e as 2 primeiras de cada grupo (observe que Y6 cuida de Y5).
Assim f(N)<=2(N-5)/3+2<2N/3.

(Obs: de fato, para N grande, o ideal se parece mais com f(N)~~N/2: divida
em grupos de 4, escolha as duas cidade do meio em cada grupo.)

---///---

Isso me faz desconfiar que o pior caso seria algo do tipo 2N/3, se tivermos
filetes de tamanho 3 ou 6. Assim fica fácil construir um exemplo onde
~~2N/3 seria necessário:

---///---

LEMA 2: k>=(2/3)*2021-1, ou seja, k>=1347.

Prova. Considere o caso onde as cidades são A, B1, B2, ..., B673, C1, C2,
..., C673, D1, D2, ..., D673, Z. Conecte apenas assim:
-- A conecta com Z;
-- A conecta com Bi para todo i;
-- Bi conecta com Ci para todo i;
-- Ci conecta com Di para todo i.
Em suma, uma cidade "central" A, donde saem 673 filetes de comprimento 3 (e
mais um filetezinho para conectar com a cidade que sobrou).

Para cuidar de Di, temos que marcar Ci. Para cuidar de Ci, temos que marcar
Bi ou Di (sendo esperto, escolhemos pelo menos um Bi para cuidar de A);
para cuidar de Z temos que marcar A.

Em suma, neste caso, a marcação deve ter pelo menos 673*2+1=1347 cidades.
---///---

Agora o que eu queria fazer era dividir o grafo em vários "filetes"
*disjuntos*, e em cada filete fazer uma marcação com 2/3 dos vértices
daquele filete, cuidando daquele filete. Juntando tudo, teríamos 2/3 dos
vértices marcados; como pelo menos um filete teria numero de vértices não
divisível por 3, eu "economizaria" uma marcação nele, chegando aos 1347.

O problema seria garantir que esses filetes tem comprimento >=3 para poder
usar o 2N/3... Se ficarmos com filetes de tamanho 1 ou 2, f(2)=2 estraga
tudo (para o de comprimento 1, também teríamos problemas). Então minha
ideia não funciona assim direto. Mas talvez ajude a pensar, ou achar um
pior caso que seja pior que a minha "estrela" ali em cima...

Abraço, Ralph.

On Sat, May 23, 2020 at 11:46 AM Jeferson Almir 
wrote:

> Amigos peço ajuda nesse problema, ou até algum resultado de grafos que
> resolva.
>
> Terra Brasilis  possui 2021 cidades, e existem voos de ida e volta entre
> algumas dessas  cidades de maneira que é possível chegar a qualquer outra
> através de voos finitos. Encontre o menor inteiro positivo k tal que,
> independente da configuração dos voos, é possível escolher k cidades de
> modo que qualquer uma das 2021 cidades possui voo direto para alguma das
> cidades marcadas.
>
> --
> Esta mensagem foi verificada pelo sistema de antivírus e
> acredita-se estar livre de perigo.

-- 
Esta mensagem foi verificada pelo sistema de antiv�rus e
 acredita-se estar livre de perigo.



[obm-l] Re: [obm-l] Re: [obm-l] Re: [obm-l] Decrescimento de Funções Exponenciais

2020-05-12 Por tôpico Ralph Costa Teixeira
Bom, o assunto me parece ser "crescimento/decrescimento assintótico"...
Não consigo pensar num texto para recomendar, mas olhe aqui:
https://en.wikipedia.org/wiki/Big_O_notation
E, em especial:
https://en.wikipedia.org/wiki/Big_O_notation#Little-o_notation

Abraço, Ralph.

On Tue, May 12, 2020 at 7:09 PM Luiz Antonio Rodrigues <
rodrigue...@gmail.com> wrote:

> Olá, Ralph!
> Tudo bem?
> Sim, melhorou muito!
> Muito obrigado!
> Então, na função (5), nós temos uma incerteza...
> Eu não havia percebido isso...
> Muito interessante...
> Vou ler mais sobre o assunto...
> Você conhece algum bom livro que trate disso com mais profundidade?
> Abraço!
> Luiz
>
>
> Em ter, 12 de mai de 2020 3:04 PM, Ralph Costa Teixeira 
> escreveu:
>
>> O assunto é delicado. Primeiro, precisamos de uma boa definição de
>> "decresce mais rápido" (a gente diz que as exponenciais decrescem rápido,
>> mas se você ler **ao pé da letra** isso é falso! A velocidade delas vai
>> para 0 quando t vai para infinito... ou seja, elas decrescem mito
>> devagar!?!?). Para esclarecer, suponho que queremos usar esta aqui:
>>
>> DEF. f(x) decresce (para 0) mais rápido (quando x vai para +Inf) do que
>> g(x) quando lim f(x)/g(x) =0 (quando x vai para +Inf).
>>
>> Agora sim, você resolve tudo:
>>
>> 1) lim h(x)^2/h(x) = 0, portanto h^2 decresce mais rapido que h;
>> 2) lim g(x)^2/h(x) = lim g(x)/h(x) . g(x) = 0.0=0, portanto g^2 decresce
>> mais rapido que h;
>> 3) lim f(x)*g(x)/h(x) = lim f(x) * (g(x)/h(x)) =0 (com f limitada),
>> portanto fg decresce mais rapido que h;
>> 4) lim sqrt(h)/h = lim 1/sqrt(h) =+Inf; assim, lim h/sqrt(h) = 0, ou
>> seja, h decresce mais rapido que sqrt(h);
>> 5) lim sqrt(g)/h = ??? Nao da para saber. Poderia ser g(x)=1/x^n e
>> h(x)=1/x. Tomando n<2 ou n>2 podemos obter ambos comportamentos.
>>
>> Melhorou?
>>
>> Abraço, Ralph.
>>
>> On Tue, May 12, 2020 at 9:52 AM Luiz Antonio Rodrigues <
>> rodrigue...@gmail.com> wrote:
>>
>>> Olá, pessoal!
>>>
>>> Bom dia!
>>>
>>> Tudo bem?
>>>
>>> Estou tentando resolver um problema há uns 10 dias.
>>>
>>> Já tentei de tudo e estou com dúvidas.
>>>
>>> O problema é o seguinte:
>>>
>>> São dadas duas funções: h(x) e g(x).
>>>
>>> A função g(x) tende a zero mais rápido do que h(x), quando x tende a
>>> infinito.
>>>
>>> O problema pede que as seguintes funções sejam comparadas com h(x):
>>>
>>>
>>>1.
>>>
>>>(h(x))^2
>>>2.
>>>
>>>(g(x))^2
>>>3.
>>>
>>>f(x)*g(x)
>>>4.
>>>
>>>sqrt(h(x))
>>>5.
>>>
>>>sqrt(g(x))
>>>
>>>
>>> A pergunta é: quais dessas funções decrescem mais rápido do que h(x),
>>> quando x tende a infinito?
>>>
>>> Eu usei, entre outras, as seguintes funções:
>>>
>>>
>>> 1/ln(x)
>>>
>>> 1/x
>>>
>>> 1/x^5
>>>
>>> 1/e^x
>>>
>>>
>>> Utilizei a regra de L’Hospital e descobri que a única função que não
>>> decresce mais rápido do que h(x) é a (4).
>>>
>>> Também utilizei softwares gráficos e confirmei o meu resultado.
>>>
>>> Só sei que a resposta não está correta, mas ainda não sei qual seria a
>>> solução.
>>>
>>> Não consigo entender o motivo...
>>>
>>> Será que preciso achar um contra-exemplo?
>>>
>>> Alguém pode me ajudar?
>>>
>>> Muito obrigado!
>>>
>>> Abraços!
>>>
>>> Luiz
>>>
>>> --
>>> Esta mensagem foi verificada pelo sistema de antivírus e
>>> acredita-se estar livre de perigo.
>>
>>
>> --
>> Esta mensagem foi verificada pelo sistema de antivírus e
>> acredita-se estar livre de perigo.
>
>
> --
> Esta mensagem foi verificada pelo sistema de antivírus e
> acredita-se estar livre de perigo.

-- 
Esta mensagem foi verificada pelo sistema de antiv�rus e
 acredita-se estar livre de perigo.



[obm-l] Re: [obm-l] Decrescimento de Funções Exponenciais

2020-05-12 Por tôpico Ralph Costa Teixeira
P.S.: Na (3) se ele nao falou que f eh limitada, a resposta passa a ser NAO
SEI.

On Tue, May 12, 2020 at 2:52 PM Ralph Costa Teixeira 
wrote:

> O assunto é delicado. Primeiro, precisamos de uma boa definição de
> "decresce mais rápido" (a gente diz que as exponenciais decrescem rápido,
> mas se você ler **ao pé da letra** isso é falso! A velocidade delas vai
> para 0 quando t vai para infinito... ou seja, elas decrescem mito
> devagar!?!?). Para esclarecer, suponho que queremos usar esta aqui:
>
> DEF. f(x) decresce (para 0) mais rápido (quando x vai para +Inf) do que
> g(x) quando lim f(x)/g(x) =0 (quando x vai para +Inf).
>
> Agora sim, você resolve tudo:
>
> 1) lim h(x)^2/h(x) = 0, portanto h^2 decresce mais rapido que h;
> 2) lim g(x)^2/h(x) = lim g(x)/h(x) . g(x) = 0.0=0, portanto g^2 decresce
> mais rapido que h;
> 3) lim f(x)*g(x)/h(x) = lim f(x) * (g(x)/h(x)) =0 (com f limitada),
> portanto fg decresce mais rapido que h;
> 4) lim sqrt(h)/h = lim 1/sqrt(h) =+Inf; assim, lim h/sqrt(h) = 0, ou seja,
> h decresce mais rapido que sqrt(h);
> 5) lim sqrt(g)/h = ??? Nao da para saber. Poderia ser g(x)=1/x^n e
> h(x)=1/x. Tomando n<2 ou n>2 podemos obter ambos comportamentos.
>
> Melhorou?
>
> Abraço, Ralph.
>
> On Tue, May 12, 2020 at 9:52 AM Luiz Antonio Rodrigues <
> rodrigue...@gmail.com> wrote:
>
>> Olá, pessoal!
>>
>> Bom dia!
>>
>> Tudo bem?
>>
>> Estou tentando resolver um problema há uns 10 dias.
>>
>> Já tentei de tudo e estou com dúvidas.
>>
>> O problema é o seguinte:
>>
>> São dadas duas funções: h(x) e g(x).
>>
>> A função g(x) tende a zero mais rápido do que h(x), quando x tende a
>> infinito.
>>
>> O problema pede que as seguintes funções sejam comparadas com h(x):
>>
>>
>>1.
>>
>>(h(x))^2
>>2.
>>
>>(g(x))^2
>>3.
>>
>>f(x)*g(x)
>>4.
>>
>>sqrt(h(x))
>>5.
>>
>>sqrt(g(x))
>>
>>
>> A pergunta é: quais dessas funções decrescem mais rápido do que h(x),
>> quando x tende a infinito?
>>
>> Eu usei, entre outras, as seguintes funções:
>>
>>
>> 1/ln(x)
>>
>> 1/x
>>
>> 1/x^5
>>
>> 1/e^x
>>
>>
>> Utilizei a regra de L’Hospital e descobri que a única função que não
>> decresce mais rápido do que h(x) é a (4).
>>
>> Também utilizei softwares gráficos e confirmei o meu resultado.
>>
>> Só sei que a resposta não está correta, mas ainda não sei qual seria a
>> solução.
>>
>> Não consigo entender o motivo...
>>
>> Será que preciso achar um contra-exemplo?
>>
>> Alguém pode me ajudar?
>>
>> Muito obrigado!
>>
>> Abraços!
>>
>> Luiz
>>
>> --
>> Esta mensagem foi verificada pelo sistema de antivírus e
>> acredita-se estar livre de perigo.
>
>

-- 
Esta mensagem foi verificada pelo sistema de antiv�rus e
 acredita-se estar livre de perigo.



[obm-l] Re: [obm-l] Decrescimento de Funções Exponenciais

2020-05-12 Por tôpico Ralph Costa Teixeira
O assunto é delicado. Primeiro, precisamos de uma boa definição de
"decresce mais rápido" (a gente diz que as exponenciais decrescem rápido,
mas se você ler **ao pé da letra** isso é falso! A velocidade delas vai
para 0 quando t vai para infinito... ou seja, elas decrescem mito
devagar!?!?). Para esclarecer, suponho que queremos usar esta aqui:

DEF. f(x) decresce (para 0) mais rápido (quando x vai para +Inf) do que
g(x) quando lim f(x)/g(x) =0 (quando x vai para +Inf).

Agora sim, você resolve tudo:

1) lim h(x)^2/h(x) = 0, portanto h^2 decresce mais rapido que h;
2) lim g(x)^2/h(x) = lim g(x)/h(x) . g(x) = 0.0=0, portanto g^2 decresce
mais rapido que h;
3) lim f(x)*g(x)/h(x) = lim f(x) * (g(x)/h(x)) =0 (com f limitada),
portanto fg decresce mais rapido que h;
4) lim sqrt(h)/h = lim 1/sqrt(h) =+Inf; assim, lim h/sqrt(h) = 0, ou seja,
h decresce mais rapido que sqrt(h);
5) lim sqrt(g)/h = ??? Nao da para saber. Poderia ser g(x)=1/x^n e
h(x)=1/x. Tomando n<2 ou n>2 podemos obter ambos comportamentos.

Melhorou?

Abraço, Ralph.

On Tue, May 12, 2020 at 9:52 AM Luiz Antonio Rodrigues <
rodrigue...@gmail.com> wrote:

> Olá, pessoal!
>
> Bom dia!
>
> Tudo bem?
>
> Estou tentando resolver um problema há uns 10 dias.
>
> Já tentei de tudo e estou com dúvidas.
>
> O problema é o seguinte:
>
> São dadas duas funções: h(x) e g(x).
>
> A função g(x) tende a zero mais rápido do que h(x), quando x tende a
> infinito.
>
> O problema pede que as seguintes funções sejam comparadas com h(x):
>
>
>1.
>
>(h(x))^2
>2.
>
>(g(x))^2
>3.
>
>f(x)*g(x)
>4.
>
>sqrt(h(x))
>5.
>
>sqrt(g(x))
>
>
> A pergunta é: quais dessas funções decrescem mais rápido do que h(x),
> quando x tende a infinito?
>
> Eu usei, entre outras, as seguintes funções:
>
>
> 1/ln(x)
>
> 1/x
>
> 1/x^5
>
> 1/e^x
>
>
> Utilizei a regra de L’Hospital e descobri que a única função que não
> decresce mais rápido do que h(x) é a (4).
>
> Também utilizei softwares gráficos e confirmei o meu resultado.
>
> Só sei que a resposta não está correta, mas ainda não sei qual seria a
> solução.
>
> Não consigo entender o motivo...
>
> Será que preciso achar um contra-exemplo?
>
> Alguém pode me ajudar?
>
> Muito obrigado!
>
> Abraços!
>
> Luiz
>
> --
> Esta mensagem foi verificada pelo sistema de antivírus e
> acredita-se estar livre de perigo.

-- 
Esta mensagem foi verificada pelo sistema de antiv�rus e
 acredita-se estar livre de perigo.



[obm-l] Re: [obm-l] teoria dos números

2020-04-11 Por tôpico Ralph Costa Teixeira
Tome por exemplo
a=1
b=xy
c=y

Mais genericamente
a=k
b=kxy
c=ky
servem para k≠0 complexo qualquer.

On Sat, Apr 11, 2020, 11:17 Israel Meireles Chrisostomo <
israelmchrisost...@gmail.com> wrote:

> Como posso provar a seguinte afirmação "Sejam x,y,z números complexos tais
> que xyz=1, mostre que existem a,b,c complexos tais que b/c=x,c/a=y,a/b=z"
>
>
> --
> Israel Meireles Chrisostomo
>
> --
> Esta mensagem foi verificada pelo sistema de antivírus e
> acredita-se estar livre de perigo.

-- 
Esta mensagem foi verificada pelo sistema de antiv�rus e
 acredita-se estar livre de perigo.



Re: Problema Republica Tcheca

2001-07-20 Por tôpico Ralph Costa Teixeira

 Seja z(n)=n+[n/4]+[n/8]+[n/16]+...

Legal sua solução, Bruno. Voce definiu g(n) como o numero de vezes que
2 divide n. Então o somatório de g(k) de 1 até n é o número de 2 que
dividem todos os números de 1 até k, por assim dizer. Você pode calcular
isso assim: olhe para os números de 1 a k e

conte 1 para cada número par
conte 1 extra para os múltiplos de 4
conte 1 mais para os múltiplos de 8
...

Chegando à sua fórmula:

SUM(g(k), 1 a n) = [n/2]+[n/4]+[n/8]+...

Para f(n), adicione uma linha às regras:

conte 1 para cada ímpar

EnTão SUM(f(k), 1 a n) = [(n+1)/2]+[n/2]+[n/4]+[n/8]+[n/16]+...

que corresponde à sua fórmula para z(n) quando n é par.

 Fazendo umas continhas, vemos que z(82308)=123457

E aqui, não sei qual método você usou para achar o 82308, mas eu pensava
em algo assim para ajudar:

z(n) = n + n/4 + n/8 + n/16 + ... = n + n/2 = 3n/2

Então n = 2z(n)/3.

Se z(n)=123457, então n=2.123457/3 = 82304,???. Assim, n=82305 é um
ótimo lugar para começar a busca...

Abraço,
Ralph






Re: sokoban

2001-07-20 Por tôpico Ralph Costa Teixeira

Oi, gente.

Que seja dito que alguns algoritmos de força bruta ainda resolvem se o
espaço de estados não for imenso; mas quando o espaço cresce... :(. Eu estou
maravilhado com o site do Sethian

http://www.math.berkeley.edu/~sethian/

que tem uma classe de algoritmos chamados Fast Marching Methods que, por um
lado, não têm nada de mais, mas são rápidos porque fazem a força bruta na
ordem certa. Para um exemplo, vejam o exemplo do robozinho na pista de
obstáculos

http://www.math.berkeley.edu/~sethian/Applets/java_robotics.html

Essencialmente, o algoritmo analisa TODAS as posições do robozinho
possíveis no grid (um espaço tridimensional -- duas variáveis para as
coordenadas do centro de massa, uma para o ângulo de rotação) e determina se
são possíveis ou não, e quão longe cada uma delas está do estado inicial (o
truque é analisar os estados na ordem correta; para quem já viu programação
dinâmica, é quase a mesma coisa; aliás, para quem quer pensar nisso e não
viu programação dinâmica, procure aprender, não é muito difícil e é bem
bonitinho; infelizmente não me lembro do texto onde eu aprendi). Daí para
achar caminhos mínimos é um pulinho.

O problema do Sokoban pode ser resolvido com um algoritmo força bruta
tipo programação dinâmica... O problema é que o número de estados possíveis
é MUITO mais alto do que nesse exemplo do robozinho. No robo, a coisa é do
tipo N^3 (digamos, NxN posições possíveis para o centro de massa, N
possibilidades de ângulos de rotação). No Sokoban, se o labirinto tem umas
M^2 casas e temos uns P objetos (mais o jogador), há então uns M^2.C(M^2,P)
possíveis estados do jogo, que tem uma cara de M^(2P)... Eu ainda apostaria
(se alguém souber melhor, confirme ou disminta) que um algoritmo tipo
programação dinâmica resolve os problemas do Sokoban QUE TÊM SOLUÇÃO que A
GENTE ENCONTRA no joguinho, mas deve se perder completamente nos
impossíveis...

Abraço,
Ralph






Re: Questão fácil ...

2001-07-17 Por tôpico Ralph Costa Teixeira


Oi gente.

Tudo o que todo mundo falou me parece correto. Eu soh queria
destacar eh que a confusao nao parece vir da logica nao, mas no
uso das palavras antes e depois. Veja bem, nao eh muito claro se
depois quer dizer na frente de ou atras de. Tem gente que usa de um
jeito, tem gente que usa de outro. Eu prefiro usar que carro A estah
depois de carro B como carro A estah atras de carro B, o que dah a
interpretacao do pessoal que respondeu a esta mensagem (imaginando que eu
vejo a frente do primeiro carro e DEPOIS vejo os outros atras deste). A
resposta seria entao Preto, como no gabarito.

Mas ha pessoas que pensam em antes como atras e depois como
na frente (imagine-se atras do ultimo, vendo a traseira deles, e esta
interpretacao passa a ser mais natural!). Acho que o Marcus estah usando
esta interpretacao, e entao tudo que ele escreveu passa a fazer sentido.
Usando esta segunda interpretacao a unica solucao seria:

Preto (atras), DEPOIS Azul, DEPOIS Amarelo e DEPOIS, na frente,
Verde.

Se voce acha esquisito, imagine-se de novo ATRAS do ultimo carro e
tudo fara sentido.

Se fosse de Olimpiada, eu votava para anular. Se fosse questao
proposta, eu votava por uniformizar e usar soh na frente de e atras
de ao inves de antes e depois -- ai diminui a confusao.

Abraco,
Ralph

On Sat, 14 Jul 2001, Odelir Maria Casanova dos Santos wrote:

 Essa questão é MUITO fácil, mas eu agradeço quem poder me ajudar .

 Quatro carros, de cores amarelas, verde, azul, e preta, estão em fila.
Sabe-se que o carro que está imediatamente antes do carro azul é menor do
que o que está imediatamente depois do carro azul; que o carro verde é o
menor de todos, que o carro verde está depois do carro azul, e que o
carro amarelo está depois do preto. O primeiro carro:

 a) é amarelo
 b) é azul
 c) é preto
 d) é verde
 e) não pode ser determinado apenas com esses dados.

 A resposta do gabarito é c) é preto, mas se o carro verde é menor que
todos e fica na frente do carro azul, ele deve ficar na do carro que fica
na frente do carro azul, e é impossível o carro preto ser o primeiro pois
o amarelo fica na sua frente. Não tenho certeza, mas acho que essa
questão é de uma olimpíada brasileira antiga, só que eu encontrei isso em
um livro antigo.

 Falou pessoal
 Marcus Dimitri






Re: Certo ou errado?

2001-07-09 Por tôpico Ralph Costa Teixeira


Oi, Gustavo.

Talvez isso ajude um pouco... ou talvez atrapalhe muito... :)

Quando eu digo que 1 litro = 10 decilitros, podemos usar uma
proporcionalidade (regra de 3) para converter unidades.
Assim, 25.4 litros = 25.4 x 10 = 254 decilitros. Ateh aqui, tudo bem, nada
de mais. A grande maioria das unidades que a gente usa respeita este tipo de
proporcionalidade, a gente pode usar regra de 3, usar a mesma unidade para
medir KOISA e VARIACAO DA KOISA e tudo funciona. Agora, tudo funciona
porque:

a) Um litro igual a 10 decilitros SEMPRE. Em outras palavras, se o primeiro
litro que entra na caixa dagua equivale a 10 decilitros, o proximo tambem
equivale a mais 10 decilitros. Isso estabelece que a relacao entre os
volumes medidos em Litros e em Decilitros EH LINEAR.

b) Uma caixa dagua com 0 litros tem 0 decilitros. Esta CONVENCAO estabelece
que, mais do que linear, a relacao eh uma PROPORCAO. Um conceito do tipo o
volume da caixa dobrou eh verdadeiro ou falso independentemente do sistema
de unidades utilizado.

Quando (a) e (b) valem, o sistema de unidades utilizado eh aditivo e
proporcional e a gente pode usar essas unidades do jeito que voce fez lah
embaixo com as regrinhas de conversao que voce usou.

Infelizmente, enquanto a regra (a) vale para temperaturas em Celsius e
Kelvin (e em Fahrenheit tambem), a (b) nao vale. Note que:

i) Cada 100 graus Celsius ADICIONAIS equivalem a 100 graus Kelvin ADICIONAIS
(e 180 Farenheit adicionais). Se TC, TK e TF sao os numeros que medem
temperaturas em Celsius, Kelvin e Fahrenheit, entao

DELTATC = DELTATK = DELTATF/1.8

Note os DELTAS, aqui usados para indicar VARIACAO de temperatura. Repito, a
gente usa Grau C, Grau K e Grau F para medir VARIACAO de temperatura, e
neste sentido pode-se escrever

1 Grau Celsius = 1 Grau Kelvin = 1.8 Graus Fahrenheit

De novo: isto soh vale no sentido de VARIACAO de temperatura, nao no sentido
de numero que representa a temperatura em C ou K ou F.

ii) No entanto, os ZEROS dessas escalas estao em lugares diferentes! Assim,
nao ha proporcionalidade! Se Tc eh o NUMERO que representa a temperatura em
Celsius, Tk em Kelvin, e Tf em Fahrenheit, temos:

Tk=Tc+273 (Pois o 0 dos C eh o 273 dos K)
Tf=Tc*1.8+32 (Pois o 0 dos C eh o 32 dos F)

Quando uma temperatura dobra em Celsius, nao necessariamente dobra em
Fahrenheit ou Kelvin.

--//--

Assim, ha um uso um pouco relaxado demais quando a gente escreve:

1K = -272C

esta frase nao quer dizer que CADA grau Kelvin vale -272 graus Celsius
(apesar de parecer isso). A gente escreveu um pouco estranho, mas a gente
simplesmente estah dizendo que a temperatura que equivale ao numero 1 na
escala K eh a temperatura que equivale ao numero -272 na escala Celsius.
Esse 1K nao se multiplica ou divide por nada; nao existe o conceito de a
temperatura dobrou ou multiplicou-se em qualquer sentido fixo e invariavel
por mudanca de escala utilizada.

Por outro lado:

0K = 0K = 0x(1K)
4K = 2x(2K) = 4x(1K) = 100x(0.04K)
...

ou qualquer outra multiplicacao dessas estah usando 1K como uma VARIACAO de
graus Kelvin. De fato, uma VARIACAO de 0K eh igual a 0 variacoes de 1 grau
K. Uma VARIACAO de 4K eh igual a 4 variacoes de 1K. Estas frases nao fazem
NENHUM SENTIDO se voce pensar em quadruplicar a temperatura de 1K, jah que
o conceito de quadruplicar depende completamente do sistema de unidades
utilizado! Por isso, ESTE 1K nao pode ser substituido por -272C (AQUELE 1K
lah de cima nao era uma VARIACAO de temperatura, era simplesmente o numero
que a gente escolheu para REPRESENTAR uma certa temperatura).

O exemplo da temperatura eh esquisito porque ainda existe uma nocao de
Graus Proporcionais na VARIACAO de temperatura, apesar da proporcionalidade
nao se extender aos numeros que representam a temperatura em si. Se a gente
matar tambem a propriedade (i), nao vale a proporcionalidade nem mesmo em
VARIACAO, e o proprio conceito de UNIDADE fica meio estranho.

Por exemplo, nao ha graus Richter no sentido proporcional da palavra.
Aumentar um terremoto em 1 grau Richter pode significar coisas diferentes
dependendo donde voce comecou a variacao: por exemplo, de 1 grau Richter
para 2 graus Richter ninguem percebe a diferenca, mas de 6 para 7 faz uma
diferenca terrivel. O conceito de 1 grau Richter no sentido de variacao,
independente de onde voce comecou, nao existe. Coisas semelhantes podem ser
ditas para Decibeis (essa eh discutivel, porem, jah que o nosso ouvido
parece perceber de uma forma logaritmica...) e Aumentos Porcentuais (2
aumentos de 10% nao sao um de 20%; um aumento de 10% significa coisas
distintas dependendo donde voce comeca; nao podemos usar regras de 3 com
porcentagens).

Em suma, soh use a regra de tres quando voce puder.

Abraco,
Ralph

-Original Message-
From: Gustavo Nunes Martins [EMAIL PROTECTED]
To: [EMAIL PROTECTED] [EMAIL PROTECTED]
Date: Friday, July 06, 2001 10:05 PM
Subject: Certo ou errado?


Lembrando 

Re: 0,8... + 0,1... = 1 ?!

2001-06-23 Por tôpico Ralph Costa Teixeira


Oi, Gustavo.

Na minha opiniao, 0,...=1. Sim, eh isso mesmo eh 1. Nao tem
nada entre esses dois numeros. O primeiro vale 1, o segundo tambem. :)

Eu mandei uma mensagem uma vez que tentava esclarecer um pouco
porque as pessoas acham isso surpreendente (nao tanto porque eh 1 -- para
isso o seu argumento funciona, e hah outros equivalentes como o do
Daniel). Procure no arquivo da Olimpiada, em Abril de 2000, sob o titulo
O Dia que nao acaba.

Abraco,
Ralph

P.S.: O arquivo da Olimpiada que o Nicolau mantem com tanto carinho estah
em http://www.mat.puc-rio.br/~nicolau/olimp/obm-l.html

P.S.2: Alias, esta questao tambem jah havia aparecido antes em Maio de
1999. Veja tambem

http://www.mat.puc-rio.br/~nicolau/olimp/obm-rj.1999/threads.html#00143
sob o titulo Dizima ou nao-dizima.

On Sat, 23 Jun 2001, Augusto Morgado wrote:

 Eu acho estranho que alguns achem estranho 0,999...=1 e não achem
 estranho 0,333...=1/3.
 Morgado

 Gustavo Nunes Martins wrote:
 
  8/9 = 0,888...
  1/9 = 0,111...
  0,888... + 0,111... = 0,999...
  8/9 + 1/9 = 9/9 = 1
  Entao:
  0,999... = 1
 
  Nao entendo que 0,999... seja IGUAL a 1. Suponho que seja diferente de
  1. Alguem pode me explicar o que esta coisa significa?
 
  Atenciosamente,
  Gustavo





Re: Desafio: De Tabela p/ Função

2001-06-18 Por tôpico Ralph Costa Teixeira

H De novo eu mandei uma mensagem e acho que ela foi engolida
pelas maquinas...

Eu tinha escrito algumas sugestoes para FORMULAS que descrevam essa
funcao dada pela tabela. Por exemplo

x(n)=4+SGN(sin(2nPi/5))

onde SGN eh a funcao (SINAL DE). Se voce quiser uma formula mais estavel
(importante se o objetivo eh usar um computador), voce pode usar:

x(n)=4+ROUND(a sin(2nPi/5))

onde a eh uma constante entre 1.5/sin(2Pi/5) e 0.5/sin(4Pi/5) (vale a
pena notar que a=1 serve). Por exemplo:

x(n)=4+ROUND(sin(2nPi/5))

Enfim, se voce preferir uma formula com INT (parte inteira de),
lembre-se de que

ROUND(x) = INT(x+0.5)

Assim, x(n)=INT(sin(2nPi/5)+0.5) tambem serve.

Isso tudo dito Voce jah tinha uma expressao simples para a funcao,
dada pela tabelinha (imagino que n soh possa assumir valores de 0 a 5, eh
isso mesmo?). A titulo de curiosidade, por que a gente estava procurando uma
FORMULA? Implementacao no computador?

Abraco,
Ralph

P.S.: Eh que eu acho legal a gente lancar essa nocao de que uma FUNCAO pode
muito bem ser determinada por um grafico ou uma tabela, sem uma formula; a
gente aas vezes acaba se prendendo muito aas formulas :)

-Original Message-
From: Bruno Schroeder [EMAIL PROTECTED]
To: [EMAIL PROTECTED] [EMAIL PROTECTED]
Date: Saturday, June 16, 2001 1:03 AM
Subject: Desafio: De Tabela p/ Função


Amigos,

 Tenho uma tabela que queria transformar em duas funções, é um desafio
interessante.
n=0 x=4 y=2
n=1 x=5 y=2
n=2 x=5 y=1
n=3 x=3 y=1
n=4 x=3 y=0
n=5 x=4 y=0

 Para y em função de n eu consegui:
 y=abs((n/2)-2) = |(n/2)-2|
 O domínio e a imagem são os Inteiros.
 Porém não consigo fazer x em função de n. Notem que p/ n=2 e n=3, os
valores podem ser trocados sem alterar em nada a função, visto que em ambos
os valores y=1.
 Para resolver eu tentei botar os pontos no plano cartesiano e buscar uma
função que se aproxime, a que eu achei foi: x=sen(n/3)+4, domínio inteiros,
porém acho que está errada.
 Alguém pode me ajudar a achar x(n), por favor?

 Amplexos,

Bruno Schroeder




Re: dúvidas

2001-06-06 Por tôpico Ralph Costa Teixeira

Para ser chato, na primeira questao falta mostrar que HA pelo menos uma
funcao satisfazendo tais caracteristicas (senao a resposta seria
INDETERMINADA). Jah perdi ponto em Olimpiada por causa disso... :( :( :( :)

Neste caso, basta notar que f(x)=0 eh uma de tais funcoes. :) :)

Abraco,
Ralph

-Original Message-
From: Eduardo Casagrande Stabel [EMAIL PROTECTED]

From: Henrique Lima Santana [EMAIL PROTECTED]

 1: sabendo q f(n)=0 se n tem algarismo das unidades = 4 e f(ab)=f(a)+
 +f(b), qual o valor de f(1998) ?

Tente fatorar 1998, com algum fator que tenha o 4 nas unidades.
f(1998) = f(54 * 37) = f(54) + f(37) = 0 + f(37) = f(37)

Agora veja que.
f(37*2) = f(74) = 0 = f(37) + f(2)

Dai f(37) = -f(2)

E f(4) = f(2 * 2) = 2*f(2) = 0, portanto f(37) = 0 e f(1998) =  0




Re: expressões

2001-06-06 Por tôpico Ralph Costa Teixeira

H filosoficamente falando, aposto que se adicoes viessem antes,
acabariamos omitindo os + e nao os x, isto eh:

a+bxc

que a gente escreve e interpreta como

a+bc = a+(bxc)

acabaria sendo escrito e interpretado como

a(bxc)=a+(bxc)

enquanto (a+b)xc que a gente escreve e interpreta da forma usual,
passaria a ser

abxc = (a+b)xc

mas isso eh bobagem minha... :) :) :)

Abraco,
Ralph





-Original Message-
From: josimat [EMAIL PROTECTED]
To: [EMAIL PROTECTED] [EMAIL PROTECTED]
Date: Monday, June 04, 2001 10:40 AM
Subject: Re: expressões



Valeu Nicolau! A questão é: quais as alterações que seriam produzidas no
mundo se 2 + 3 x 5 = 25, isto é, a multiplicação ou divisão feita depois da
adição ou da subtração. Acredito que a resposta seja: nenhuma.
Estabelecendo
também que, por exemplo,  5x3^2= 15^2, a equação 3*x^2 - 5*x = 0 seria
escrita como
(3*(x^2)) - (5*x)=0. E, com isso, ter-se-ia muito mais trabalho, mas creio
que nada além disso seria mudado.

[]s, Josimar
-Mensagem original-
De: Nicolau C. Saldanha [EMAIL PROTECTED]
Para: OBM [EMAIL PROTECTED]
Data: Sexta-feira, 1 de Junho de 2001 09:02
Assunto: Re: expressões




On Thu, 31 May 2001, josimat wrote:

 Olá amigos!  2 + 3 x 5 = 17  e  (2 + 3) x 5 = 30.  Conversando com um
amigo
 meu, ele me perguntou se eu poderia mostrar que a regra só poderia ser
esta,
 isto é, na primeira igualdade, a multiplicação deve ser feita antes da
 adição. Respondi-lhe que isto é uma convenção, pois poderia ser
diferente.
 Por exemplo, para efetuarmos a multiplicação antes, poderíamos
estabelecer
 que deveríamos escrever 2 + (3 x 5).  Meu amigo se foi mas deixou comigo
um
 sentimento de ter dito uma baita de uma besteira. Hoje, durante o pouco
tempo
 que tive, andei pensando na estrutura de corpo e coisas do gênero.
Liguei
 para dois amigos meus e ambos disseram que iriam pensar. Então suspeitei
de
 que a coisa não fosse assim tão boba, conforme pensara quando dei a
resposta
 a meu amigo. Pensei também que talvez valesse a pena evocar o tema nesta
 lista, pois o que é bobeira para alguns pode ser de interesse para
alguém.
 []s, Josimar


Para mim é bem claro que isto é pura convenção de notação, exatamente
como você respondeu no primeiro momento. []s, N.





Re: Ajuda urgente: cálculo do volume de um tanque.

2001-03-01 Por tôpico Ralph Costa Teixeira


Oi todo mundo.

Voltando ao problema do tanque deitado, as noticias nao sao nada boas
para o resto do problema. Acaba numa integral muito feia que eu creio
soh poder ser feita mesmo numericamente.


I. O CILINDRO

Na ultima mensagem eu disse que, se o nivel do liquido eh h a partir do
fundo de um cilindro de raio r e "comprimento" a (pois o cilindro estah
deitado), entao o volume do liquido lah dentro eh:

V1 = a.r^2.  [Pi + (m-1)sqrt(m(2-m)) - arccos(m-1)]

onde eu uso m=h/r para facilitar as coisas. Seria legal marcar o zero
da escala no centro do cilindro, isto eh, tomar h1 = h-r como variavel
ao inves de h. Assim, se m=h1/r

V1 = a.r^2. [Pi + m.sqrt(1-m^2) - arccos(m)]

Daqui por diante vou usar esta notacao, marcando h=0 no meio, e assim h
vai de -r a r. Quem nao gostar, troque h por h+r de volta. :)


II. CADA UMA DAS CALOTAS

Uma secao *horizontal* da calota esferica aa altura z (z=0 eh o plano
horizontal passando pelo centro da esfera) eh um segmento circular. Eu
peguei uma destas secoes HORIZONTAIS e desenhei-a aqui vista de cima,
preenchida com s's. O x marca o centro do circulo, R0 eh seu raio e d eh
a distancia entre o centro e o segmento que delimita o segmento
circular.

 |\
 |s\
 |ss\
  d  |sss|
x|sss|
 \   |sss|
  \  |ss/
 R0\ |s/
\|/

Como a secao horizontal estah aa distancia |z| do centro da esfera,
temos R0=sqrt(R^2-z^2).

Por outro lado, pode-se notar que d eh tambem a distancia do centro da
ESFERA (que nao eh necessariamente x! O centro da esfera estah na secao
horizontal z=0!) ao plano usado para corta-la em uma calota. Em outras
palavras, d=sqrt(R^2-r^2).

Enfim, lembre-se que a area do segmento circular eh a area de um setor
circular menos um triangulo escolhidos a dedo... A formula eh:

A = (R0)^2.arccos(d/R0) - d.sqrt(R0^2-d^2)

Substitua R0 e d:

A = (R^2-z^2).arccos(sqrt(R^2-r^2)/sqrt(R^2-z^2))
- sqrt(R^2-r^2).sqrt(r^2-z^2)

Agora voce teria que integrar isso de z=-r a z=h para achar o volume do
liquido. A segunda parte da integral (a segunda linha da area) eh facil
por substituicao, eh igual ao calculo feito para o cilindro. Tome de
novo m=h/r e fique com:

V3 = -r^2.sqrt(R^2-r^2). [Pi + m.sqrt(1-m^2) - arccos m]

A primeira parte eh pior ainda. Use z=R.cost, r/R=p e h/R=q para obter:

V2 = R^3 INT(t = arccos(q)  a   t = Pi - arccos(p))
 (sint)^3 . arccos(sqrt(1-p^2)/sint) dt

Ateh onde eu sei, esta integral nao pode ser resolvida analiticamente
(o arccos(K/sint) me faz acreditar nisto), a menos eh claro que p=1 (o
caso em que r=R, ou seja, em que as calotas sao de fato dois
hemisferios).

Assim, a melhor opcao eh fazer um calculo numerico desta integral
usando os seus dados a=14500, r=500 e R=3142... Note que V2 depende de q
de maneira "simples". Ponha varios valores de q e faca uma tabela... :(


III. JUNTANDO TUDO

Em suma, pegue um computador e calcule as seguintes quantidades para
cada h desejado de -r a r:

p=r/R; q=h/R; m=h/r=q/p;

DENTRO DO CILINDRO:
V1 = a.r^2.[Pi + m.sqrt(1-m^2) - arccos m]

NAS CALOTAS:
2V2 = R^3 INT(t = arccos(q); t = Pi - arccos(p))
 (sint)^3 . arccos(sqrt(1-p^2)/sint) dt

(Resolva numericamente para o valor p fixo que voce tem e usando
diversos valores de q)

2V3 = -r^2.sqrt(R^2-r^2). [Pi + m.sqrt(1-m^2) - arccos m]


O volume que voce quer eh V1+2V2+2V3.

Eu sei que a resposta parece um pouco decepcionante, mas espero que
tenha ajudado. As vezes eh mais facil fazer ao contrario: vah enchendo o
tanque com volumes conhecidos e marcando os valores de h para cada um,
montando assim a escala "experimentalmente"... Ou faca isso para um
tanque igual mas menor em escala... :)

   -
 / \
/   \
   | |
\---/
 \ /
  -
 
 
 Nesse caso temos:
 a = 14500mm; r = 500mm; R = 3142mm;
 em que, a = comprimento do cilindro (no considerar as calotas, e sim apenas
 o cilindro plano nos lados); r = raio do cilindro; R = raio da calota at a



Re: Ajuda urgente: cálculo do volume de um tanque.

2001-03-01 Por tôpico Ralph Costa Teixeira

Oi, Jose Paulo.

O seu ponto eh valido. Na minha opiniao, nenhum problema terrivele em
ficar com um problema numerico nas maos. A integral eh feia para
resolver "no braco", mas nem sempre uma formula "fechadinha" (no caso
aqui, sem integrais, usando soh as funcoes mais "comuns" como
exponenciais, trigonometricas e aritmeticas) eh melhor do que uma que
tenha um sinal de integral. Afinal, mesmo que a resposta fosse V=e^h,
o calculo disso acaba sendo feito "numericamente" na hora de marcar o
volume lah na escala do tanque. E esse calculo tambem envolve um erro
numerico, assim como o da integral --  e ambos os erros podem ser bem
controlados (dependendo do que estah dentro da integral).

Ha de se lembrar que muitas das funcoes que a gente aceita facilmente
podem muito bem ser consideradas como apelidos para integrais... Por
exemplo, muita gente DEFINE a funcao log natural por

ln x = INT (t=1 a t=x) 1/t dt

e entao DEFINE e^x como a funcao inversa de ln x. Assim, tais funcoes
nao seriam "melhores" que expressoes integrais...

Talvez esse outro exemplo reforce o meu ponto: a nem tao conhecida
funcao erf(x) eh definida como:

erf(x) = 2/sqrt(Pi) INT (t=0 a t=x) e^(-t^2) dt

Muita gente diria que a integral do lado direito "nao tem solucao".
Mas, se voce conhece erf(x), a integral eh trivial. Se voce trabalhar
bastante com a erf(x) e acostumar-se com suas propriedades, voce acaba
aceitando-a tanto como e^x ou ln(x) ou cos(x) -- diga-se de passagem,
a funcao erf de fato tem varias propriedades interessantes e aparece
naturalmente em varios contextos, especialmente quando falando da
distribuicao normal de probabilidade; suas tabelas sao bem conhecidas,
seu comportamento eh bem entendido. Hoje em dia, se eu me deparo com
uma resposta que tenha a integral de e^(-t^2) na expressao, eu me dou
por satisfeito e considero o problema resolvido, mesmo que eu nao
chegue a escrever erf(x) no lugar da integral.

Assim, nao vejo nada de intrinsicamente terrivel na integracao
numerica nao... Mas vejo como a minha mensagem anterior parece indicar
isso. :) Eu eh que fiquei meio decepcionado de nao conseguir uma
formula "fechadinha" para o problema... :) :) Uma das principais
razoes da minha decepcao eh que o Software que eu costumo usar para
gerar graficos nao consegue lidar automaticamente com a funcao
definida via a integral (ele se confunde e acha que ha variaveis
demais na expressao), mas conseguiria se eu arrumasse a tal formula
fechadinha (bom, ele conhece erf(x), mas nao conhece aquela integral
do volume do tanque). Talvez o Matlab consiga lidar com as duas formas
igualmente? Nao sei...

Abraco,
Ralph

Jos Paulo Carneiro wrote:
 
 Proponho que se rediscuta o conceito de integral feia. Qual eh o problema de
 calcular uma integral numericamente?
 (So para provocar...)
 JP




Re: Ajuda urgente: cálculo do volume de um tanque.

2001-02-23 Por tôpico Ralph Costa Teixeira


Ok entao.

A parte que esta dentro do cilindro da para calcular com uma integral
tripla (ou dupla, ou simples) que da para resolver. Se o raio do
cilindro eh *r*, comprimento eh *a* e a altura do liquido eh *h* (medida
desde o fundo do cilindro), entao:

V1 = INT(z=0 a z=h) 2a sqrt(z(2r-z))dz

Para escrever assim, como integral simples, note que a area da secao ao
cilindro horizontal aa altura h eh um retangulo de area 2a
sqrt(z(2r-z)). Integre essas areas de zero a h para obter o volume. Faca
a integral por substituicao (dah um certo trabalho, fica para o pessoal
tentar) e obtenha:

V1 = (h-r)sqrt(h(2r-h)) + (Pi - arccos((h-r)/r)).r^2

Este eh o volume soh dentro do cilindro... Escreva m = h/r (a razao
entre a altura do liquido e o raio do cilindro) se quiser:

V1 = r^2.  [Pi + (m-1)sqrt(m(2-m)) - arccos(m-1)]

Dah ateh para plotar o que estah dentro do colchete como funcao de m...

Depois eu ataco o problema da calota em si, eh BEM mais feio.

Abraco,
Ralph

Thomas de Rossi wrote:
 
 Mais informaes sobre o problema:
 
 O desenho que mais se aproxima nessas aplicaes  o seguinte:
 (r/2  RAIO DA CALOTA  +INFINITO):
   -
 / \
/   \
   | |
\   /
 \ /
  -
 
 
 Nesse caso temos:
 a = 14500mm; r = 500mm; R = 3142mm;
 em que, a = comprimento do cilindro (no considerar as calotas, e sim apenas
 o cilindro plano nos lados); r = raio do cilindro; R = raio da calota at a
 linha de centro do cilindro na horizontal, ou seja, linha de simetria que
 divide o cilindro).
 Dessa forma, realmente fica mais claro e melhor de deduzir a funo.
 
 Um abrao,
 Thomas.



Re: Ajuda urgente: cálculo do volume de um tanque.

2001-02-21 Por tôpico Ralph Costa Teixeira


Ok. Entao tem-se um cilindro de raio r e comprimento a, digamos,
centrado em (0,0,0). Em outras palavras, isto d a regio:

-a/2 = x = a/2
y^2 + z^2 = r^2

Nas pontas x=a e x=-a, voce encaixa uma calota esfrica, certo? O
problema  que o seu tanque ainda no est bem definido: esta calota
poderia ter centro em x=a/2 (e a dava um hemisfrio certinho, de raio r
tambm) ou pode ter centro bem longe do ponto (a/2,0,0) -- por exemplo,
poderia ter centro em (0,0,0) (dando uma calota que no chega a
completar meia-esfera) ou at (-1000a,0,0) com raio enorme (dando uma
calota quase plana).

Voc tem como nos dizer exatamente como a calota encaixa nas bases do
cilindro?

Abrao,
Ralph

P.S.: Vou tentar dar alguma idia das possibilidades, apesar das
limitacoes do ASCII. O raio das calotas pode ser:

Grande (prximo a +INFINITO), dando calota QUAAASE plana:

-
   / \
  |   |
  |   |
  |   |
   \ /
-

Pequeno (prximo a r/2), dando exatamente dois hemisfrios:

   _-_
 / \
   / \
   | |
   \ /
 \ _ _ /
-

Coisas entre essas duas (r/2  RAIO DA CALOTA  +INFINITO):

-
   / \
  /   \
 | |  
  \   /
   \ /
-

Qual  o raio da sua calota? :)


 Pessoal,
 
 Tudo bem? Gostaria da ajuda para resoluo de um problema prtico numa
 aplicao industrial. Como no tenho muitos conhecimentos de calculo (se
 somente por clculo for possvel). L vai: deseja-se saber o volume de um
 tanque em funo da altura interna ocupada por um lquido (v = f(h)), pois
 externamente ao tanque ser colocado uma escala em que a altura ser
 correspondida ao volume. A forma geomtrica do tanque  um cilindro deitado
 de raio 'r', comprimento 'a', logicamente a altura mxima de lquido ser h
 = 2*r; porm as bases desse cilindro no so retas mas abauladas, da forma
 de uma 'meia-esfera'. (Obs: s que esta 'meia-esfera' se juntada a outra
 'meia' do lado oposto no dar o volume de uma esfera, pois no  possivel
 completar est meia esfera j que esta interseciona a 'base' do cilindro
 antes do raio da esfera ter valor 'r', o raio do circulo).
 
 NOTA: se no foi possvel ilustrar como  a configurao do tanque com as
 esferas, calcule apenas o volume do cilindro deitado, de 'bases' retas.
 
 Grato pela ajuda, []'s.
 Thomas de Rossi.



Re: Duas Questoes interessantes !

2001-01-15 Por tôpico Ralph Costa Teixeira

Oi, Iolanda.

 2)Num campeonato com 36 clubes, cada time joga uma unica vez com
 todos os demais. A Vitoria vale 3 pontos, o empate um ponto e a
 derrota nao confere pontos. Qual a quantidade minima de pontos que
 um clube precisara fazer para ter certeza que ficara entre os 12
 primeiros ?

De fato, a gente j tinha discutido aqui na lista esse problema para
um octagonal onde apenas 4 times se classificam... Eu acho que agora
consegui generalizar a idia que a gente tinha usado antes para
qualquer n par (n  o nmero de clubes que se classificam). Mas
primeiro, deixa eu dar a idia usando o problema da Iolanda -- 36
times, 12 se classificam.

Nota: a idia principal do problema est nos itens (i) e (ii), o
resto so detalhes tcnicos necessrios para evitar surpresas.

i) Primeiro eu vou mostrar que 87 pontos no so suficientes. De
fato, imagine a seguinte situao: h dois tipos de times, sendo 13
fortes e 23 fraquinhos, e os fraquinhos sempre perdem dos fortes.
Chame os "fortes" de T1, T2, ..., T13, e imagine que:

T1 ganha de T2,T3,...,T7 mas perde de T8,T9,...,T12,T13
T2 ganha de T3,T4,...,T8 mas perde de T9,T10,...,T13,T1
T3 ganha de T4,T5,...,T9 mas perde de T10,T11,...,T1,T2
...
Ti ganha de T(i+1),...,T(i+6) mas perde de T(i+7),...,T(i+12)
...
T13 ganha de T1,T2,...,T6 mas perde de T7,T8,...,T12

onde os ndices so calculados mdulo 13. Uma maneira compacta de
escrever isto 

Ti ganha de Tj se (i-j)mod13 = 1,2,3,4,5 ou 6

 importante notar que as hipteses acima so coerentes, isto , se a
linha i diz que Ti ganha de Tj, ento a linha j diz que Tj perde de Ti
(ainda bem -- a notao compacta ajuda a ver isto).

Assim, esta  uma situao onde esses 13 times ganham 23+6=29
partidas e ainda assim um deles fica de fora das finais (no saldo de
gols ou no tapeto, sei l)... Isso prova que 29x3=87 pontos no 
suficiente para GARANTIR um lugar nas finais.

ii) Por outro lado, uma quantia de 88 ou mais pontos  suficiente. Se
voc olhar para os 13 times melhor classificados, eles esto
envolvidos em 13x23+13x12/2=13x29 jogos (a primeira parcela
corresponde aos jogos contra os outros 23 e a segunda aos jogos que
esses 13 disputam entre si). Assim, esses 13 times disputam
13x29x3=13x87 pontos NO MXIMO.  impossvel que esses 13 times tenham
88 ou mais pontos (13x8813x87) assim 88 (ou qualquer quantia
maior) garante a 12a colocao (ou melhor) e a sua presena nas
finais. De certa maneira, a situao em (i)  a situao "limite"...

iii) Vale a pena notar que, s porque 87 no  suficiente, no quer
dizer que 86 no  (apesar de a intuio querer gritar isso)! Para
provar que 86 tambm no d, comece da situao em (i) acima e
modifique-a ligeiramente colocando empates nos seguintes jogos:

T1 vs. T2, T2 vs. T3 e T3 vs. T8

O resto mantenha como antes. Assim, h 10 times fortes com os mesmos
87 pontos de antes, e esses 3 citados acima passam a 86 (cada um troca
uma vitria e uma derrota por dois empates). Um deles fica de fora...
ento 86 no  suficiente.

iv) No  difcil ver que 85 ou menos definitivamente tambm no 
suficiente (basta modificar a situao de (i) onde o seu time perdeu
pontos contra os "fracos" e acabou com n85 pontos, enquanto os
confrontos envolvendo os fortes so os mesmos... isso d para fazer
com n entre 18 e 85 mantendo a tabela de confrontos entre os fortes;
se n18, p, a no d mesmo... precisamente,  fcil modificar tambm
os jogos contra os fortes e fazer voc perder mais pontos ainda sem
piorar a vida dos "fortes").

==//==

Ufa! Isso fecha o problema. Note que o raciocnio acima pode ser
facilmente generalizado sempre que o nmero de classificados for par.
Em outras palavras, se m times disputam um campeonato, cada um jogando
com cada um apenas uma vez, e os n=2k primeiros se classificam (com
nm), ento 3(m-k)-2  o menor nmero de pontos necessrio para
GARANTIR a vaga... De fato:

-- Um grupo qualquer de n+1 times disputa
(n+1)(m-n-1)+n(n+1)/2 = (n+1)(2m-n-2)/2 partidas, ou seja,
3(n+1)(2m-n-2)/2 pontos.

-- Assim, o pior desses n+1 times ter no mximo
3(2m-n-2)/2 = 3(m-k-1) pontos; isto prova que
3(m-k-1) + 1 = 3m-3k-2  suficiente para passar  fase final.

-- PARA n PAR,  sempre possvel criar uma tabela onde n+1 times
fortes ganham de todos os m-n-1 times fracos e ganham exatamente
metade (n/2) das partidas jogadas entre si (basta repetir o processo
acima, feito para n+1=13 times). Neste caso, cada time alcana
exatamente 3m-3k-3 pontos e, portanto, isto no  suficiente -- um
deles vai ficar de fora.

iv) Enfim, a idia de botar trs dos times do grupo forte para
empatarem entre si funciona e mostra-se que 3m-3k-4 tambm no d.
Para valores ainda menores,  fcil ver que voc no garante
classificao.

S para conferir... No caso m=36 e n=12 (k=6), precisamos de 3(m-k)-2
= 3x30-2 = 88 pontos. Beleza,  isso a! Na "querida" copa JH, m=25 e
n=12 indica 

Potências de 2 começando por 1

2000-12-20 Por tôpico Ralph Costa Teixeira


Oi, Igor.

 "Mostre que, pelo menos 30% dos naturais n entre 1 e 1.000.000, o primeiro
 digito de 2^n é 1."

Em primeiro lugar, pense: quantos dígitos tem 2^(1.000.000) ?

Resposta: como log(2^1.000.000)=1.000.000 log(2)  1.000.000 (0.301)
= 301.000, concluímos que 2^1.000.000 tem mais de 301.000 algarismos
(para ser exato, tem 301.030 algarismos, já que log(2)=0.301029995...).

Por outro lado, qualquer que seja o número de algarismos N que você
escolher entre 2 e 301.000, há uma destas potências de 2 que tem N
algarismos e começa por 1 (pense bem, a primeira potência de 2 com N
algarismos SEMPRE começa por 1... caso contrário, se tal potência
começasse com 2 ou mais, sua metade também teria N algarismos e seria
uma potência de 2 menor, certo?).

Assim, há pelo menos 300.099 potências de 2 (dentre estas) que começam
com 1, isto é, 30.01% delas começam por 1 (de fato, dentre estas,
exatamente 301.029 potências de 2 começam por 1, isto é, 30.1029%).

Note que a informação necesária para resolver isso acaba sendo
simplesmente esta: log(2)  0.31 (todos os logs aqui na base 10).

Abraço,
Ralph



Re: problema de dizima

2000-12-11 Por tôpico Ralph Costa Teixeira


Olá.

Estranho, eu não achei resposta para esse problema, devo ter errado
algo. Confiram abaixo se eu errei.

Marcelo Souza wrote:
 
 Oi pessoal!
 Alguém poderia me ajudar com o problema abaixo:
 - Sabendo que letras diferentes significam algarismos diferentes. Qual o
 valor de
 ABA/CDC = 0,DEFGDEFGDEFG...
 obrigado
 abraços
 Marcelo

Experimente fazer assim:

ABA/CDC = DEFG/
(ABA)=(CDC)(DEFG)

Como 101 divide , (e 101 é primo), temos que 101 | CDC ou 101 |
DEFG.
No primeiro caso, D=0 (absurdo pois ABA/CDC  100/1000 = 0,1).
Assim, 101 | DEFG, isto é, DEFG = 101k; como 1000=DEFG=, tem-se
10=k=99. Escreva k=MN com dois algarismos e note que DEFG = MNMN,
isto é, DE=FG=MN.

Estritamente falando, então, o problema não tem resposta pois D e F
deveriam ser diferentes... 

--//--

Eu acho que o problema era ABA/CDC = 0,DEDEDEDE... Aí fica assim:

99(ABA)=(CDC)(DE)

Como D!=E, 11 não divide DE, isto é, 11|CDC e portanto 11|2C-D.

C= 0   1   2   3   4   5   6   7   8   9
D=2C%11 =  0   2   4   6   8   10  1   3   5   7
CDC=  000 121 242 363 484 5?5 616 737 858 979   
CDC/11=0   11  22  33  44  ??  56  67  78  89
(2C%11 é o resto de 2C na divisão por 11)

Se 9|DE, então D+E=9 e inserimos:
E=9-D= 9   7   5   3   1   ?   8   6   4   2
DE/9=  1   3   5   7   9   ?   2   4   6   8
ABA=   0   33 110 231 396  ?  112 268 468 712
(usando ABA = CDC/11 . DE/9)
Nenhum deles presta pois os dígitos das pontas não batem.

Caso contrário, 3|(CDC/11). Sobram apenas duas opções:
a) CDC/11 = 33: (ABA)= 11.(6E)/3
E =0   3   9
ABA = 220 231 253
Nada presta.

b) CDC/11 = 78; (ABA)= 26.(5E)/3
E =1   4   7
ABA = 442 468 494
Solução: 494/858 = 0,575757...

--//--

O problema original permitindo dígitos iguais para letras diferentes
teria também o caso D=E. Então, 9(ABA)=D(CDC), ou seja:

101 (9A-CD) = 10 (D^2 - 9B)

Assim, 101 | D^2-9B e portanto D^2=9B (já que B e D são algarismos) e
9A=CD. A tabelinha é:

D =3   6   9
B = D^2/9  1   4   9
A/C = D/9  3/9 (1/3, 2/6)  6/9 (2/3, 4/6)  9/9 (A/A)

Soluções:
111/333 = 212/636 = 313/939 = 0,
242/363 = 444/666 = 646/969 = 0,
A9A/A9A = 0,



Re: Probleminhas

2000-12-01 Por tôpico Ralph Costa Teixeira


Quase, Marcos... Sua solução é boa, mas há algo que se corrigir...

Marcos Eike wrote:
 
 Se a resposta estiver correta, podemos solucionar seu problema deste modo:
 
  * Três estudantes de Matematica, de passeio por uma cidade, notaram que o
  condutor de um automovel infringira o regulamento de transito. Nenhum dos
  estudantes lembrava do numero (de quatro algarismos obviamente) da licença
  do carro, mas, como os tres eram matematicos, cada um deles havia
 registrado
  alguma particularidade desse numero. Um deles notara que os dois primeiros
  algarismos eram iguais. O segundo percebera que tambem os dois ultimos
 eram
  iguais. E, quanto ao ultimo, garantia ele que o numero inteiro era um
  quadrado perfeito. Qual era o numero da placa ?
 
 A = aacc = 10^3a + 10^2a + 10c + c = 1100a + 11c = x^2 = 11(100a + c) =
 x^2
 
 x^2 é múltiplo de 11.

Ok até aqui. Agora está errado:
 
 a+a - (c+c) = 11t

Não, a ordem está invertida, seria a-a+c-c=0 é múltiplo de 11, e isto
não ajuda.

O que funciona é ver agora que 100a+c é múltiplo de 11 (11|x^2
implica que 11^2|x^2) e então:

11 | a0c  =  11 | a+c  = a=c=0 ou a+c=11 (já que 0=a,c=9)

No segundo caso, aacc = 11(100a+c) = 11(100a+11-a) = 121(9a+1) é um
quadrado perfeito, então 9a+1 é quadrado perfeito. Enquanto dá para
fazer mais "álgebra especulativa", a esta altura do campeonato eu iria
para a "listagem":

a   1  2  3  4  5  6  7  8  9
9a+1   10 19 28 37 46 55 64 73 82

O único quadrado destes dá a=7 e c=11-a=4.

Assim, há duas respostas:  e 7744. Sinceramente, se fosse  eu
acho que os matemáticos iam lembrar direto sem essa complicação toda,
então a resposta deve ser 7744. :)

Abraço,
Ralph



Re: Equação

2000-11-29 Por tôpico Ralph Costa Teixeira



Eduardo Favarão Botelho wrote:
 
 Olá, pessoal!
 
 
 y = 1998x/(x -1998)  =  1998 (x - 1998 + 1998)/(x-1998)  = 1998. (1
 + 1998/(x-1998) ). que é  inteiro se 1998/(x-1998) é inteiro.

O problema está aqui, 1998/(x-1998) não PRECISA ser inteiro para que y
o seja. De fato,

y = 1998 + (1998^2)/(x-1998)

é inteiro se e somente se (x-1998) divide 1998^2.

Note que y0 se e somente se x-19980 (olhe para a primeira expressão).
Assim, você quer divisores POSITIVOS de 1998^2 =
= (2^2)(3^6)(37^2)... Há 3.7.3 = 63 deles, e todos são válidos para este
problema.

Abraço,
Ralph



Re: Equação

2000-11-29 Por tôpico Ralph Costa Teixeira

Eduardo Favarão Botelho wrote:
 
 Olá Ralph!
 
 é verdade... foi um deslize.  Este problema, no entanto, me suscitou
 algumas dúvidas a respeito da existência da equação. No caso de  1/x + 1/y =
 1/1998,  x pode ser zero? Porque se for, 1/x não existe. Como fica, então, o
 y? Vale 1998? Veja que eu tentei excetuar o caso em que que y e x davam zero
 justamente por achar que não dava.

Oi, Eduardo.

Você está corretíssimo em dizer que x=0 ou y=0 não valem. A sua solução
original, em particular excluía corretamente o caso
x - 1998 = -1998 exatamente por isso.

No entanto, esta solução nova já AUTOMATICAMENTE elimina tais casos
pois procuramos apenas os divisores POSITIVOS de 1998^2. Tais divisores
são os valores de x-1998 Assim, o caso x=1998 (x-1998=0) não
aparecerá, nem x=0 (x-1998= -1998) (nem 0 nem -1998 estão na nossa lista
dwe divisores).

Por outro lado, x-1998=1998 não é algo que se deva excluir. Este caso
gera x=2.1998 e y=2.1998, que é uma solução válida do problema.
 
  Há 3.7.3 = 63 deles, e todos são válidos para este
 problema.
 
 Do mesmo modo, e se x = 1998? (suponho que aconteça para y =0)

Abraço,
Ralph



Re: Eureka

2000-11-15 Por tôpico Ralph Costa Teixeira


Essa eu sei responder porque eu tenho conta nessa agência... A
agência do banco do Brasil que tem todas as contas do IMPA mudou há
pouco tempo de 0598-3 para 1564-4; assim, o número correto é, agora,
1564-4 (Agência Ataulfo de Paiva, eu acho).

Abraço,
Ralph

Carlos Stein Naves de Brito wrote:
 
 Qual a verdadeira Agencia para depositar:
 1564-4 ou 0598-3?? uma ta na internet outra na eureka 6



Re: Alguém pode me ajudar?

2000-11-08 Por tôpico Ralph Costa Teixeira

 Davidson Estanislau wrote:
 
 Simplifique a expressão:
 
 (((3)^2)^0 + 1)(((3)^2)^1 + 1)(((3)^2)^2 + 1)(((3)^2)^3 +
 1)...(((3)^2)^n + 1)
 

Essa expressão nada mais é do que:

P=(3^0+1)(3^2+1)(3^4+1)(3^6+1)...(3^2n+1)

Para simplificá-la ainda mais, aqui vai uma idéia: multiplique ambos os
lados por 8=(3^2-1) e use o produto notável junto com o (3^2+1), daí
note que... e por aí vai.

Abraço,
Ralph



Re: Desculpe

2000-11-07 Por tôpico Ralph Costa Teixeira


GRR :)

Brincadeira... Para falar a verdade, eu até achei que tinha escrito
errado e já estava preparando um `erratum' (é assim mesmo o singular?
Aliás, faz sentido o singular?) quando eu vi esta última mensagem. Eu
vivo escrevendo "associativo" quando eu quero escrever "distributivo" e
vice-versa, e achei que era um desses casos... Mas relendo a minha
mensagem vi que evitei os termos escrevendo direto a propriedade
(a+b)xc= axc + bxc... He-he... :)

Abraço,
Ralph

 Jorge Peixoto Morais wrote:
 
Eu interpretei errado a mensagem do Raplph. Ele não disse em
 momento algum que o produto era associativo. Tomara que ele não tenha
 ficado muito bravo...



Re: mais-probabilidade

2000-10-26 Por tôpico Ralph Costa Teixeira


Oi, Biscoito.

Sim, seu raciocínio também está perfeito... Por isso, encontramos o
mesmo resultado: 1/10! Note que o ! é o símbolo de fatorial:

n! = n(n-1)(n-2)(n-3)...1

então 1/10! = 1/(10x9x8x7x6x5x4x3x2), como você encontrou.

Abraço,
Ralph

Biscoito wrote:
 
 Foi mal, mas creio que vc esteja um tanto equivocado. Como há dez
 cartões todos com a mesma probabilidade de ser retirado, então vc deve
 tirar um a um com a probabilidade da proporção do número de cartões na
 caixa no momento. Traduzindo, vc tem 1/10 de tirar o T no começo,
 depois 1/9 de tirar o R, depois 1/8 de tirar o I, depois 1/7 de tirar
 o A, depois 1/6 de tirar o ^, depois 1/5 de tirar o N, depois 1/4 de
 tirar o G, depois 1/3 de tirar o U, depois 1/2 de tirar o L e depois
 só sobra o O na caixa. Assim, a sua probabilidade é de 1/10!, ou seja
 1/10x9x8x7x6x5x4x3x2, ou melhor, uma em cada 3.628.800 séries de
 tentativas de retiradas.
 
 Victor.
 
   Ralph Costa Teixeira [EMAIL PROTECTED] wrote:
 
  Bom, são 10 cartões, assim há 10! maneiras igualmente
  prováveis de
  retirá-los (é verdade que há apenas 9.9! palavras possíveis;
  é verdade
  que ^AGILNORTU dá a mesma palavra que A^GILNORTU; no
  entanto, nada disso
  me interessa no momento).
 
  Há apenas uma maneira de sair TRIÂNGULO, a saber,
  T-R-I-A-^-N-G-U-L-O,
  nesta ordem. Assim, a probabilidade disto acontecer é 1/10!.
 
  Abraço,
  Ralph



Re: Vetores no espaço (talvez eu devesse comprar um bom livro; mas qual?)

2000-10-26 Por tôpico Ralph Costa Teixeira



 Jorge Peixoto Morais wrote:
 
 Antes de tudo: valeu, Ralph, pela atencao aa minha pergunta; seu
 e-mail foi extremamente instrutivo. Agora o principal: seu último
 e-mail me deixou com umas duvidas (se achar inconveniente me
 responder, me indique um bom livro):
 a) pelas regras que voce definiu, parece que mesmo atuando soh nos
 vetores em que z=0 (ou seja, no plano xy) as regras sao totalmente
 diferentes das que regem o plano dos complexos! Por que?

Bom, sim, esse produto cartesiano a esse produto escalar realmente não
batem com o produto de números complexos quando z=0... Por quê? Bom,
para dizer a verdade, não esperaria que fossem o mesmo, de fato... 

 a2) Vendo que essas regras sao diferentes das que regem o plano de
 Gauss, me pergunto: de onde, entao, elas vem?
 b)"ixj=-j. Mas isso nao eh perpendicular ao plano determinado por i e
 j!

Oops... Se eu digitei isto, eu errei. Era pra ser ixj=k e ixk=-j.

De onde elas vem... Bom, eu não sei historicamente onde que elas
surgiram... Mas eu costumo pensar assim: quando eu tento arrumar a
fórmula para o ângulo entre dois vetores, a conta u1v1+u2v2+u3v3
aparace; quando eu tento achar a projecao de u na direcao de v, a conta
acima tambem aparece; depois de achar um monte de lugares onde ela
aparece, eu resolvi dar um nome para ela para facilitar a minha vida: o
PRODUTO ESCALAR. Imagino algo semelhante para o produto cartesiano...
mas o fato é que a necessidade do conceito só parece intuitiva para
alguém *DEPOIS* que o conceito é bastante usado... Se alguém souber
melhor, favor me ajudar aqui. :)

Na minha cabeça, produto escalar é uma ferramenta para achar ângulos
entre vetores, e o produto cartesiano para achar a área de seu
paralelogramo. *Começa* assim, e depois você vai achando um monte de
outras utilidades...

Abraço,
Ralph

 Mais uma vez, obrigado pelo trabalho de me escrever e-mails tao longos
 (mas com uma enorme densidade de informacao)



Re: mais-probabilidade

2000-10-25 Por tôpico Ralph Costa Teixeira


Bom, são 10 cartões, assim há 10! maneiras igualmente prováveis de
retirá-los (é verdade que há apenas 9.9! palavras possíveis; é verdade
que ^AGILNORTU dá a mesma palavra que A^GILNORTU; no entanto, nada disso
me interessa no momento).

Há apenas uma maneira de sair TRIÂNGULO, a saber, T-R-I-A-^-N-G-U-L-O,
nesta ordem. Assim, a probabilidade disto acontecer é 1/10!.

Abraço,
Ralph

 Filho wrote:
 
 Numa caixa são colocados 10 cartões com as letras A,G,I,L,N,O,R,T,U e
 com o acento
 circunflexo ( ^ ). Uma pessoa vai tirando cartão por cartão. Quando
 sai o acento circunflexo
 , ela o coloca sobre a última letra até então retirada. Se o
 circunflexo for o primeiro cartão,
 ela o coloca sobre a primeira letra que tirar em seguida. Qual a
 probabilidade de essa pessoa
 montar a palavra  TRIÂNGULO?



Re: Uma questão de Geometria Espacial

2000-10-24 Por tôpico Ralph Costa Teixeira


Hmmm... É isso, mas falta multiplicar por um fator... e botar um
módulo acho que é 1/6 vezes isso... Vejamos do jeito que eu sei
fazer:

Considere os vetores ei=(Xi-X4;Yi-Y4;Zi-Z4) para i= 1,2,3. O que eu
lembro é que o volume gerado pelo paralelepípedo com lados e1, e2 e e3 é
dado pelo módulo do produto misto:

e1.(e2 x e3) = [e1,e2,e3]

que, algebricamente, dá o determinante de

X1-X4X2-X4X3-X4
Y1-Y4Y2-Y4Y3-Y4
Z1-Z4Z2-Z4Z3-Z4

O tetraedro tem metade da (área da) base e a mesma altura do
paralelepípedo... e o volume do tetraedro é 1/3 base.altura enquanto o
paralelepípedo é base.altura... Então é isso mesmo, falta um fator 1/6
no determinante.

Agora, é o mesmo determinante? Ah, sim, basta começar com o seu
determinante, fazer cada linha menos a última, a última coluna vira
0;0;0;1 e o que sobra é o meu determinante. Perfeito!

Se eu lembro direito, isto funciona em dimensão N e o fator era...
hmmm... acho que era 1/N!...

Abraço,
Ralph

P.S.: Se você quiser manter o sinal, sem tomar o módulo, você ganha
alguma informação sobre a orientação relativa dos vetores, além do
volume.

 Jorge Peixoto Morais wrote:
 
 Por acaso o volume da pirâmide com vértics em (X1;Y1;Z1)... (X4;Y4;Z4)
 é |X1;Y1;Z1;1|
|X2;Y2;Z2;1|
|X3;Y3;Z3;1|
|X4;Y4;Z4;1|
 ?



Re: Sobre as funções trigonométricas de números complexos

2000-10-17 Por tôpico Ralph Costa Teixeira



 Jorge Peixoto Morais wrote:
 
 Eu estava procurando uma maneira de definir  funcoes trigonometricas
 de numeros complexos, e lembrei a formula de Euler (exp(ix)= cos(x) +
 i*sen(x)). Sera que entao
 cos(i) + i*sen(i)= exp(i*i)=exp(-1)= 1/e ? Eu ficaria feliz so por
 conseguir a resposta dessa ultima pergunta. Mas, se tambem der para
 explicar mais sobre seno e cosseno de numeros complexos, melhor.
 
 PS: A*B denota "A vezes B".

Sim, é comum definir

cos(z) = (e^(iz) + e^(-iz))/2
sin(z) = (e^(iz) - e^(-iz))/2i

para números complexos quaisquer. Não é difícil ver que o cos e o sin
de números reais é igualzinho aos já conhece (bom, pelo menos se você
partir da fórmula do e^(ix) que você mencionou)...
Note que algumas propriedades comum entre os reais se mantêm
verdadeiras com essa definição... Tente ver que cos(z)=cos(-z),
sin(z)=-sin(-z) e que (cosz)^2+(sinz)^2=1; também mostre que
cos(z+2Pi)=cos(z); sin(z+2Pi)=sin(z); cos(Pi/2-z)=sin(z); etc etc.
Note que NÃO vale que |cosz|1 nem que |sinz|1 em geral. Para o seu
caso, z=i, temos:

cos(i) = (e^(-1)+e^(1)) / 2 = (e+1/e)/2

Em geral,
cos(bi) = (e^(-b)+e^b)/2 = cosh(b)
sin(bi) = (e^(-b)-e^b)/2i = i sinh(b)

E assim
cos(a+bi) = cos(a)cos(bi)-sin(a)sin(bi) =
= cos(a)cosh(b)-sin(a)sinh(b) i

podia ser a definição a partir de z=a+bi com a e b reais (dá no mesmo).

Abraço,
Ralph



Re: Dúvida cruel...

2000-07-13 Por tôpico Ralph Costa Teixeira


Oi, Rodrigo.

Para n=1, tem-se (2n)!/(n-1)!=2=(n+1)!, então qualquer primo p2
satisfaz a equação pedida.

Mas é verdade que tal primo não existe para n1... Se existisse,
teríamos:

(2n)!/(n-1)!-(n+1)!=kp
(2n)!/((n-1)!(n+1)!) - 1 = kp/(n+1)!

Note que o lado esquerdo é um inteiro (a fração é um número binomial,
combinação 2n tomados n-1 de cada vez). Assim, o lado direito também o
é. Como p(n+1)! e p é primo, concluímos que p e (n+1)! são primos entre
si, assim (n+1)! divide k, digamos, k=a(n+1)!

(2n)!/((n-1)!(n+1)!) - 1 = ap

Mas o lado esquerdo é positivo (para n=2) e menor que (n+1)! (por
quê?); como p(n+1)!, temos uma contradição.

Abraço,
Ralph

 Rodrigo Villard Milet wrote:
 
 Será que alguém podia me ajudar nesse problema ???
 Verificar se existe primo p tal que p(n+1)! e   (2n)!/(n-1)! = (n+1)!
 mod p
 *a igualdade deve ser lida como congruência...(é claro !)
 ¡ Villard !



Re: ajuda

2000-07-13 Por tôpico Ralph Costa Teixeira


Ok, Eduardo, vou reescrever a sua idéia de uma maneira mais... hmmm...
finita, digamos assim.

Suponha que (36x+y)(36y+x) é potência de 2. O pessoal já notou que
então 36x+y e 36y+x são potências de 2.

Todo número natural positivo n pode ser escrito (de maneira única) na
forma (2^a)b onde a é natural e b é natural ímpar (nem que seja b=1;
aliás, n é potência de 2 se e somente se b=1). Assim, escreva x=(2^a)b e
y=(2^c)d com b e d ímpares.

Suponha c=a; então
36y+x = 36(2^c)d+(2^a)b = (2^a) (36.2^(c-a)d+b)
é uma potência de 2 vezes um número *ímpar* maior do que 1, isto é,
36y+x não é potência de 2. Assim, precisamos tomar ca.

Por outro lado, similarmente, para ter 36x+y como potência de 2,
precisamos de ca. Contradição!

Assim, não existem x e y satisfazendo a condição "(36x+y)(36y+x) é
potência de 2".

Legal?

Abraço,
Ralph

Eduardo Grasser wrote:
 
 vejamos... ser potência de 2 significa ser da forma 2^n, né? Logo o número
 não pode ser divisível por um ímpar maior que 2.
 Bem, se x é impar, 36y + x também o é, além de ser maior que 36 (x,y ou =
 1). Se y é ímpar, 36x + y também o é.
 logo x é da forma 2j e y=2k. Ok?
 assim, podemos escrever o número como:
 (36*2j+2k)*(36*2k+2j)= 2(36j+k)*(36k+j)
 Bem, caímos no caso anterior. j e k precisam ser pares.
 Podemos repetir esse passo várias vezes, mas sempre teremos que o tal número
 é divisível por um impar maior que 36.
 
 não está bem escrito, mas a idéia é boa. Se alguem quiser reescrever, não
 ficarei chateado
 
 abraços
 
 Eduardo Grasser



Re: sem cálculo

2000-07-01 Por tôpico Ralph Costa Teixeira


Hmmm sem cálculo fica mais feio... mas dá para fazer...

Tome f(x)=x^3+2x+k. Eu imagino que no segundo grau ainda dê para usar
o Teorema de Bolzano: como f é contínua (polinomial), f(-1)=k-30 e
f(1)=k+30, há pelo menos uma raiz em (-1,1).

Mas se houver duas raízes distintas entre -1 e 1, digamos, a e b,
então

a^3+2a+k=b^3+3b+k=0
a^3-b^3+2a-2b=0
(a-b)(a^2+ab+b^2+2)=0

Como supus raízes distintas, podemos cortar (a-b):

(*) a^2+ab+b^2+2=0
Mas como |a|,|b|1, temos |ab|1, e portanto
a^2+ab+b^2+2 = ab+2  -1+2 = 1
E temos uma contradição.

(Ou: olhando (*) como uma equação quadrática em a,
delta=(b^2-4b^2-8)0 e não há raízes; de fato, isto prova que
f(a)=f(b) implica a=b não só para raízes em (-1,1), mas para quaisquer
a e b na reta real; assim, isto prova que f é estritamente crescente)

Abraço,
Ralph

 Filho wrote:
 
 Caro Wellington no final do seu comentário, você usou recursos de
 cálculo. A questão foi de um vestibular que no programa não consta
 nada de cálculo.
 Grato pelo primeiro comentário, mas o que torna a questão diferente
 é exatamente não poder usar tais recursos. O problema
 continua
 
 Mostre que a equação x^3 + 2x +k=0, com k real no intervalo aberto
 ]-3,3[, possui exatamente uma raiz no intervalo aberto ]-1,1[.
 
 Seja f(x)=x^3+2x+k;
 Primeiramente substituiremos x nos valores extremos do intervalo:
 para x=-1 a imagem da funcao estara em ]-6,0[;
 para x=1 a imagem da funcao estara em ]0,6[;
 ou seja, independente do valor de k dentro do intervalo em questao
 ( ]-3,3[ ), a funcao retornara valores com sinais opostos. Isso
 garante a existencia de um numero impar de raízes nesse intervalo
 (Teorema de Bolzano).



Re: soluções inteiras

2000-06-16 Por tôpico Ralph Costa Teixeira


Carlos: você também está com o vírus Happy95 que o pessoal falou há
pouco na lista. Limpe seu computador...

Quanto à questão:

(x+1)(y+2)=2xy
xy+y+2x+2=2xy (abrindo tudo)
xy-2x-y-2=0 (tudo dum lado só)
(x-1)(y-2)=4 (fatore adicionando ou tirando um termo constante)

Como x e y sao inteiros, há poucas possibilidades:

x-1 = -4,-2,-1,1,2,4 (e y-2 = -1,-2,-4,4,2,1, respectivamente)

Entao:

(x,y) \in {(3,1),(-1,0),(0,-2),(2,6),(3,4),(5,3)}

Essas sao as inteiras; as positivas sao os quatro pares que nao incluem
o zero.

Abraço,
Ralph

Carlos Gomes wrote:
 
Alô pessoal como estão todos, tudo ok?
 
Gostaria que alguém me ajudasse com s seguinte questão que
 está no livro teoria elementar dos números do Edgar de Alencar, logo no
 primeiro capítulo:
 
  Achar todas as soluções inteiras e positivas da equação
 (x+1)(y+2)=2.x.y
 
 Um abraço a todos,
 
 Carlos A. Gomes
   15.06.00



Re: como achar?

2000-06-03 Por tôpico Ralph Costa Teixeira


Hmmm... Para explicar o valor 2... Eu já vi a seguinte pergunta:

"Que número natural nos dá o máximo de n^(1/n)?"

Neste caso, a resposta é 3 (ok, não é bem *2*), e o problema pode
ser feito de maneira "elementar" ("elementar" quer dizer sem o uso de
cálculo -- neste sentido, muitas das coisas "elementares" são bem
difíceis).

Como proposto, dentro dos reais, o pessoal da lista já matou o
problema: a resposta é de fato x=e; a princípio, não sei fazer esta
sem cálculo...

Abraço,
Ralph   

Benjamin Hinrichs wrote:
 
 Pessoal,
 perguntinha para vós: como se acha o valor máximo de x^(1/x). Como se chega
 ao valor máximo da 'função'? Tentei fazer a derivada mas não concluí nada,
 acho que fiz errado. Num livro eu acho ter lido que o valor mais alto é
 e^(1/e) mas já vi que com 2 o valor obtido é mais alto, acho. Um grande
 abraço, Benjamin Hinrichs



Re: Octógono

2000-05-21 Por tôpico Ralph Costa Teixeira


Oi, Marcelo.

Minha dica (complete os pontinhos):

Os triângulos APE, BPF, CPG e DPH têm algo especial, não? Afinal, AE,
BF, CG e DH são diâmetros, então

...e portanto PA^2+PD^2 = ... e PB^2+PF^2=...
e...   e...

Voilá! :)

Abraço,
Ralph


Marcelo Souza wrote:
 
 E aê, galera!
 
  Será que alguém pode resolver este problema para mim?
 1. Um octógono regular ABCDEFGH está inscrito num círculo de raio 1. P é um
 ponto arbitrário no menor arco FG. Calcule o valor de:
 PA^2 + PB^2 + PC^2 ++ PH^2
 Obrigado
 Abraços
 Marcelo



Re: Me ajudem!

2000-05-16 Por tôpico Ralph Costa Teixeira


Oi, Alexandre.

Este problema é *muito* complicado. Eu concordo que se a soma dos
algarismos de A=2^a é a mesma de B=2^b, com AB então 6 | (a-b). Mas é
só isso que eu vejo. Pode ser que a=b+6, pode ser a=b+12... Apesar de eu
achar "provável" que a=b+6 (e portanto A=4096B), vejamos (bendito
Excel!):

Diferença de potências 12:
2^32=4294967296 (soma 58 - média dos dígitos 5.80)
2^44=17592186044416 (soma 58 - média dos dígitos 4.14)

Pior, chega a 24 com:
2^46=70368744177664 (soma 70 - média digital 5.00)
2^58=288230376151711744 (soma 70 - média digital 3.89)
2^70=1180591620717411303424 (soma 70 - média digital 3.18)
e até mesmo a hipótese de que sejam no máximo 2 números falha.

Usando *só* as 120 primeiras potências, eu achei também:
2^67  =147573952589676412928 (soma 110 - média 5.24)
2^73  =   9444732965739290427392 (soma 110)
2^79  = 604462909807314587353088 (soma 110)
2^103 = 10141204801825835211973625643008 (soma 110 - média 3.44)
(diferença em potências 36 da primeira a última! São ONZE algarismos a
mais)

e algumas outras triplas, com soma de algarismos 107, 112, 118, 125.
Calcule mais potências e eu aposto que algumas dessas triplas viram
quadras (como a do 110), e não duvido nada que mais alguém dê 110... etc
etc etc

Eu apostaria que, dado n positivo qualquer, *existem* n potências de 2
com a mesma soma de algarismos... Mas isso deve ser *MUUITO* difícil
de provar. Até n=4 está feito acima... :)

Como exercício, de novo, adivinhe como eu fiz as contas acima em
Excel... :)

Abraços,
Ralph

Alexandre Gomes wrote:
 
Caros colegas
Há alguns dias enviei um probleminha que tinha elaborado há um tempo.
 Como não consegui uma boa idéia para resolvê-lo, ou melhor, não consegui
 resolvê-lo, decidi enviá-lo para a lista. Mesmo sendo uma questão sem
 atrativos, continuei trabalhando nela, quando o tempo me permitia e esperei
 alguma solução ou sugestão, que não apareceu, não sei se é porque o problema
 é realmente difícil ou se é muito idiota, a ponto de se recusarem a
 respondê-lo. Mando novamente o problema e uma observação que fiz enquanto
 tentava solucioná-lo. Se algum dos colegas conseguirem solucioná-lo ou
 tiverem alguma sugestão, por favor, respondam.
   Dado o conjunto A de todas as potências inteiras de 2, escolhe-se um
 elemento ao acaso, cuja soma dos algarismos vale x. Diga, com prova, o
 número máximo de elementos de A que podem ser escolhidos tais que a soma dos
 algarismos de cada um deles também seja igual a x.
OBSERVAÇÃO FEITA(não sei se é a melhor):
Podemos escolher 2 números a e b de A, com ab, com a seguinte
 propriedade: Se a soma dos algarismos de cada um dos números a e b é x,
 então b=64a. Notar que a soma dos algarismos de c=64b é maior do que x. Se
 construirmos uma demonstração formal para isso, parte da solução está
 encaminhada, faltando apenas demonstrar que não existe um outro elemento de
 A, além de a e b, cuja soma dos algarismos também seja x. Desta forma,
 concluitremos que o número máximo de elementos de A que tenham soma dos
 algarismos iguais a x é 2.
Mais uma vez digo que não sei se esta é a melhor saída. Quem me ajuda a
 dar uma demonstração formal à minha observação ou tem uma saída melhor?
Conto com vocês.
Um abraço
Alexandre S. Gomes.



Re: Polinomio

2000-05-12 Por tôpico Ralph Costa Teixeira


Oi, Jackson.

Eu não sei quanto você sabe de polinômios... Se eu fosse resolver esse
problema, eu tentaria assim:

Suponha que o polinômio indicado tenha 6 raízes reais, digamos, A, B,
C, D, E, F (algumas podem ser repetidas).

- Se você souber as relações entre raízes e coeficientes:
Então A+B+C+D+E+F=0
e AB+AC+AD+AE+AF+BC+...+EF=0

- Se você não souber a tal relação, pense assim: o polinômio será
fatorável como (x-A)(x-B)(x-C)(x-D)(x-E)(x-F). Olhe para os coeficinetes
em x^5 e x^4 e você descobre o que eu falei acima.

Eu espero que um desses dois sirva para você... De um jeito ou de
outro, a soma das raízes e a soma dos seus "duplos produtos" ambas são
zero. Tente chegar a uma contradição a partir daí.

Quando você terminar, note que a solução é bem generalizável: um
polinômio da forma x^n+ax^(n-3)+(termos de menor grau) não pode ter n
raízes reais (a menos que sejam todas nulas).

Abraço,
Ralph

P.S.: Dica (não leia se você quiser tentar sozinho):






Eu avisei...






Olhe para (A+B+C+D+E+F)^2...





Jackson Graziano wrote:
 
 Caros Amigos,
 
 Faz algum tempo que entrei na lista, mas até agora só acompanhei as
 discussoes.
 Sou estudante da 3a serie do Ensino Medio e recebi a lista de treinamento da
 Olimpiada da Unicamp. Como ainda nao tive polinomios, ha um problema que nao
 consegui resolver. Segue o enunciado do mesmo:
 
 Sejam a, b, c e d numeros reais nao todos nulos. Mostre que as raizes do
 polinomio p(x) = x^6 + ax^3 + bx^2 + cx + d nao podem ser todas reais.
 
 Como posso resolver esse problema sem conhecer equacoes polinomiais?? Ha
 alguma maneira?
 
 Jackson Graziano
 [EMAIL PROTECTED]



Polinômios e primos

2000-05-12 Por tôpico Ralph Costa Teixeira


Essa pergunta é muito legal, Eduardo, e você tem razão -- tal polinômio
não existe.

No entanto, sua demonstração resolve só 80% do problema; afinal, e se o
termo independente (digamos, a) for 1, -1 ou um primo p? Pode ser que o
valor de P(a) seja 1, -1 ou p e isto *pode* ser primo (apesar de
divísivel pelo termo independente).

Mas se você continuar o raciocínio, dá para consertá-lo na boa (só
falta 20%). Vou deixar vocês tentarem.

Abraço,
Ralph

Eduardo Grasser wrote:
 
 Pergunta: quem disse que 6k + 5 só nos fornece primos???
 k=5 = 6k + 5 = 35
 o que me leva a perguntar: existe polinômio que só nos forneça primos?
 Não! não existe... basta fazer a incógnita valer o valor do termo
 independente e o polinômio será divisível por este...
 
 Eduardo Grasser
 Campinas SP
 ICQ - 54208637



Re: O dia que nao acaba

2000-04-18 Por tôpico Ralph Costa Teixeira

 Elon Santos Corrêa wrote:
 
 Tem que haver um "salto" !
 
 Caros amigos,
 
 quanto a questao: 1 = 0,999... , gostaria de suscita-la.
 
 Por exemplo, hoje e dia 18 de abril ate as 23 horas 59 minutos 59
 segundos e 0,999... de um segundo. Se nao houver um salto, (ponto de
 parada) quando comecara o dia 19, ou ainda, se a aproximacao nao parar
 quando acabara o dia 18 ?
 
 Aproveito para perguntar a opiniao de todos, 1 = 0,999... ? (Igual,
 nao que representa)
 
 Ate mais, Elon.

Para mim, 0,9 = 1 perfeitamente.

Tambem, 18/4, 23h 59m 59,9...s = 18/4, 24h 0m 0s = 19/4, 0h 0m 0s.
O salto que voce estah procurando eh o tal momento 18/4 24h = 19/4 0h. A
que dia este momento pertence, isto eh outra estoria.
Se o dia eh o intervalo [0h, 24h) -- fechado em 0h, aberto em 24h --
entao digo categoricamente que 18/4, 23h 59m 59,...s pertence ao dia
19. (Por que a surpresa? Com esta definicao de dia, 18/4 24h 0m 0s
tambem eh dia 19!)
Se o dia for [0h, 24h] -- fechado dos dois lados -- entao tal momento
pertence a ambos os dias 18 e 19. (Por que a surpresa? Quem disse que
dias tem de ser disjuntos? se voce quiser dias disjuntos, volte para a
outra definicao!)

A gente jah discutiu um bocado o porque disso ser verdadeiro
matematicamente. Eu nao lembro se jah disse... sabe *por que* as pessoas
nao gostam de aceitar isso, e sempre gera polemica? Eu vou exagerar para
explicar melhor:

SIR KASMO FALA
O problema vem dessa noção absolutamente absurda de que, quando
comparando dois numeros, o que tem o primeiro digito menor eh o menor
numero! Absurdo! Heresia! Essas pessoas que acreditam nisto dizem que
para numeros naturais (um ? para cada digito)
1  2  pois 12
9??? = 09???  1   pois 01
sem nem mesmo ver o que sao os digitos escondidos! Como podem faze-lo?
O horror! O caos!

Essa mesma gente olha para numeros reais e faz coisas como
3,...  6,... pois 36
sem olhar uma INFINIDADE de digitos escondidos! Oh, a dor! A angustia
Vulcaniana!

E eles tambem dizem que
3,...  4,... pois 34
e nem sabem de que numeros estao falando! Daonde veio essa ideia
erronea, ilogica e iconoclasta? Revolucao! Temos de destruir o sistema!
SIR KASMO FALOU

Pois é, "essas pessoas" fazem o raciocinio que SIR KASMO estah
criticando o tempo todo, e parece facil, natural, absolutamente obvio e
correto. Sabe por que parece facil, natural, obvio e correto? Por que eh
facil, natural e estah correto em *QUASE* todas as ocasioes! *QUASE*
sempre, basta comparar o primeiro digito para ver que numero eh menor
(supondo digitos de mesma ordem), e se empatar passar para o proximo,
etc etc. Isto funciona *TAO* frequentemente que todo mundo usa este
metodo sem nem pensar, inclusive eu. Para piorar o problema, a gente
aprende desde pequeno a ordenar as coisa em ordem alfabetica, e o metodo
utilizado eh exatamente esse: compare a primeira letra, se empatar
compare a segunda, etc... Ninguem nunca errou na hora de ordenar nomes
em ordem alfabetica porque no caso de palavras (que sao todas finitas) o
metodo ACERTA.

Mas SIR KASMO tem razao, apesar de seu argumento ser um pouco extasiado
demais. O algoritmo que essas pessoas fazem funciona *QUASE* sempre com
numeros. Eu disse *QUASE*! Alias, diga-se de passagem, tal algoritmo SOH
FALHA QUANDO UMA DIZIMA DO TIPO 999... APARECE! SIR KASMO exagerou,
já que o algoritmo que "essas pessoas" usam soh fura na ultima
desigualdade, e somente no caso raro em que os digitos escondidos forem
99... do lado esquerdo e 0... do lado direito... mas o algoritmo
*PODE* falhar.

Esse eh o pior tipo de contra-exemplo, o contra-exemplo solitario.
"Essas pessoas", ao inves de questionarem o metodo que usavam para
comparar numeros, passam a questionar o contra-exemplo. "Poxa, se soh
tem esse exemplo onde meu metodo estah errado, o *EXEMPLO* deve estar
errado, nao?", elas dizem. E eh dificil demove-las desta ideia, baseada
em senso comum.

Eh como voce falar para uma crianca que 3, 5, 7, 11, 13, 17, 19, 23,
..., 199102912801278801737156361623 sao primos. Depois essa crianca
aprende o que sao numeros pares e impares, e ela nota que *TODOS* os
numeros primos que ela conhece sao impares. Ela vive com esse fato por
30 anos; sempre que ela se depara com um numero par, ela diz "nao eh
primo!" e, estando correta, ganha doces e beijos (por sorte, o numero 2
*nunca* apareceu nesse contexto). Por outro lado, ela vive encontrando
numeros primos em sua vida, e todos sao impares (o 2 tambem nunca
apareceu nesse contexto). Ela viu tantos deles, decorou uma tabela tao
enorme de tais numeros, que ela jura saber exatamente o que sao os tais
numeros primos. Entao, um dia, voce fala pra ela: "2 eh primo". Ela nao
vai aceitar isso de jeito nenhum! "Nao eh possivel!", ela dirah. "Poxa,
2 nao estava na minha tabela..." e entao ela 

Re: d = a^1999 + b^1999 + c^1999

2000-04-03 Por tôpico Ralph Costa Teixeira


Ok. Minha idéia:

 Triplas de inteiros a,b,c com a+b+c=0 são consideradas. Seja
 d= a^1999 + b^1999 + c^1999.

 a) Podemos ter d = 2 ?
 b) Podemos ter d primo ?

Se a=0, então b=-c e d=0. Assim, podemos desconsiderar o caso de
qualquer um deles ser nulo (a,b ou c). Similarmente, podemos
desconsiderar o caso de dois deles serem simétricos.

Se você souber congruências, olhe tudo módulo a; tem-se
d=b^1999+c^1999=0 (pois b=-c modulo a). Assim, a|d.
Senão, note que b^1999+c^1999 =
=(b+c)(b^1998 - b^1997 c + b^1996 c^2 -... + c^1998)
Assim, d=a^1999-aK é divisível por a.

Similarmente, b|d e c|d. O único jeito de tudo isso acontecer e d ser
primo é se {a,b,c} estiverem em {+-1,+-d}. Como a+b+c=0, para que d
seja primo só resta {a,b,c}={-1,-1,d} com d=2. É fácil ver que esta
opção também não serve, pois

2 != 2^1999 - 1 - 1

então d não pode ser primo.



Poligonos regulares de angulo inteiro

2000-03-31 Por tôpico Ralph Costa Teixeira


Olá, Via.

 Via Lux wrote:
 
 1)Quantos sao os poligonos regulares tais que seus angulos internos

(em graus)

 sao inteiros?
 Assumi que um angulo interno de um poligono regular eh do
 tipo:
 ((n-2)180)/n, a partir dai, peguei todos os divisores
 naturais de 180, com exceção de 1 e 2, e a eles juntei alguns
 multiplos de 8 que tambem geram poligonos regulares, tais como o
 próprio 8, 24,40,72, etc. Detalhe: alguns desses multiplos nao geram
 os poligonos desejados. Obtive-os todos por tentativas...

O angulo interno de um poligono regular eh

180(n-2)/n = 180 - 360/n

Isto eh inteiro se e somente se n eh divisor de *360* (por isso que
voce nao achou todos). Como 360 = 2^3 3^2 5, há (3+1)(2+1)(1+1)=24
divisores. Dois deles nao prestam (1 e 2), entao ha 22 poligonos
regulares satisfazendo a tal condicao...



Re: TRADUZINDO RALPH

2000-03-22 Por tôpico Ralph Costa Teixeira

Não, é o contrário.

O problema que eu propus e que o Morgado completou mostra que há a e b
irracionais tais que a^b é racional, mas a gente fica sem saber
exatamente que a e b são esses. A gente mostrou que ou a=raiz(2) e
b=raiz(2) serve OU a=raiz(2)^raiz(2) e b=raiz(2) serve (ou ambos). 

A afirmação do José é que a=b=raiz(2) *não* serve para o *meu* problema
(e tem como consequencia o fato de que a=raiz(2)^raiz(2) e b=raiz(2)
serve). A gente não sabe disso, a menos que use proposições mais
difíceis (como a que o Nicolau mencionou).

Abraço,
Ralph

P.S.: Espero que os caracteres estranhos que eu escrevi não tenham
gerado estranho Control Codes no E-mail da galera... Senão... Vocês
leram as letras que estavam dentro dos caracteres ilegíveis da minha
mensagem, né? 

Marcos Eike Tinen dos Santos wrote:
 
 Moragado, acho que não pesquei a idéia, mas neste caso vc está demostrando
 para qualquer n-ésima raiz, n=/2
 Pois, senão o problema do José já estaria resolvido.
 Pois o problema do José, é raiz quadrada de 2.
 "raiz quadrada de 2 elevado a raiz quadrada de 2 é irracional"
 
 Atenciosamente,
 Marcos Eike
 - Original Message -
 From: Augusto Morgado [EMAIL PROTECTED]
 To: [EMAIL PROTECTED]
 Sent: Quarta-feira, 22 de Março de 2000 08:38
 Subject: TRADUZINDO RALPH
 
  Vou tomar a ousadia de traduzir o que o Ralph escreveu e que esses
  mistérios computacionais verteram para tão estranho idioma.
  Um problema interessante é o seguinte:"Existe um número irracional que
  elevado a um irracional dê como resposta um racional?"
  A resposta é sim.
  Tomemos o número x = (raiz de 2) elevado a (raiz de 2). Se x for
  racional, está provado que a resposta é sim. Se x for irracional então x
  elevado a raiz de 2 será igual a (raiz de 2) elevado a 2, isto é, 2
  e está provado que a resposta é sim.
  Morgado



Re: interessante

2000-03-21 Por tôpico Ralph Costa Teixeira

 On Tue, 21 Mar 2000, José Fabrício Maia wrote:
 
  Colegas gostaria de saber como se mostra que raiz quadrada de 2 elevado a raiz 
quadrada de 2 é irracional.

Aqui vai um problema relacionado que não é tão difícil (talvez até seja
o problema que inspirou a questão do José, não sei...), a ser resolvido
sem uso de nenhum teorema pesado:

Problema:
Prove que existem números irracionais a e b tais que a^b é racional.

Solução curta (a ser completada):
"Se a afirmativa do José Maia não for verdadeira $^(#^$%%CARRIER OUT
e está provado.

Por outro lado, se a afirmativa do José Maia for verdadeira...
%$%Q%#%U$E A$Z$A^$R D%E# N$O###V^*$$O e então basta notar que
#J%^US%^$T(^O*^ $AG^%O#$$@R##A#? ... e acabou."

Abraço,
Ralph



Re: sistema

2000-03-21 Por tôpico Ralph Costa Teixeira

Oi, Elon.

Argh, estou sem tempo para comentar isso, então cuidado: grande
possibilidade de eu falar besteira aí embaixo! Vou "xutar" umas coisas,
por favor verifiquem.

Basicamente, você quer x^y=y^x com yx0, certo? Afinal, depois que
você encontrá-los, basta tomar a=y/x e a temos uma solução da sua
equação.

Parte deste problema já foi discutido aqui, disfarçado... Tinha um
thread sobre x^x^x^x^x..., cujo artigo mais informativo é o do Gugu:

http://www.mat.puc-rio.br/~nicolau/olimp/obm-rj.1999/msg00169.html

Siga o thread todo se você não lembra. O que o Gugu disse (e que eu não
sei se é fácil de provar) é que y=x^x^x^x... (definido de cima para
baixo, como no artigo do Gugu) existe quando e^(-e)  x  e^(1/e); neste
caso, x^y = y^x e y  x... eu acho.

Acredito que para x fora desse intervalo não há solução com y!=x...

Acho que há um artigo numa das revistas da AMS que trata disso... e
acho que há algo assim na RPM também...

Finalmente, acho que dá para mostrar a existência da solução para f(y)
= x^y-y^x = 0 para um x fixo naquele intervalo usando um pouco de
cálculo... talvez não para o intervalo do Gugu, mas para intervalos
menores.

Finalmente, a única solução inteira, acredito ser 2^4=4^2, se é isso
que você quer.

Argh, tantos achismos, mas eu tenho que ir embora para pegar minha
carona... :)

Abraço,
Ralph


 Elon Santos Corrêa wrote:
 
 Olá pessoal da lista,
 
 gostaria de ver soluções para o sistema de equações:
 
 x^y = y^x
 e
 y = ax
 
   com a  0 e diferente de 1.
 
 Obs.: o símbolo " ^ " significa elevado ao expoente ...
 
 Antecipadamente agradeço as respostas, Elon.



Re: bicondicional

2000-03-01 Por tôpico Ralph Costa Teixeira


Oi, Mira.

De fato, da maneira enunciada, o resultado eh falso, porque a
volta eh falsa. Em outras palavras, a afirmativa:

"Sejam A, B e C tres conjuntos quaisquer.
SE (A inter B eh vazio) ENTAO (A contido em C-B)"

estah incorreta. Como contra-exemplo, tome A={1}, B={2} e C={2};
note que A inter B eh vazio E AO MESMO TEMPO A *nao* estah contido em C-B.
(Provavelmente semelhante ao exemplo que voce achou)

Voce tem razao ao dizer que A contido em C-B implica que A contido
em C; no entanto, nao hah como usar isto na volta (para provar a volta,
voce soh pode usar que A inter B eh vazio, o enunciado nao oferece mais
nada).

Sempre que houver uma afirmativa "bicondicional" (do tipo SE E
SOMENTE SE, ou EH EQUIVALENTE), hah de fato duas coisas a demonstrar: a
ida e a volta. Para ser seguro, digamos que elas sao independentes
(existem casos onde a ida ajuda a provar a volta de uma maneira um tanto
estranha, mas nao pense nisso ainda).

Como modificar o enunciado para que ele se torne correto? Bom, uma
alternativa eh a mais simples, onde nao mencionamos a volta:

"Sejam A, B e C tres conjuntos quaisquer.
SE (A contido em C-B) ENTAO (A inter B eh vazio)"

Isto estah correto (e pode ser demonstrado, parece-me que voce
jah o fez).

Hah outras possibilidades. Como voce deve ter notado, as
seguintes alternativas sao validas:

"Sejam A, B e C conjuntos quaisquer tais que A contido em C.
Prove que (A eh subconjunto de C-B) SE E SOMENTE SE (A inter B eh
vazio)"

Quando algo eh escrito assim, voce pode usar que A estah contido
em C tanto na ida como na volta. Eh uma condicao do teorema todo, nao soh 
da ida ou soh da volta. Assim, o enunciado estah ok.

Outra opcao eh:

"Sejam A, B e C conjuntos quaisquer.
(A eh subconjunto de C-B)
SE E SOMENTE SE
[(A inter B eh vazio) E (A estah contido em C)]"

Escrito assim, o fato de que A estah contido em C soh pode ser
usado *A PRIORI* para a volta. Para a ida, voce tem que usar que A eh
subconjunto de C-B, dai provar que A estah contido em C, e soh entao voce
pode usar este fato.

Espero nao ter complicado as coisas demais... As demonstracoes em
si nao foram o foco do que estah acima; minha intencao era mais descrever
um pouco mais minuciosamente a ORDEM DA CADEIA DE RACIOCINIO (o que
me pareceu ser a sua pergunta).

Abraco,
Ralph

 On Wed, 1 Mar 2000, Mira wrote:

 Ola, se alguem puder me ajudar...
 
 Bom, estou tentando entender o que acontece com a frase a seguir: (Sejam A,B
 e C conjuntos)
 
 "A eh subconjunto de (C-B) se, e somente se A inter B eh vazio"
 
 Na ida deste bicondicional concluo que A eh subconjunto de C. Devo usar essa
 conclusao para demonstrar a volta?
 A volta esta de alguma forma presa a ida? Ou ela eh independente? Pois,
 poderia escolher 3 conjuntos disjuntos e a volta nao seria verdadeira!
 Preciso demonstrar o "teorema" ( se eh que ele eh!).
 
 Bom,ja deu pra perceber que me perdi!
 
 Obrigado antecipadamente!
 
 Mira
 
 
 



Re: Induçao

2000-02-28 Por tôpico Ralph Costa Teixeira

  Provar que para todo n Natural vale
  (1+1/1).(1+1/2).(1+1/3)...(1+1/n)=n+1

Resolver por inducao o Flavio jah fez, mas eu queria
rapidinho fazer uma nota. Se voce nao tiver que resolver por
inducao, escreva o lado esquerdo como um produto de fracoes:

2/1 . 3/2 . 4/3 . 5/4 ... (n+1)/n = n+1

E a igualdade sai Direto se voce cancelar o numerador
de cada fracao com o denominador da proxima. Por isso que a
demonstracao por inducao do Flavio parecia curtinha...

Abraco,
Ralph




Re: G. Polya

2000-02-14 Por tôpico Ralph Costa Teixeira


Oi, Marcelo.

Talvez eu tenha entendido errado, mas me parece impossivel. Para provar
isso, considere o resto desta soma na divisao por 9.

Lembre-se que o resto de um numero na divisao por 9 eh o mesmo resto
que a soma de seus algarismos deixa na divisao por 9... Em outras
palavras, imagine os numeros todos escritos e faca um 'noves fora'. O
que fica?

Eu nao sei a ordem em que os digitos aparecem, mas 'noves fora' fica...
bom fica exatamente 1+2+3+4+5+6+7+8+9 = 45 = 0 (noves fora). Assim, se
voce montar alguns numeros usando os digitos 0-9 uma vez cada,
exatamente, o resto na divisao por 9 da soma deles serah 0. Como 100
deixa resto 1 na divisao por 9, o problema me parece impossivel...

Por favor me esclareca se eu entendi o problema errado.

Obrigado,
Ralph

Marcelo Souza wrote:
 
 “Escrever números usando cada um dos dez algarismos uma só vez, de tal modo
 que a soma desses números seja exatamente 100.”
 
 Se alguém puder explicar o raciocínio eu agradeço antecipadamente
 Abraços
 Marcelo



Re: triangulo

2000-02-14 Por tôpico Ralph Costa Teixeira


Marcelo, aqui vao duas sugestoes distintas:

a) Sugestao trigonometrica
(Eu nao segui este caminho; se voce souber senos e cossenos de angulo
como 15, 75 e tal tenho certeza que dah certo; sem eles, ainda eh
possivel, mas mais complicado; para falar a verdade, nao fiz assim
porque acho a segunda solucao mais elegante):

i) Note que o triangulo BDC estah bem determinado; use a lei dos senos
ali para obter o comprimento de CD
ii) Agora que voce tem CD, AD e o angulo ADC (60), voce pode obter AC
pela lei dos cossenos
iii) Agora voce tem os lados de ACD e o angulo ADC; uma lei dos
cossenos pode determinar o cosseno do angulo desejado

b) Magica geometrica

i) Vamos tentar criar um bando de triangulos isosceles (isso sempre
ajuda a transportar angulos). Assim, marque Y no segmento DC tal que
DY=10; marque X no segmento AD tal que DX=XA=10.

ii) Crie os segmentos BY, XY e AY.

Se voce nao quiser mais dicas, isto eh um bom comeco. Se quiser mais
dicas, leia abaixo o segundo nivel de sugestoes:




iii) BDY eh isosceles, encontre seus angulos;
 entao BCY eh isosceles, e BY=CY
 DXY eh isosceles e mais, entao XY=10
 entao AXY eh isosceles, encontre seus angulos

 NOte agora que ABY eh isosceles, entao AY=BY

 Agora voce tem YAX do triangulo AXY... e como AY=BY=CY, ACY
eh isosceles, encontre CAY.

Abraco,
Ralph  


Marcelo Souza wrote:
 
 E aí, galera da lista,
 
 Já fiz de tudo para achar solução deste problema abaixo, mas não consegui
 nada, se alguém puder me ajudar agradeço bastante.
 É assim
 
 "Num triângulo ABC, de C traça-se a ceviana CD, com D pertencente a AB, tal
 que AD=20 e DB=10. O ângulo DCB mede 15° e o ângulo ABC mede 45°. Calcule o
 ângulo BAC."
 
 Espero resposta
 Obrigado
 Marcelo
 
 __
 Get Your Private, Free Email at http://www.hotmail.com



Re: Viciados em Math. LOOK:

2000-01-24 Por tôpico Ralph Costa Teixeira

 
 Sejam Q e Z, conjunto dos racionais estritamente positivos e o conjunto dos
 inteiros. Determine todas as funções f: Q -- Z satisfazendo as seguintes
 condições:
 
 1) f(1999) =1
 f(ab) = f(a) + f(b), para quaisquer a, b pertencente Q
 f(a + b) = min {f(a), f(b)}, para quaisquer a,b pertencente a Q

Basta defini-la nos naturais. De fato, suponha que f satisfaz estas
tres condicoes nos naturais. Entao hah uma maneira unica de
estende-la... Dado p racional, escreva p = a/b com a e b naturais primos
entre si; entao

f(a) = f(pb) = f(p) + f(b) = f(p) = f(a) - f(b) estah definida

Note que mesmo que escrevamos p como uma fracao redutivel, digamos,
p = (am)/(bm) entao f(am) - f(bm) = f(a) + f(m) - f(b) - f(m) = f(a) -
f(b).

Note que se (i), (ii) e (iii) valem nos naturais, entao valem nos
racionais... De fato, (i) eh imediato. Para (ii) e (iii), suponha que
p=a/b e q=c/d sao 2 racionais quaisquer; entao

(ii)f(pq) = f(ac/bd) = f(ac) - f(bd) = f(a/b) + f(c/d) = f(p) + f(q)

(iii)   Suponha f(p) = f(q) sem perda de generalidade para obter
f(a/b) = f(c/d); f(a)-f(b)=f(c)-f(d); f(ad) = f(bc);
f(ad+bc) = f(bc) (aqui, usando (iii) nos naturais)
e entao
f(p+q) = f((ad+bc)/bd) = f(ad+bc) - f(bd) = f(bc) - f(bd) =
= f(c/d) = f(q) = min{f(p),f(q)}

Agora, que funcoes f SATISFAZEM (i), (ii) e (iii) nos naturais, eu
ainda nao sei mesmo... Eu sei de uma que serve:

f(n) = expoente de 1999 na decomposicao de n em fatores primos

certamente funciona, mas nao sei se eh a unica.

Abraco,
Ralph



Re: Ajuda em problema

1999-12-21 Por tôpico Ralph Costa Teixeira


Oi, Eric.

Use este fato:

x/(x+1) = 1 - 1/(x+1)

em conjunto com

(x+1)/x = 1 + 1/x

e é claro note que

x/(x+1)=1/((x+1)/x)

Coragem, vai dar certo. :)

Abraco,
Ralph

Eric Campos wrote:
 
 Determine todas as funcoes f:R-R que satisfazem as condicoes:
 
 (i) f(-x)=-f(x)
 (ii) f(x+1)=f(x)+1
 (iii) f(1/x)=f(x)/(x^2) para x0